EM COMAT

अब Quizwiz के साथ अपने होमवर्क और परीक्षाओं को एस करें!

A 40-year-old male presents with a syncopal episode of unknown duration. The patient states that right before the event he began feeling dizzy and weak and that these symptoms have continued since regaining consciousness. An electrocardiogram reveals sinus bradycardia and shortly after the patients blood pressure is noted to be 80/60 mmHg and symptomatic again. The most appropriate pharmacologic management is A. adenosine B. amiodarone C. atropine D.diltiazem E. metoprolol

The correct answer is: C Atropine is an acetylcholine antagonist which functions in bradycardia by both up-regulating pacemaker activity at the sinoatrial (SA) node and hastening conduction through the atrioventricular (AV) node. This medication is first line therapy for symptomatic bradycardia, as in the above case with hypotension and acutely altered mental status. Other options considered if atropine fails include: dopamine, epinephrine, and transcutaneous pacing. Answer A: Adenosine is a very short acting medication that works by blocking the action of the AV node. When the affects wear off, normal excitability has been reestablished within the AV node and normal rhythmic contractions begin again. Adenosine would be the first step after vagal maneuvers are tried. This treatment would be ineffective if the source of hyper-excitability/re-entry is something other than the AV node. Obviously adenosine should not be used in any patient with pre-existing conduction problems, such as prolonged QT syndrome or any heart block, as it's action of delaying conduction will acutely worsen these conditions. Answer B: Amiodarone is a class III anti-arrhythmic agent that works mainly by slowing down the repolarization phase of conduction via potassium channel blockade. However, amiodarone is unique among anti-arrhythmias in that it has some action at every phase of cardiac conduction and could be classified anywhere from I-IV. This medication is typically used, in conjunction with other therapies, for ventricular fibrillation and ventricular tachycardia, which can sometimes be confused with wide complex SVT. Common tested side effects of amiodarone include pulmonary fibrosis and hypo or hyperthyroidism. As with adenosine, amiodarone should not be used in patient's with pre-existing heart blocks. Answer D: Diltiazem is a class IV anti-arrhythmic which functions by calcium channel blockade which shortens the "plateau" phase of conduction and thus decreases the heart's contractility. This medication is best used in an irregular rhythm narrow QRS complex SVT usually caused by atrial fibrillation or flutter. You would also consider diltiazem after vagal maneuvers and adenosine have failed in the treatment of regular interval narrow SVT. In the setting of bradycardia, diltiazem can cause induction of heart failure by decreasing contraction on top of an already decreased rate. Answer E: Metoprolol is a selective β1 receptor blocker that functions to slow cardiac rate by decreasing sympathetic activity to the heart. As with most beta blockers, metoprolol is a Class II anti arrhythmic agent. Along with diltiazem, metoprolol is mainly used for irregular rhythm narrow complex SVT with atrial fibrillation or flutter and is a common option for chronic suppression of these arrhythmias in conjunction with high blood pressure or congestive heart failure. In bradycardia, metoprolol will slow the heart rate down even more and can cause cardia arrest. Bottom Line: Atropine is the first line therapy for symptomatic bradycardia.

A 56-year-old male presents to the emergency department with chest pain. He says that he was eating at a restaurant when he suddenly developed chest pain with radiation to his back between his shoulder blades. Past medical history reveals uncontrolled hypertension and hyperlipidemia without a history of coronary artery disease. Social history is pertinent for a 25 pack-year history of cigarettes. Physical examination reveals that the heart has a regular rate and rhythm without murmurs, rubs, or gallops, clear lung sounds bilaterally, a soft, non-tender abdomen without masses. You note paravertebral muscle spasms and decreased range of motion of the thoracic vertebrae from T1-4. A plain film chest radiograph is obtained as shown in the exhibit. The most likely diagnosis is A. aortic dissection B. myocardial infarction C. myocarditis D.pneumothorax E. pulmonary embolism

The correct answer is: A This chest x-ray shows a widened mediastinum associated with chest and interscapular pain consistent with aortic dissection. An aortic dissection is the disruption of the aorta causing blood to dissect into the false lumen. Risk factors include advancing age, uncontrolled hypertension, connective tissue diseases such as Marfan syndrome, a bicuspid aortic valve, family history, stimulant use, recent surgery or cardiac catheterization, giant cell arteritis and others. Symptoms include severe neck, back, and/or chest pain described as ripping or tearing and maximal and abrupt at onset. Pain may migrate as a dissection migrates. There can also be associated nausea/vomiting diaphoresis along with neurologic symptoms if there is involvement of the carotid artery or spinal artery. Exam may show hypertension (although it is possible to be normotensive or hypotensive if there is a rupture present), a new harsh diastolic aortic murmur (secondary to a proximal aortic dissection), asymmetric extremity pulses or blood pressures, and neurologic findings if the carotid or spinal arteries are involved. You may see TART changes from T1-4 as a result of viscerosomatic reflexes from the underlying cardiovascular pathology. Workup includes and EKG to rule out myocardial infarction, chest x-ray, cardiac enzymes, and CT angiography. Chest x-rays are usually normal in the vast majority of patients with aortic dissections; however, findings associated with dissection include a widened mediastinum, loss of aortic knob, pleural capping, and aortic shadow extending greater than 5 mm from aortic calcification. CT angiography is the confirmatory test and has a high sensitivity and specificity. Treatment includes reducing shear forces by decreasing blood pressure (if hypertensive) and heart rate and immediate cardiothoracic surgery consultation for further management. Answer B: The patient in this example has chest pain and myocardial infarction should be included on the differential diagnosis. However, diagnosis of myocardial infarction is by cardiac enzymes (for NSTEMI) or EKG (ST elevation in STEMI). Chest x-ray is included in the workup for myocardial infarction to rule out other causes of chest pain. Answer C: Myocarditis is the inflammation of the myocardium. Most common causes are viral agents in the United States and Trypanosoma cruzi worldwide. Symptoms include flulike symptoms, chest pain, palpitations, shortness of breath and syncope or near syncope. Exam shows peripheral edema, JVD, rales, and sinus tachycardia. Workup includes EKG (may show low voltage, AV blocks, non-specific ST changes, and tachycardia), chest x-ray (may show cardiomegaly or pulmonary edema), CBC (may be normal or slight elevation of WBC), ESR (elevated), and cardiac enzymes (may be elevated). Treatment includes addressing the infectious etiology if present, supportive care, diuresis for findings of CHF and ACE inhibitors to reduce myocardial inflammation. Answer D: A pneumothorax is a collapsed lung that can be spontaneous or caused by a traumatic etiology. Symptoms include shortness of breath and/or chest pain. Signs include crepitus or tenderness to palpation if trauma present, and decreased breath sounds on the affected side. Workup includes a chest x-ray which will show absence of lung markings in the area of the collapsed lung. Treatment includes 100% non-rebreather oxygen for small spontaneous pneumothoraces and tube thoracostomy for larger pneumothoraces or traumatic pneumothoraces along with admission. Answer E: Pulmonary emboli can present with chest pain and shortness of breath. Chest x-ray, EKG, cardiac enzymes, and CT angiography are included in the workup. Chest x-ray findings in patients with pulmonary emboli include: a normal chest x-ray (in the majority of patients), Westermark's sign (a cut off of pulmonary arteries with dilation of the proximal pulmonary) and Hampton's hump (a wedge-shaped opacity in the periphery of a lung field indicative of a lung infarction secondary to a pulmonary embolism cutting off blood supply to a particular portion of the lung). Management includes admission and anticoagulation for hemodynamically stable patients. Hemodynamically unstable patients may require embolectomy and/or pharmacologic thrombolysis. Bottom Line: Patients presenting with chest pain, neck pain, back pain, neurologic deficits, unequal extremity pulses could have an aortic dissection. Chest x-ray may show mediastinal widening, pleural capping, loss of the aortic knob, or the aortic shadow extending 5 mm past the area calcification. However, most of the time chest x-ray findings are normal and the gold standard test for confirming aortic dissection is CT angiography of the chest.

A 74-year-old female presents to the emergency department with chief complaint of generalized weakness, headache, nausea and lightheadedness after sitting in a sauna. Vital signs reveal a blood pressure of 102/62 mmHg, a heart rate of 102/min, and a respiratory rate of 16/min. Physical examination reveals a diaphoretic female in mild distress. She is alert and oriented to person, place, and time, and a cardiovascular exam reveals a mild sinus tachycardia without murmurs, rubs, or gallops. Her lungs are clear to auscultation bilaterally and her abdomen is soft, non-tender, and without masses. The most likely diagnosis is A.heat cramps B. heat exhaustion C. heat stroke D. heat syncope E. malignant hyperthermia

The correct answer is: B Heat exhaustion is the most common type of heat related illness. It is differentiated from heatstroke by the fact that there is no change in the mental status. Symptoms include headache, nausea/vomiting, lightheadedness, weakness and irritability. Signs on exam include evidence of perspiration and a normal core temperature (although it may be elevated but is always less than 40°C). Diagnosis is based on clinical presentation although there may be abnormal electrolytes including hypochloremia, hyponatremia, and elevated BUN. Another differentiating lab factor from heatstroke is normal liver function tests. Management includes replacing fluids with electrolyte solutions and removing the patient from the heat source. Note that the patient in this is example has evidence of perspiration (diaphoresis) and has a normal mental status with some of the above symptoms. Answer A: Heat cramps are the painful contractions of large muscles that occur after exertion secondary to relative hyponatremia. This occurs when individuals exercise and replace fluids with hypotonic solutions (i.e., water) without replacing sodium content. Lab results shows normal electrolytes. Treatment is supportive and the condition is usually self-limited. Answer C: Heatstroke is the most serious heat related illness. There is associated high mortality rate due to the fact that homeostatic the regulatory mechanisms fail leading to hyperthermia and multiorgan failure. The two types of heatstroke are exertional and classic heatstroke. Exertional is a type of heatstroke that occurs in young individuals undergo heavy exertion in hot conditions. Classic it usually occurs in elderly individuals without access to air conditioning. The key clinical differentiating factor between heatstroke and heat exhaustion is the presence of altered mental status and/or CNS dysfunction. On exam the patient can also possibly have a seizure, the presence of an hydrolysis, core temperature greater than 40.5°C, cardiovascular dysfunction, and jaundice 24 to 72 hours later. Diagnosis is based on clinical presentation however lab values show elevation of AST/ALT and possible renal injury. Electrolyte disorders vary. Treatment includes correct electrolyte imbalances, rapid cooling mechanisms (removing cooling placing water on the skin and blowing a fan on the skin, cold water immersion, ice packs to various body sites, cold blanket and last-ditch efforts include cardiopulmonary bypass, gastric/pleural/bladder lavage). IV benzodiazepines should be used to prevent shivering. Complications include ARDS, renal failure, rhabdomyolysis, hepatic injury, DIC, cerebral edema and multi-organ dysfunction leading to death. Answer D: Heat syncope is associated with short-term or near loss of consciousness and typically occurs in a dehydrated patients secondary to vasodilation. The increase in peripheral vasodilation associated with an elevated ambient temperature can lead to the peripheral pooling of blood, causing postural hypotension and subsequently a syncopal event. Manifestations include visual changes, bradycardia, light-headedness with the core body temperature either normal or only mildly elevated. Answer E: Malignant hyperthermia is a genetic disorder in which skeletal muscle has instability when exposed to certain anesthetic agents including succinylcholine. This leads to a massive release of calcium and causes muscle rigidity and hyperthermia. Diagnosis is clinical and treatment includes dantrolene. Bottom Line: Heat exhaustion is differentiated from heat stroke by clinical presentation. There is the presence of perspiration, core temperature is usually normal, there is no abnormal liver function tests, and the patient has a normal mental status. There may be hyponatremia and hypochloremia present. Treatment includes electrolyte replacement solutions and removal from heat.

A 52-year-old female presents to the emergency department via EMS with a chief complaint of altered mental status. The patient was found by her boyfriend in an apartment lying on the floor next to a suicide note and multiple empty pill bottles. Per EMS, the patient had a blood glucose of 112 mg/dL at the scene. In the emergency department her vitals reveal: Blood pressure 92/44 mmHg Heart rate 128/min Respirations 8/min Oxygen saturation 89% on a 100% non-rebreather Physical examination reveals she now only withdraws to painful stimulation, is not opening her eyes to pain, and is groaning incomprehensible sounds. She is provided naloxone, but this has no effect on her mental status. Question 1 of 2 in this set The patient's current Glasgow coma scale score is A. 3 B. 6 C. 7 D.11 E. 14

The correct answer is: C The Glasgow coma scale (GCS) is a neurologic scale used to define a patient's level of consciousness. The score ranges from minimum of 3 to a maximum of 15. There are three components of the Glasgow coma score including: eye-opening, verbal response, and motor response. The patient in this scenario scores a one for no eye-opening, a two for groaning incomprehensible sounds, and a four for withdrawing to pain. Therefore, her Glasgow coma score is a seven. Bottom Line: The Glasgow coma scale (GCS) is a neurologic scale used to define a patient's level of consciousness. The score ranges from minimum of 3 to a maximum of 15. There are three components of the Glasgow coma score including: eye-opening, verbal response, and motor response. The patient in this situation has a history of a suicide attempt by overdosing on unknown medications. Common medications that may cause a depressed level of consciousness include: opioids, barbiturates, benzodiazepines, tricyclic antidepressants, and different type of seizure medications. Initial management of overdose patient's include the "ABCs:"airway, breathing, and circulation." The patient in this situation has a GCS of less than 8 which is an indication for intubation along with the fact that she is hypotensive, has shallow respirations, and is hypoxic on a 100% non-rebreather. In this situation the first step of management include securing the airway via intubation. Answer A: A CT scan is indicated in the workup of a patient with altered mental status to rule out other pathology including intracranial hemorrhage, mass effect, etc. However, rapid sequence intubation is more imperative in this patient's management sequence. Answer B: An EKG is indicated in the management of overdose patient's as certain EKG changes can lead the practitioner to which type of poison/drug the patient may have ingested. However, intubation is more imperative in this patient's management sequence. Answer D: The administration of thiamine and glucose is indicated in treatment of a comatose patient with a coma cocktail which includes: dextrose, oxygen, Naloxone, and thiamine (mnemonic: DON'T). However, rapid sequence intubation is more imperative in this patient's management sequence. Answer E: A toxicology screen is indicated in a poisoned/overdosed patient. However this usually does not help with management of the patient. Airway is of paramount importance in this situation. The workup of a poisoned/overdosed patient includes but is not limited to: CBC, CMP, calculation of the anion gap, calculation of the osmole gap, a pregnancy test, acetaminophen and salicylate level, EKG, and discussion with EMS personnel and family members. Bottom Line: Managing the airway is the first goal in taking care of patients with severely altered mental status.

A 16-year-old female presents to the emergency department with nausea, vomiting, and right upper quadrant abdominal pain. History reveals the patient left a suicide note next to an empty bottle of over-the-counter analgesic medication. Physical examination reveals a diaphoretic patient in mild distress with right upper quadrant abdominal pain. Vitals reveal blood pressure 135/85 mmHg, heart rate 72/min, respiratory rate of 16/min, oxygen saturation 98% on room air, and a temperature 37.1oC (98.7oF). The most appropriate pharmacologic therapy is A. flumazenil B. glucagon C. N-acetylcysteine D.naloxone E. penicillamine

The correct answer is: C This patient is showing the early signs of acetaminophen toxicity. It causes centrilobular hepatic necrosis. If ingested in large amounts the patient may require a liver transplant. In the United States and England it is the most common case of acute liver failure and leading indication of liver transplant in patients with drug induced liver failure. In the first few hours the patient may present asymptomatic and then progress to nausea/vomiting, pallor, diaphoresis and progress to have right upper quadrant pain. After several hours to days (72-96 hours) the liver starts to break down and necrosis occurs causing scleral icterus and jaundice. The treatment is N-acetylcysteine counteracts the toxicity. This works directly and indirectly. Directly it detoxifies N -acetyl-p -benzoquinoneimine to non-toxic metabolites and indirectly it is converted to cysteine allowing glutathione stores to be replenished and conjugates the toxic metabolite of acetaminophen. Answer A: Flumazenil is used to treat benzodiazepine overdose. It works as a benzodiazepine antagonist. It needs to be used in caution as it may cause seizures in patients who take benzodiazepines chronically. Benzodiazepine overdose is a patient who is more lethargic and decreased respiratory rate and suspect this with a history of anxiety. Answer B: Glucagon is used as a beta-blocker overdose. Beta-blocker overdose would have a patient with hypotension and bradycardia. They may be altered due to decreased cerebral blood flow. Answer D: Naloxone is used to treat opioid overdose. It works as an opioid antagonist. An opioid overdose would have someone who is lethargic, decreased respiratory rate, and pin-point pupils. Suspect this with chronic pain patients. Answer E: Penicillamine works as a chelator. It is used in copper toxicity. Acute copper toxicity is seen as nausea and vomiting, especially hematemesis. Melena may be observed, along with lethargy, and jaundice. The history would need to explain some type of exposure; whether it is work, environmental, or supplements. Bottom Line: Antidotes are commonly tested. Suspect a young person with nausea and vomiting leaving a suicide note to be clues of acetaminophen toxicity. This is further understood with the over-the-counter analgesic medication and the right upper quadrant pain.

A 45-year-old female presents to the emergency room with altered mental status. History reveals that the patient ingested an unknown substance approximately three hours prior to arrival. The patient is nauseous, has vomited, and has been defecating and urinating almost continuously since presentation. Vital signs reveal a heart rate of 98/min, respiratory rate 24/min and blood pressure of 115/89 mmHg. Physical examination reveals an obese, diaphoretic female who appears delirious with a Glasgow coma score of 12. Activated charcoal, 50 grams by mouth, is promptly provided without response. This initial intervention was unsuccessful because the most likely offending agent is A. carbamazepine B. glipizide C. lithium D.phenobarbital E. theophylline

The correct answer is: C This patient is suffering from acute lithium toxicity. Lithium is commonly used in psychiatric disorders such as bipolar disorder and acute episodes of mania. Activated charcoal in many instances works well for adsorption of an ingested toxin. However, activated charcoal does not adsorb certain compounds. Absorption by the intestines of these compounds are not impeded and will diffuse across the intestinal lining to make it into the body's circulation. The drugs that activated charcoal does not work for are as follows: Ionic compounds: lithium, calcium, potassium, sodium, magnesium, fluoride and iodide Heavy metals: arsenic, lead, mercury, iron and zinc cadmium Acids/Bases Hydrocarbons: alkenes, alkanes, alkyl halides, and aromatic compounds Essential oils Alcohols: acetone, ethanol, ethylene glycol, methanol and isopropanol Activated charcoal is otherwise relatively safe but caution should always be used if a perforation of the gut is suspected. In addition, if endoscopy is a possibility charcoal should not be used as it will coat the lining of the GI tract and make seeing any pathology impossible. Answer A: Carbamazepine is a medication commonly used in the treatment of seizures, bipolar disorder and trigeminal neuralgia. The mechanism of action of this drug is unknown for bipolar disorder and trigeminal neuralgia, however it works for preventing seizures by reducing post-tetanic potentiation and decreasing seizure spread. As this drug is a tricyclic derivative an overdose of this medication would present with an anticholinergic toxidrome. The molecule itself is large enough (activated charcoal works best for molecules between 100-1000 Da) and would readily be absorbed by activated charcoal and thus this treatment would be effective in aiding elimination. Answer B: Glipizide is an oral hypoglycemic agent in the sulfonylurea category. This patient's presentation does somewhat match an overdose of a sulfonylurea. She exhibits nausea, vomiting and diaphoresis which could be attributed to hypoglycemia. In addition the polyuria with polydipsia may have been attributed to uncontrolled diabetes mellitus, however lithium toxicity will acutely present like this as well. Lithium interacts with the kidney's ability to concentrate urine and is the most common drug causing nephrogenic diabetes insipidus. Glipizide would readily be adsorbed by the activated charcoal. Answer D: Phenobarbital is in the barbiturate class of seizure medications. Acute overdose would produce symptoms similar to alcohol intoxication at mild to moderate levels and in severe levels would present in a range of neurologic depression from stupor to coma. These patients can present with severe respiratory depression and prompt gastroenteric decontamination with activated charcoal will prove beneficial. Activated charcoal should be given to alert and cooperative patients as it can pose a dangerous risk for aspiration. Multi-dose charcoal may reduce serum levels of barbiturates but no significant difference in outcomes have been noted. Answer E: Theophylline is a methylxanthine used in asthma/COPD maintenance and acute episodes of bronchospasm. Theophylline does have a very narrow therapeutic window and has fallen out of favor for this reason. Methylxanthines include other drugs like caffeine and nicotine. Toxicity would present with nausea, vomiting, headache, agitation, possible seizures, and sinus tachycardia. Activated charcoal should be considered in the patient with a potentially life threatening overdose and no contraindications such as an unsecured airway, nausea/vomiting, ileus or bowel obstruction, or the need for emergent endoscopy. Bottom Line: Activated charcoal adsorbs toxic substances, thus inhibiting GI absorption. It can be very beneficial in the acutely poisoned patient, however, it is important to know what substances will be least effective.

A 43-year-old female presents via EMS with the chief complaint of an overdose on her antidepressant medication. A bottle of amitriptyline was found empty in the house along with a suicide note. Per her husband, he saw her take a "couple handfuls of pills" approximately 1 hour ago. She was extremely lethargic when EMS arrived, and the patient was subsequently intubated her for airway protection. The most likely additional finding upon further evaluation is A. an AST of 868 IU and ALT of 967 IU B. diarrhea, vomiting, lacrimation, diaphoresis, and miosis C. formation of NAPQI metabolite D. prolongation of the QRS segment E. sedation, anxiolysis, muscle relaxation

The correct answer is: D Overdose of amitriptyline, which is a tricyclic antidepressant (TCA), leads to a sodium channel blockade causing prolongation of phase 0 of the cardiac action potential. This leads to quinidine-like effects which include: QRS widening, decreased contractility, and hypotension. QRS prolongation >100 msec can lead to ventricular dysrhythmias, cardiovascular collapse, and death. It is recommended that sodium bicarbonate be administered in patients with QRS prolongation greater than 100 msec to prevent these effects from occurring. Answer A: AST and ALT are liver enzymes and the values of 868 and 967 respectively are elevated levels of these enzymes. TCA overdose is not associated with elevation of liver function tests - this is associated with acetaminophen overdose. Acetaminophen overdose if untreated is associated with 4 stages of toxicity. Stage 1 occurs in the first 24 hours and is associated with nonspecific symptoms such as anorexia, malaise, nausea and vomiting. Stage 2 occurs in the first 72 hours and is associated with elevation of the liver enzymes (AST/ALT), right upper quadrant pain, elevation of the INR and bilirubin. Stage 3 occurs between 72 and 96 hours and is associated with the peak injury pattern and sequelae. It is associated with peak elevation of AST/ALT, hepatic necrosis with resulting liver failure. Stage 4 is the recovery phase which occurs 4 days 2 weeks post-ingestion and is associated with AST/ALT enzymes returning to baseline and regeneration of the liver. Answer B: The findings of diaphoresis, miosis, vomiting, diarrhea, lacrimation are all cholinergic effects. TCA overdose is associated with anticholinergic effects. This can lead to findings of dry, flushed skin, CNS excitation or coma, seizures, tachycardia, urinary retention, mydriasis, or hyperthermia. (Mnemonic: mad as a hatter, red as a beet, dry as a bone, hot as a hare, blind as a bat). Answer C: In normal therapeutic use of acetaminophen, 90% of acetaminophen is combined with sulfate or glucuronide to form nontoxic metabolites. The remaining acetaminophen is oxidized by the cytochrome P450 system to form a toxic metabolite, NAPQI (N-acetylpbenzoquinoneimine). Liver stores of glutathione combine with NAPQI to form additional nontoxic metabolites. However, in acetaminophen overdose the sulfate and glucuronide conjugation systems are overwhelmed and a large amount of NAPQI is formed. Glutathione is therefore also overwhelmed which leads to free NAPQI binding to liver proteins causing hepatotoxicity (centrilobular). Answer E: Sedation, anxiolysis and muscle relaxation are GABA agonistic effects. TCA overdoses are associated with GABA receptor blockade. Gamma-aminobutyric acid (GABA) is the chief inhibitory neurotransmitter in the human central nervous system and is responsible for inhibiting neuronal excitability. Therefore, inhibition of the inhibitory neurotransmitter, leads to neuronal excitation which can lead to seizures in TCA overdose. Treatment of seizures in TCA overdoses include benzodiazepines such as Lorazepam which act to enhance GABA effects. Bottom Line: Tricyclic antidepressant overdose is associated with anticholinergic effects, and cardiac effects including prolongation of the QRS segment on electrocardiogram.

A 4-year-old male is brought into the emergency department by his parents after reportedly falling off of a couch and landing on his side. The child is holding his right forearm and will not allow it to be examined. After calming him and examining the forearm you feel a mild deformity and associated edema. Plain radiographs of the forearm reveal a spiral fracture in the mid-shafts of the ulna and radius. The most appropriate management is to A. apply a radial gutter splint B. apply an ulnar gutter splint C. obtain an MRI of the hand D. report to the Division of Child and Family Services E. wrap with a compressive dressing and sling

The correct answer is: D Report to Division of Child and Family Services (DCFS). DCFS is a governmental agency, also known as Child Protective Services, that is the agency responsible for investigating and if necessary removing a child from an abusive home. Physicians are required to notify this agency in any suspected abuse case of anyone under the age of 18. Different states have different laws regarding the specifics of abuse; all 50 states do require that a minimal verbal report to the police department be made. A spiral fracture as indicated by the radiograph is highly indicative of child abuse and suspicion should be high when these are found, especially if the story does not match the mechanism of injury. For example if the parent report the child fell out of their crib with their arm caught and rotated on the fall, that would fit the mechanism of injury. However this child's story of falling straight off of the end of a couch would be unlikely to produce a fracture and extremely unlikely to produce a spiral fracture. Answers A & B: Splints are a common method of securing and stabilizing a fracture until follow-up with orthopedics is possible. Splints are made from fiberglass that after being moistened will harden into whatever form they are shaped, making them ideal to fit each individual. These can be uniquely shaped to form to any extremity and isolate any fracture. An ulnar gutter splint is placed along the ulna extending from the proximal third of the forearm to the metacarpal-phalangeal joint. This splint is ideally suited to protect the ulna and immobilize the wrist. A radial gutter splint is placed along the radius and is indicated for scaphoid and distal radius fractures as well as DeQuervain's disease. The proper splint for this child's injury would be a "sugar tong" splint. Once hardened these resemble a pair of sugar tongs. The splint is place along the posterior aspect of the forearm and with the elbow flexed to 90 degrees it continues along the anterior aspect of the forearm. It extends from the wrist joint posteriorly to the wrist joint anteriorly and prohibits supination and pronation of the forearm. This immobilization would then prohibit the fracture from becoming unstable. Answer C: This option would be best for evaluating a scaphoid fracture suspect for avascular necrosis. This imaging modality is highly sensitive and specific for scaphoid fractures when standard radiography is not definitive. Typically these fractures occur with a fall on outstretched hands, not the twisting motion expected in this patient. Answer E: The compressive dressing would be an ACE (elastic) wrap and may help with swelling but will not stabilize the fracture. While a sling may help and prevent excessive movement of the forearm, a young child would not have the facultative reasoning to be able to constantly remember to keep his arm steady and therefore would be high risk for displacing his fracture. A more permanent splint and eventual cast would be the best treatment option. Bottom Line: Suspected cases of child abuse in which a child demonstrates an injury congruent with child abuse must be reported to the proper authorities.

A 57-year-old female presents to the emergency department by ambulance for altered mental status. On presentation, the patient is actively vomiting but appears to be moving all extremities. A CT scan of the head is obtained as shown in the exhibit. The most likely diagnosis is A. epidural hematoma B. intraparenchymal hemorrhage C. ischemic stroke D. subarachnoid hemorrhage E. subdural hematoma

The correct answer is: D Subarachnoid hemorrhage has noted characteristic findings on CT, which is shown in the exhibit. On CT scans, subarachnoid hemorrhage (SAH) appears as a high-attenuating, amorphous substance that fills the normally dark, CSF-filled subarachnoid spaces around the brain. The normally black subarachnoid cisterns and sulci may appear white in acute hemorrhage. These findings are most evident in the largest subarachnoid spaces, such as the suprasellar cistern and Sylvian fissures. Subarachnoid hemorrhage typically presents as a sudden headache with maximal intensity at onset. It is usually described as a "thunderclap headache" and "the worst headache of my life". Associated symptoms may include nausea and vomiting, meningismus, or loss of consciousness. Sometimes there will be a "sentinel" headache occurring 6-20 days in advance of the subarachnoid hemorrhage which usually resolves spontaneously. Most subarachnoid hemorrhages are secondary to aneurysm rupture and may require neurovascular intervention to stop the bleeding. Example of SAH: Blood noted in suprasellar and ambient cisterns. It can also collect in the various sulci. If the blood does not look like it is in the epidural, subdural, or intraparenchymal space, it is likely in the subarachnoid space. The picture below is the typical image seen with SAH. Answer A: This is not an epidural hematoma. Typically there should be a "lens"-shaped area of hemorrhage noted on CT, and history usually involves some type of trauma to the temporal area. Epidural hematoma is caused by trauma to the middle meningeal artery. Answer B: There is no bleeding noted within the brain parenchyma. The blood is within the Sylvian and Intrahemispheric fissures as well as the Suprasellar and Ambient cisterns. Answer C: CT scan shows obvious blood within the subarachnoid space, this is not an ischemic stroke. Answer E: Subdural hematoma is usually associated with a "crescent"-shaped area of hemorrhage noted on CT. Bottom Line: In a patient with sudden loss of consciousness, altered mentation, and vomiting, intracranial hemorrhage should be at the top of the differential. Know the CT findings of subarachnoid hemorrhage. COMBANK Insight : It is imperative that you review head imaging, EKGs, and plain x-rays prior to exam day. You will likely have several items that will rely on your ability to interpret these common ER tests.

A 26-year-old female at 28 weeks' gestation presents with painless vaginal bleeding. Vitals are normal. The most appropriate next step in management is A. blood transfusion B. digital vaginal examination C. fetal fibronectin D. nonstress test E.OB consultation for Cesarean section

The correct answer is: D The causes of vaginal bleeding in the second half of pregnancy include abruptio placentae, placenta previa, premature labor, and lesions of the vagina or lower genital tract. Clinical features of abruptio placentae include painful vaginal bleeding, severe uterine pain or tenderness, uterine hypertonicity, and hypotension. A high index of suspicion for abruptio placentae should be maintained even with minimal or no bleeding and only minimal abdominal cramping. In contrast, patients with placenta previa present with painless vaginal bleeding of bright red blood, which should be differentiated from the normal passage of blood-stained mucus that occurs near the onset of labor. Severe bleeding or hemorrhaging from any of these causes can severely harm both mother and fetus, and can be fatal. A digital examination should never be performed in patients who present with vaginal bleeding in the second half of pregnancy because this may precipitate catastrophic hemorrhage and death, particularly in cases of placenta previa. Ultrasound should be used to identify the position of the placenta. The fetus should also be monitored with a nonstress test (NST). A non-reassuring test (nonreactive NST, positive CST) is sometimes associated with adverse fetal or neonatal outcomes, while a reassuring test (reactive NST, negative CST) is usually associated with a neurologically intact fetus. If the fetus has a non-reassuring NST the management of this patient may change drastically. Answer A: There is no indication to prepare for transfusion at this point in time as to the severity of the vaginal bleeding is not specific and the vitals appear to be normal. While it would be beneficial to prepare for a transfusion if the patient is hemodynamically unstable, this is not the most appropriate management at this time. We need to determine the etiology of the bleeding first. Alternatively, a CBC would be appropriate. Answer B: A digital examination should never be performed in patients who present with vaginal bleeding in the second half of pregnancy without identifying the location of the placenta because doing so may precipitate catastrophic hemorrhage and death. Answer C: Fetal fibronectin (fFN) is used to distinguish women in true preterm labor from those with false labor. FFN is an extracellular matrix protein present at the decidual-chorionic interface. Disruption of this interface due to subclinical infection or inflammation, abruption or uterine contractions releases fFN into cervicovaginal secretions, which is the basis for its use as a marker for predicting spontaneous preterm birth. Answer E: Indications for cesarean section include conditions that benefit the mother like repeat cesarean delivery, obstructive lesions, and pelvic abnormalities; conditions that benefit the fetus like infections, malpresentations, congenital malformations or skeletal disorders; and conditions that benefit both mother and fetus like abnormal placentation, cephalopelvic disproportion, or situations in which labor is contraindicated. Bottom Line: Evaluation of vaginal bleeding in the latter half of pregnancy should first include ultrasound to evaluate position of the placenta as well as an NST to monitor fetal cardiac status.

A 38-year-old female presents to the emergency department with abdominal pain, nausea and vomiting. History reveals a very irritable patient with complaints of hearing difficulty and ringing in her ears. Vitals reveal blood pressure 138/88 mmHg, heart rate 135/min, respiratory rate 26/min, oxygen saturation 96% on room air, and temperature of 36.2oC (97.2oF). Which of the following is the most likely etiology? A. acetaminophen B. barbiturate C. benzodiazepine D.opiate E. salicylate

The correct answer is: E Acute salicylate toxicity presents with abdominal pain that may be epigastric in origin. Common GI complaints are nausea and vomiting. Common neurologic findings are very irritable patients, possibly anxiety or acute psychoses. A common side effect of salicylate toxicity is tinnitus (ringing in the ears). The key to this presentation is in the vitals, a tachycardic patient with tachypnea. Patients with acute salicylate toxicity initially present with respiratory alkalosis (tachypnea) before they present with a metabolic acidosis. Checking an arterial blood gas is very helpful. If caught within the first hour treatment is with activated charcoal. The patient is also placed on sodium bicarb to cause alkalization of the urine. Answer A: Acetaminophen toxicity presents with acute abdominal pain with nausea and vomiting. Right upper quadrant pain is the typical location. The patient presents with signs of acute liver failure (scleral icterus, jaundice, altered mental status). The treatment is N-acetylcysteine. Answer B: Barbiturate toxicity presents very similar to benzodiazepine toxicity. However, there is not a reversal agent. Answer C: Benzodiazepine toxicity would present with a decrease respiratory rate and somnolent patient. The patient's pupils would not be pinpoint. The reversal agent is flumazenil. Answer D: Opiate toxicity presents as a lethargic patient with a decrease respiratory rate. Physical exam would reveal a somnolent patient, that is breathing slow, and pin point pupils on exam. Reversal would be with naloxone. Bottom Line: Acute salicylate toxicity initially presents with respiratory alkalosis before metabolic acidosis.

A 40-year-old healthy male with no past medical history presents with left-sided facial weakness that started yesterday and has been constant. History reveals it is associated with pain behind the left ear and decreased lacrimation from the left eye. Physical examination reveals inability to raise the left forehead, flattening of the left nasolabial fold, ptosis left eye, and preferential right mouth movements. Neurologic examination is otherwise normal. The most appropriate management is A. aspirin and observation B. carotid ultrasound C. CT without contrast D. MRI with contrast E. prednisone and valacyclovir

The correct answer is: E Bell's palsy is a dysfunction of cranial nerve VII involving the ipsilateral upper and lower face. Possible etiologies include herpes zoster, Lyme disease, syphilis, Epstein-Barr virus, CMV, HIV, diabetes mellitus, and hypertension. Treatment of Bell's palsy remains controversial, however, the most recent literature concludes that either corticosteroids alone or antivirals with corticosteroids are beneficial. Note: Genu VII Bell's palsy is a stroke involving cranial nerve VI and the ipsilateral cranial nerve VII as it "genuflects" around the nucleus of cranial nerve VI. This results in a presentation identical to a typical Bell's palsy but with the additional finding of an inability to abduct the ipsilateral eye due to cranial nerve VI palsy. Therefore, it is of the utmost importance to conduct extraocular motion testing when evaluating for Bell's palsy. Answer A: Neither medical therapy with aspirin nor an observational stay are warranted in the management of Bell's palsy. Answer B: Carotid ultrasound is commonly used to assess risk for stroke. It is not part of the workup of Bell's palsy. Answers C & D: A comprehensive history and physical examination will lead to the diagnosis of a Bell's palsy. Imaging is not necessary unless the paralysis is prolonged, lasting greater than 8-10 weeks, or if there is concern for tumor or vascular compression. Bottom Line: Treatment of Bell's palsy should consist of either corticosteroids alone or antivirals with corticosteroids. COMBANK Insight : Commonly, the orbicularis muscles are involved in Bell's palsy resulting in the incomplete closure of the eyelid on the affected side leading to corneal exposure keratitis. In order to prevent the breakdown of corneal epithelium, lubricant eye drops and patching of the affected eye should be employed. An ophthalmology referral is also warranted

A 27-year-old female presents after a seizure. Laboratory studies reveal the following: sodium 115 mEq/L potassium 3.5 mEq/L chloride 115 mEq/L bicarbonate 25 mEq/L BUN 18 mEq/L creatinine 1.0 mg/dL serum osm low urine osm high The most likely cause is A. adverse effect of diuretics B. metabolic acidosis, uncompensated C. psychogenic polydipsia D. renal tubular acidosis, type 2 E. syndrome of inappropriate antidiuretic hormone secretion

The correct answer is: E The most notable cause of euvolemic hyponatremia is the syndrome of inappropriate secretion of antidiuretic hormone or SIADH. Etiologies include trauma, tumors, infection, cerebral vascular accident like subarachnoid hemorrhage, chronic obstructive pulmonary disease, and tuberculosis. The diagnosis is characterized by several criteria including hypotonic hyponatremia, inappropriately elevated urinary osmolality (>200 mOsm/kg), elevated urinary sodium (>20 mEq/L), and clinical euvolemia. It is correctable with water restriction. If hyponatremia is severe (< 115 mEq/L) or associated with symptoms (coma, seizures), administration of 3% saline solution should be considered at 25 to 100 mL/hr. The rise in sodium should be no greater than 0.5 to 1.0 mEq/L per hour. If seizures are present, this can be increased to 1 to 2 mEq/L per hour. Answer A: Diuretic use is associated with hypotonic hyponatremia. Serum osmolality and urine osmolality are low. Answer B: Metabolic acidosis is a condition where the body produces excessive quantities of acid and is associated with low pH due to increased production of hydrogen ions and low bicarbonate due to inability of the kidney to form bicarbonate. Answer C: Psychogenic polydipsia is a less common cause of euvolemic hyponatremia related to a patient's increasing fluid intake due to the sensation of having a dry mouth. It is often associated with mental disorders or rather the treatment of mental disorders with medications like phenothiazines. Laboratory workup reveals low serum osmolality and low urine osmolality. Answer D: Renal tubular acidosis type 2 is caused by a failure of the proximal tubular cells to reabsorb filtered bicarbonate from the urine, leading to urinary bicarbonate wasting and acidemia. It is usually associated with Fanconi syndrome in which there are abnormalities like phosphaturia, glycosuria, aminoaciduria, uricosuria, and tubular proteinuria. Bottom Line: SIADH is characterized by elevated urinary osmolality, elevated urine sodium, and euvolemic hyponatremia.

Corneal abrasion

A corneal abrasion can occur when a patient gets a foreign body in their eye or accidentally scratches their cornea with an object. Symptoms include pain and a foreign body sensation along with eye redness. There can also be decreased visual acuity if the abrasion is central. Exam includes fluorescein staining with slit lamp examination to detect epithelial defect/foreign bodies, eversion of the upper and lower eyelids to detect foreign bodies, and visual acuity testing. Treatment includes removal of foreign bodies and associated rust rings, topical antibiotics and ophthalmology follow-up.

A 2-year-old male presents in November with a two-day history of upper respiratory infection, a rectal temperature of 39.1°C (102.4°F), and the sudden onset of inspiratory stridor, bark cough, and mild retractions. Question 1 of 2 in this set The most likely diagnosis is A. acute asthma exacerbation B. cystic fibrosis C. laryngomalacia D.spasmodic croup E. viral laryngotracheal bronchitis

The correct answer is: E Viral laryngotracheal bronchitis, also known as croup, is a very common upper airway disease affecting children 6 months to 6 years old. As the name implies, the etiology is often viral with most cases being caused by parainfluenza virus, although Staphylococcus aureus and Streptococcus pneumonia have been known to cause bacterial croup as well. Children will classically present in late autumn to early winter with inspiratory stridor, fever, retractions, and decreased pulse oxygen saturation. Diagnosis is made clinically, although plain chest films will show the classic "steeple sign" due to narrowing of the trachea. Answer A: Asthma is an obstructive pulmonary process common in the pediatric population with a multi-faceted etiology including environmental and genetic factors. Acute exacerbation can be caused by allergen exposure and leads to bronchospasm with associated wheezing and shortness of breath. Asthma exacerbations can certainly have retractions and coughing, inspiratory stridor and fever are not consistent with this diagnosis. Additionally, the child would likely have had a history of pulmonary problems. Answer B: Cystic fibrosis is an autosomal recessive disorder that is caused by a mutation in the CTFR gene which is involved in sweat and mucus production. This disease affects multiple organs, but specifically the lungs via excess mucus production and decreased bronchial cilia action leading to mucus plugging and bronchiectasis. These chronic mucus plugs can also lead to bacterial pneumonia. In this scenario, fever and cough can both occur with pneumonia, but stridor is specific to the upper airways/laryngotracheal area which neither cystic fibrosis nor pneumonia sequelae particularly affect. Additional, this child was previously healthy which would not be the case in cystic fibrosis. Answer C: Laryngomalacia is a congenital maldevelopment of the laryngeal cartilage leading to supraglottic collapse and associated stridor. It is the most common cause of stridor in infants and typically occur at 4-6 weeks of age and then regress by the second year. In this scenario, stridor is associated with laryngomalacia, but the fever, patient's age, and acute onset make this diagnosis unlikely. Answer D: Spasmodic croup is the exact same pathology as croup or viral laryngotracheobronchitis with upper airway involvement, viral origin and stridor. However, spasmodic croup is a short interval of spasm without associated fever or retractions. Bottom Line: Inspiratory stridor, fever, retractions, and decreased oxygenation levels in a child are classic symptoms associated with viral croup. The normal pulse oxygen saturation and lack of retractions are consistent with mild croup, which can be managed by close observation at home and with humidified air. The newest guidelines do not recommend humidified air, however, keep in mind that the exam is not up to date on current literature. Viral croup is self-limiting and usually last 2 days. Answer A: This patient has mild croup and does not require admission. Answer B: Albuterol is a β2-agonist used to reduce bronchospasm in cases of asthma exacerbations. The respiratory complications in croup are due to bronchial swelling via inflammation from viral infection; albuterol is not an effective treatment in this setting. Additionally, in this scenario, the pulse oxygen saturation in 99% indicating no respiratory compromise or need for further airway function. Answer D: Antibiotics are an inappropriate therapy for this patient as his disease is almost certainly viral in origin. Historically, physicians have over prescribed antibiotics, specifically giving them in the setting of viral illness. This overuse has contributed to the emergence of significant bacterial resistance to even the strongest of antibiotics. Answer E: Racemic epinephrine is an emergent option and would be more appropriate for patients with severe croup with significant stridor leading to respiratory compromise. Bottom Line: Mild cases of croup must be observed as symptoms can worsen acutely. Humidified air was traditionally recommended in the past, however, new literature does not recommend this. Keep in mind the exam is not current with new recommendations.

A 32-year-old female presents with fatigue and decreased urine output. Laboratory studies reveal a creatinine of 2.2 mg/dL and a baseline of approximately 1.2 mg/dL. Question 1 of 2 in this set Which of the following results will confirm a prerenal etiology in this patient? A. blood urea nitrogen/creatinine ratio of > 20 mg/dL B. fraction of filtered sodium > 2% C. granular casts in urine sediment D.urine osmolality <400 mOsm/kg water E. urine sodium >40 mEq/L

The correct answer is: A There are 3 classifications of renal pathology: prerenal, renal, and postrenal. Typically, prerenal causes of acute kidney injury will have a BUN/Cr ratio >20. Causes all have something to do with decreased blood flow to the kidneys, wether it is due to hypovolemia, low cardiac output, or constriction of the renal arteries. Treatment is aimed at correcting the cause of decreased blood flow to the kidneys. If hypovolemia is the problem, the patient needs IV fluids or blood products to replace the intravascular volume. If low cardiac output is the problem, the patient may need inotropic agents to help the heart pump stronger, or decreased systemic vascular resistance to allow a weak heart to pump more efficiently. Lastly, if the problem is renal artery stenosis, the patient would benefit from medical therapy and possibly revascularization therapy as well if that fails. Answer B: FENa of >2% is associated with acute tubular necrosis or other causes of intrinsic kidney damage. The reason the Na is elevated can be secondary to damaged glomeruli which allow more sodium to become excreted, or secondary to volume overload, which also allows for Na excretion to get rid of excess water. Typically FENa is <1% in prerenal causes because the kidneys are trying to retain water to increase pressure and ultimately perfusion to the kidneys. Answer C: Granular casts in the urine can be indicative of chronic renal disease, but "muddy brown casts" are associated with acute tubular necrosis. Answer D: Prerenal kidney pathology is usually associated with a urine osmolality of >400mOsm/kg water because the kidneys are trying to retain water, which in turn causes concentration of the urine. Answer E: In prerenal kidney injury, urine sodium concentration should be < 20mEq/L, and this is because the kidneys are trying to retain Na in order to retain water. Bottom Line: BUN/Cr ratio >20mg/dL is usually associated with a prerenal cause of kidney injury. This is secondary to the passive reabsorption of urea along with sodium in order for the kidney to try and retain water. Creatinine is not reabsorbed, which is why the BUN/Cr ratio will continue to increase until the underlying cause of the kidney injury is resolved. However, keep in mind that BUN will also be elevated in patients with GI bleeding. Viscerosomatic reflexes can vary by the area of the spine affected. The parasympathetic nervous system is noted to be at the cervical and sacral spinal levels, whereas the sympathetic nervous system is found at the thoracic spinal levels. The vagus nerve which is related to C1 and C2 dysfunction innervates the kidney and upper ureters, and can cause a viscerosomatic reflex involving these specific areas. The vagus nerve also affects the ovaries and testes as well as the ascending and transverse colon. The pelvic splanchnic nerves from S2-S4 affect the lower ureter and bladder, uterus, prostate, genitalia, descending colon, sigmoid colon, and the rectum.

Central renal artery occlusion

A central retinal artery occlusion is another cause of painless unilateral vision loss. It is sudden and onset and occurs in patients with the following risk factors: temporal arteritis, trauma with fat embolus, patients with sickle-cell disease or diabetes, hyperviscosity syndrome (multiple myeloma), and atherosclerotic plaque embolization to the retinal artery (usually from the internal carotid artery). Symptoms include sudden onset unilateral vision loss and on fundoscopic exam there is evidence of a cherry red macula and a pale spot between the macula and the optic disc. The goal of therapy is to move the clot into a distal retinal artery branch to decrease the size of the infarct. This can be achieved via digital massage of the orbit to help dislodge the clot and by inspiring carbon dioxide (by having the patient breathe into a paper bag) to dilate the distal retinal arterioles. Emergent ophthalmology consult is required.

A 65-year-old Caucasian male presents to the emergency department with abdominal pain that began suddenly 2 hours prior to arrival and has been worsening since onset. The pain is in the midline and is associated with some mild nausea. Past medical history is significant for nicotine dependence and hypertension. Vital signs reveal a blood pressure of 80/45 mmHg and heart rate of 115/min. What is the most likely etiology of the patient's symptoms? A. abdominal aortic aneurysm B. appendicitis C. cholecystitis D.diverticulitis E. small bowel obstruction

The correct answer is: A Abdominal aortic aneurysm rupture typically presents with sudden onset abdominal pain, hypotension, and a pulsatile abdominal mass. This is a life threatening emergency which requires emergent surgical intervention and is a DO NOT MISS diagnosis. Most patients do not even know they have an abdominal aortic aneurysm until it is incidentally discovered or they become symptomatic. Diagnosis is made by CT or ultrasound of the abdomen showing an aortic diameter > 3 cm. Surgical intervention is not usually necessary unless it is > 5.5 cm, there is rapid expansion, or there is rupture. Answer B: Appendicitis typically presents with right lower quadrant abdominal pain, fever, nausea, or vomiting. This patient does not have a fever or right lower quadrant abdominal pain. Answer C: Cholecystitis typically presents with fever, right upper quadrant abdominal pain, nausea, and vomiting. This patient does not have a fever or right upper quadrant abdominal pain. Answer D: Diverticulitis typically presents with left lower quadrant abdominal pain, nausea, vomiting, and fever. This patient does not have left lower quadrant abdominal pain or fever. Answer E: Small bowel obstruction usually presents with bloating and diffuse abdominal pain. The patient usually has a history of previous abdominal surgeries which cause adhesions and can result in small bowel obstruction. This patient has acute onset abdominal pain with a pulsatile mass, not characteristic of small bowel obstruction. Bottom Line: Sudden midline abdominal pain in a hypertensive smoker who presents with hypotension and tachycardia with a pulsatile mass on physical exam should make you worry for a ruptured AAA.

A 35-year-old male presents with a severe allergic reaction causing lip and tongue swelling associated with shortness of breath. Paramedics report the patient has been uncooperative and that they were unable to administer medications or start an IV. The patient's mother is present and reports a past medical history is significant for bipolar disorder and schizophrenia. Before you can perform a physical exam, the patient becomes agitated and throws a trash can at the door. It is most appropriate to A. apply physical or chemical restraints B. consult psychiatry for evaluation and treatment C. discharge the patient home against medical advice D.have police place a medical hold on the patient E. have the patient's mother sign the patient out against medical advice

The correct answer is: A As a physician, it is your responsibility to always do what is in the best interest of the patient. In this scenario, the patient is obviously psychotic, enraged, is not thinking clearly, and has been off his normal psychiatric medications. To make matters worse, he appears to have severe angioedema which can progress and eventually occlude his airway. It is considered battery if you touch a patient without his/her consent, however, when a patient does not have the capacity to decide what is in his best interest, you as the physician will need to assume responsibility for the patient. The most important thing at this time is to intervene medically and in order to do that, the patient will need to be restrained. Determining capacity: Patient needs to demonstrate understanding of the situation (diagnosis/risks/benefits). Patient needs to have the ability to make a choice. Patient needs to appreciate how the information presented applies to oneself. Patient needs to be able to compare information and infer consequences of choices. Answer B: Psychiatry will eventually need to evaluate the patient, however, he needs urgent medical treatment at this time. Answer C: The patient does not have the capacity to sign out against medical advice. The patient needs to be restrained, sedated, and treated before he develops life threatening edema of the airway. Answer D: Placing a medical hold on the patient will allow you to keep the patient in the hospital, however, it does not allow MEDICAL TREATMENT of the patient unless he is deemed to not have capacity to make the right decisions. Answer E: The patient's mother cannot sign the patient out against medical advice. The patient is an adult, but he does not have capacity to make decisions at this point, therefore, the doctor must assume care of the patient and treat him based on what he deems would be in the best interest of the patient. Bottom Line: If a patient does not have capacity to make an informed decision, the doctor caring for the patient must do what is in the best interest of the patient even if it means restraining and sedating the patient in order to care for him and save his life.

A 28-year-old female is brought into the emergency department by her family for reported seizure activity. The family reports the patient was walking across the floor at home, stopped, fell to the ground, and began shaking violently. They estimate this lasted 2-3 minutes total. History reveals a patient with known seizure disorder who has run out of her levertiracetam. Vital signs are a heart rate of 98/min, respiratory rate of 22/min, and a blood pressure 115/74 mmHg. Physical examination reveals an obese female who is alert and oriented to person, place, and time. The remainder of the examination cannot be completed because the patient begins seizing again. What is the next most important step in management at this time? A. CT of the head with contrast B. left lateral decubitus position C. rectal diazepam D.serum prolactin level E. start an IV catheter

The correct answer is: B Rolling the patient into the left lateral decubitus position is also known as the recovery position. This position is used in many instances including seizures, pregnant patients with unstable vitals and after successful CPR in the field. The purpose of doing this maneuver prior to any other interventions is to prevent the patient from aspirating. Many patients may vomit before, during or after a seizure and therefore may aspirate on the vomitus. Rolling them to the left lateral position allows the patient to expel their stomach contents without gravity and inhalation pulling the secretions into the oropharynx and the trachea. For this patient and many patients the seizure may only last a few minutes. Not rolling the patient could cause them to aspirate and potentially leaving them alone to either get supplies or medications would not be in their best interest. The patient could aspirate, roll of the bed and potentially hurt themselves. Some emergency departement's are staffed with several other people and calling out for help to get the necessary supplies would be appropriate. Answer A: CT scan of the head with contrast would be indicated in the patient with a first time seizure or an alteration in their normal seizure activity. This would be to rule out any intracranial causes for the seizure such as infection or mass. This patient has a known seizure disorder with a known cause, a lapse in her treatment, so a CT of the head would not be indicated for her. If this were a new onset seizure she would need be first a glucometer reading, then electrolyte studies, a CT of the head and toxicologic profile. Answer C: Benzodiazepines are indicated for the treatment of seizures. In this scenario however the ABC's still apply. The patient must have their airway protected first and this is done so by rolling them to prevent aspiration. When treating a patient that is having a seizure lorazepam 2mg would be the first line treatment and drug of choice. This can be administered IV or IM if an IV has not yet been established. There are preparations of rectal diazepam that are available commercially in kits for patients to use at home or while in the hospital if no other means are working. As per the history, this patient is suffering from the most common cause of seizures which is either a missed dose or running out of their antiepileptic medication. After the lorazepam administering the patients levertiracetam would be the next step in treatment. Answer D: Serum prolactin levels are useful in helping to diagnose a pseudoseizure from real seizure. This laboratory test is extremely time sensitive however and some labs may not have the ability to rapidly perform this test. If it is to be used it must be drawn within 15-30 (up to 60 in some texts) minutes after the seizure activity has occurred. This is a very acute phase reactant and will be elevated for a short period of time following a seizure. A falsely positive prolactin may be from a prolactinoma, acromegaly, and pituitary micro- and macroadenomas. Answer E: The first line treatment for a patient who is actively convulsing would be lorazepam and if this is not available then diazepam. Both of these medications may be administered intramuscularly and therefore establishing an intravenous line, while important, is not the first step. In any situation on any patient presenting to emergency department, the first steps should always be the ABC's; airway, breathing, and circulation. In this patient to protect her airway she needs to be rolled to prevent aspiration. Bottom Line: Any patient with any presentation must first have their ABC's addressed. In this patient she needed to be rolled to the left lateral decubitus position to prevent her from aspirating on her vomitus. COMBANK Insight : Be mindful of how the final question is stated for management questions (i.e., first step, next step, definitive step, etc.). Emergency medicine is very algorithm-oriented, so it is imperative that you review as many of these evidence-based algorithms as possible. The ABC's (airway, breathing, circulation) always come first. You

A 25-year-old female presents with right-sided abdominal pain, dysuria, nausea, and vomiting. History reveals last menstural period of 2 weeks ago and limited oral intake over the past 24 hours. Vitals reveal a temperature of 38.1ºC (100.5ºF). Physical examination reveals tenderness in the right and left lower quadrant with rebound, no costovertebral angle tenderness, and right-sided rectal tenderness. Hemoccult is negative. You note a tender nodule at the tip of rib 12 on the right. What is the most likely diagnosis? A. acute appendicitis B. acute cholecystitis C. acute pyelonephritis D.interstitial cystitis E. ovarian cyst

The correct answer is: A This patient's presentation of right-sided abdominal pain, fever and peritonitis on exam is most consistent with the diagnosis of acute appendicitis. It has been estimated that the lifetime risk of developing acute appendicitis in the U.S. is 12% for males and 25% for females. More than 300,000 appendectomies for acute appendicitis are performed each year in the U.S. Appendicitis is caused by luminal obstruction of the vermiform appendix, typically by a fecalith. The signs and symptoms of acute appendicitis lie along a spectrum that correlates with pathophysiology and varies depending on the location of the appendix. Early on, patients classically complain of nonspecific symptoms of general malaise, indigestion, or bowel irregularity. Anorexia is common but not universally present. Alterations in bowel function are variable and can include constipation, diarrhea, and even obstruction as a late complication. Periumbilical or central abdominal pain generally develops after nonspecific symptoms. Nausea, with or without emesis, if present, typically follows the onset of pain. Subjective or objective fever is a frequent but variable complaint. As the clinical course progresses, discomfort migrates to the right lower quadrant. Flank pain, dysuria, or hematuria can occur given the typical proximity of the appendix to the urinary tract. As the natural course of acute appendicitis progresses, examination findings also evolve. Minimal to mild periumbilical or diffuse abdominal tenderness to palpation can be present early in the course. Progressive inflammation and peritoneal irritation results in reproducible tenderness to palpation in the right lower quadrant, an exception being cases of the retrocecal appendix, which does not contact the anterior parietal peritoneum. Rebound tenderness and involuntary guarding suggest peritonitis. There is no evidence that the digital rectal exam aids significantly in the diagnosis of acute appendicitis. Rovsing sign (palpation of the left lower quadrant worsens right lower quadrant pain) results in the reproduction of pain over the McBurney point as the clinician palpates the descending colon. A positive psoas sign or obturator test suggests an inflammatory process adjacent to these respective structures. The psoas sign is elicited if abdominal pain is produced with extension of the right leg at the hip while the patient lies on the left side. The obturator test elicits pain with internal and external rotation of the thigh at the hip. The presence of abdominal rigidity, a positive psoas sign, fever, or rebound tenderness are examination findings that increase the likelihood of acute appendicitis, but the individual test characteristics are such that the presence or absence in isolation is neither sufficiently sensitive nor specific to rule out or rule in the diagnosis. The anterior chapman point for the appendix is on the tip of rib 12 on the right. Answer B: Although acute cholecystitis can present with abdominal pain that radiates to other areas of the abdomen, the classic findings of acute cholecystitis consist of right upper quadrant pain and tenderness (Murphy's sign), fever and associated nausea or vomiting. Rebound and right lower quadrant pain is more consistent with acute appendicitis. Answer C: Pyelonephritis is an infection of the upper urinary tract. Acute pyelonephritis is infection of the renal parenchymal and pelvico-calyceal system with a clinical syndrome of flank pain or costovertebral angle (CVA) tenderness, fever, and frequently other systemic symptoms such as nausea or vomiting. Signs of peritonitis such as rebound or guarding are not associated with pyelonephritis. Answer D: Interstitial cystitis also known as bladder pain syndrome (BPS) is a chronic inflammatory condition of the submucosal and muscular layers of the bladder, the cause of which is currently unknown. It is associated with suprapubic pain, dysuria, urinary urgency, urinary frequency, and nocturia in the setting of sterile urine cultures. Fever and peritonitis are not seen with interstitial cystitis. Answer E: Ovarian cysts become symptomatic by several mechanisms: rupture of contents with subsequent chemical peritonitis, torsion of enlarged ovaries, and mechanical pressure on adjacent structures due to large size. Although peritonitis can develop from a ruptured ovarian cyst, it would most often be preceded by sudden onset unilateral pain, which is not consistent with this patient's more insidious presentation. Bottom Line: Acute appendicitis is a common cause of acute atraumatic abdominal pain and remains largely a clinical diagnosis. The presence of abdominal rigidity, fever or rebound tenderness are examination findings that increase the likelihood of acute appendicitis. COMBANK Insight : Acute appendicitis is a bread-and-butter emergency medicine topic that requires prompt recognition. Be able to recognize signs of peritonitis in the questions stem that point to this likely diagnosis.

A 62-year-old female presents to the emergency department with diffuse crampy abdominal pain associated with nausea and vomiting. She states her symptoms began yesterday but she feels more bloated and is unable to eat or drink, which is why she came to the hospital. She had a similar episode 2 months ago which resolved on its own. Past medical history is significant for 3 Cesarean sections and a hysterectomy. Physical exam reveals a soft distended abdomen, diffusely tender without guarding or rebound, and high-pitched tympanic bowel sounds noted in all four quadrants. The most likely diagnosis is A. appendicitis B. bowel obstruction C. cholecystitis D.diverticulitis E. perforated peptic ulcer

The correct answer is: B Small bowel obstructions typically present with crampy abdominal pain, nausea, vomiting, and constipation. Sometimes patients will note a few episodes of diarrhea or loose stool which can be misleading because this is usually the stool distal to the obstruction. On examination the patient will usually have a distended abdomen which is diffusely tender, but sometimes can be localized depending on the severity and how localized the obstruction is. The most important clue is a history of previous surgeries and previous episodes of "partial obstructions" that resolve spontaneously. Adhesions secondary to previous abdominal surgery are the most common cause of small bowel obstructions. Treatment is usually supportive with IV fluids and a nasogastric tube, but if the patient does not improve, surgical intervention may be necessary. Patients need to remain NPO regardless to allow for bowel rest and decompression. Answer A: This patient does not have a fever or right lower quadrant pain and given the information provided in the question stem, appendicitis is not the most likely cause of the patient's symptoms. Answer C: This patient does not have a fever or right upper quadrant pain and given the information provided in the question stem, cholecystitis is not the most likely cause of the patient's symptoms. Answer D: This patient does not have a fever or left lower quadrant pain and given the information provided in the question stem, diverticulitis is not the most likely cause of the patient's symptoms. Answer E: A perforated peptic ulcer usually presents with sharp epigastric pain which began suddenly and a history of peptic ulcer disease. An upright chest x-ray will show free air under the diaphragm which is a sign of intraabdominal free air. This is a surgical emergency and needs immediate surgical evaluation. Bottom Line: In a patient with previous abdominal surgeries who presents for abdominal pain, nausea, and vomiting, with "tympanitic" bowel sounds on exam, small bowel obstruction should be at the top of the differential.

An 86-year-old female with a history of diverticulosis, presents to the emergency department for abdominal pain, nausea, vomiting, and a fever. She states her symptoms began 2 days ago and have been progressively worsening. Her pain is localized to the left lower quadrant without radiating symptoms. Her last bowel movement was this morning which was normal. Physical examination reveals a soft abdomen that is moderately tender in the left lower quadrant without guarding or rebound noted. What is the most likely cause of the patient's symptoms? A. appendicitis B. diverticulitis C. fibroids D.ovarian cancer E. small bowel obstruction

The correct answer is: B This patient has left lower quadrant pain with fever and a history of diverticulosis, which is consistent with diverticulitis. Diverticulitis is an inflammation of the "outpouchings of the colon". The most common location of pain is in the left lower quadrant. Depending on how bad the diverticulitis is, the patient may have rebound tenderness and guarding. Complications include bowel obstruction, fistula formation, perforation, or development of a localized abscess in the area. Symptoms may include fever, chills, nausea, vomiting, or diarrhea. Treatment is usually supportive with IV fluids and antibiotics, but sometimes surgical intervention is necessary. Answer A: Appendicitis should always be on the differential if a patient has lower abdominal pain and a fever, however, the patient's pain is localized to the left lower quadrant, which is atypical for appendicitis. Sometimes patient's will exhibit Rovsing sign, where they get right lower quadrant pain with palpation of the left lower quadrant. This patient's history and physical exam are not entirely consistent with appendicitis, therefore it is not the MOST likely cause of the patient's symptoms. Answer C: Fibroids usually do not cause acute onset abdominal pain and would not be associated with a fever. Also, this patient is likely post menopausal, which usually causes the fibroids to shrink in size, and most women are no longer symptomatic after menopause. Answer D: Ovarian cancer usually does not present with acute onset abdominal pain and fever. Answer E: Small bowel obstruction usually does not cause a fever unless an underlying infection caused the obstruction. The patient had a normal bowel movement earlier in the day, which makes a small bowel obstruction less likely. Most of the time patient's will present with gradually increased abdominal bloating, not tolerating anything by mouth, constipation, or diarrhea. Diarrhea is counterintuitive in an "obstruction", however, it is usually the stool distal to the obstruction. The most common cause of SBO is adhesions from previous abdominal surgeries. Bottom Line: This is the most likely cause of the patient's symptoms considering her history of diverticulosis and clinical presentation of fever, nausea, vomiting, and left lower quadrant abdominal pain.

A 50-year-old male presents with palpitations, shortness of breath, and chest pain. An electrocardiogram reveals absent P waves and a widened QRS complex occurring at a rate of 120/min. The most appropriate management at this time is A. adenosine B. atropine C. cardioversion D. defibrillation E.epinephrine

The correct answer is: C Cardioversion is a low energy electrical current delivered to the patient in synchronization with their heart rate in order to correct potentially fatal arrhythmias. The is classically used in the setting of ventricular tachycardia while the patient still has a pulse, in order to correct the heart rhythm before it progresses to ventricular fibrillation and cardiac arrest. Cardioversion is first line therapy for patients with ventricular tachycardia with a pulse or symptomatic supraventricular tachycardia. Answer A: Adenosine is a very short acting medication that works by blocking the action of the AV node. When the affects wear off, normal excitability has been reestablished within the AV node and normal rhythmic contractions begin again. Adenosine would be the first step after vagal maneuvers are tried. This treatment would be ineffective if the source of hyper-excitability/re-entry is something other than the AV node. Obviously adenosine should not be used in any patient with pre-existing conduction problems, such as prolonged QT syndrome or any heart block, as it's action of delaying conduction will acutely worsen these conditions. Answer B: Atropine is an acetylcholine antagonist which functions by both up-regulating pacemaker activity at the sinoatrial (SA) node and hastening conduction through the atrioventricular (AV) node. This medication is first line therapy for symptomatic bradycardia. Other options considered if atropine fails include: dopamine, epinephrine, and transcutaneous pacing. In the setting or SVT or ventricular tachycardia, atropine is contraindicated as it's mechanism of action may lead to worsening increase in heart rate which could cause or worsen an ischemic episode. Answer D: Defibrillation is a high energy electrical shock used in the setting of ventricular tachycardia or ventricular fibrillation when the patient has no pulse and the CPRS cycle has begun. Defibrillation is used at two-minute intervals with ongoing compressions occurring in between shocks. This electrical current is unsynchronized as the heart rate in ventricular fibrillation is erratic and cannot be matched with the delivery of the shock. This is the first line therapy for pulseless ventricular tachycardia/fibrillation. Answer E: Epinephrine is a neurotransmitter and is a adrenergic receptor agonist. This treatment causes an increased heart rate and is used in most acls algorithms including ventricular fibrillation, bradycardia, and asystole/PEA. In the setting of ventricular tachycardia, epinephrine may be used once patient is pulseless and requires defibrillation with interval compressions. Bottom Line: Synchronized cardioversion is first line therapy in the setting of ventricular tachycardia where the patient still has a pulse.

A 65-year-old female is brought to the emergency department via ambulance after several episodes of hematemesis. History reveals she is an alcoholic and upon arrival she is speaking incoherently but responds appropriately to verbal commands. Vital signs are blood pressure of 102/46 mmHg, heart rate of 115/min, respirations of 24/min and a pulse ox of 95% on room air. Physical exam reveals jaundice to the level of the nipples, mild abdominal distention and worsening epigastric pain with palpation. The patient is immediately started on an IV bolus of normal saline and intravenous pantoprazole. Laboratory studies reveal the following: hemoglobin 9.1 g/dL albumin 2.7 g/dL ammonia 42 mcg/dL (reference: 20-70 mcg/dL) INR 3.4 Which of the following pharmacological agents is most appropriate at this time? A. dicyclomine B. furosemide C. lactulose D. octreotide E. vitamin K

The correct answer is: D Gastroesophageal variceal hemorrhage is a medical emergency that occurs in up to 10% of patients with cirrhosis each year. All patients with cirrhosis should undergo endoscopic surveillance for early detection of varices. If present, patients should be started on a beta-blocker, commonly nadolol or propranolol, to decrease portal venous pressure to prevent bleeding. Patients who present with active bleeding should receive octreotide, is a somatostatin analogue that acts within the GI system to lower portal blood pressure and prevent rebleeding during the patient's initial hospitalization. It acts by inducing vasoconstriction of the splanchnic vessels. This action would decrease the amount of blood flow to the bleeding varicocele and therefore decrease the patients overall bleeding. In addition to octreotide, a proton pump inhibitor should also be part of the initial treatment plan. Patients with significant variceal hemorrhage should undergo urgent endoscopy within 12 hours of presentation. Band ligation is preferred for control of bleeding and is considered the definitive treatment of choice. Answer A: Dicyclomine is an anticholinergic drug that antagonizes the muscarinic receptors of the smooth muscles of the intestinal tract. It is traditionally used as an antispasmodic for patients suffering from irritable bowel syndrome. Dicyclomine would have no role for treatment in this patient. Using too much of this drug would result in an anticholinergic syndrome. This would be exhibited by myosis, skin flushing, tachycardia, hyperthermia, and diaphoresis. Answer B: Furosemide is a loop diuretic that acts on the luminal Na-K-2Cl symporter in the thick ascending limb of the loop of Henle leading to free water clearance. It is used in several different clinical scenarios to increase urinary fluid excretion in volume overloaded states. It is indicated in cirrhosis to control mild to moderate ascites. It is not indicated in the management of esophageal varices. Answer C: Lactulose is used for constipation and hepatic encephalopathy. This patient is suffering from alcoholic cirrhosis which could produce hepatic encephalopathy, however with a normal serum ammonia level this is not the cause for the patients altered mental status. It is much more likely given her presentation that she is intoxicated from alcohol usage. Lactulose works within the bowel to lower serum ammonia levels by trapping ammonia ions within the intestinal lumen to be excreted through defecation. Answer E: Patients with cirrhosis commonly develop coagulopathies secondary to poor synthetic function of the liver, leading to decreased production of coagulation factors. This is why she has an increased INR. In patients with acute bleeding who have evidence of coagulopathy should receive fresh frozen plasma. Vitamin K will not correct the coagulopathy because the issue is with clotting factor production, not lack of vitamin K. Bottom Line: Hematemesis should be urgently addressed. Knowing the possible etiologies of the bleed and how to properly and quickly address the etiologies of the bleed in the emergency department can mean potentially saving the patient from significant morbidities.

A 39-year-old female presents to the emergency department with a chief complaint of abdominal pain, nausea and vomiting. Vital signs reveal a temperature of 37.1ºC (98.8ºF), blood pressure of 101/77 mmHg, pulse of 105/min, respiratory rate of 18/min, and oxygenation saturation of 98% on 2 liters nasal cannula. Physical examination reveals that the patient is in moderate discomfort. On physical examination, her heart rate and rhythm are regular and without murmurs, rubs or gallops, and the lungs are clear to auscultation bilaterally. The abdomen is soft, non-distended, and moderately tender to palpation in the epigastric region with an associated Cullen's sign. The most likely diagnosis is A. acute cholecystitis B. appendicitis C. gastroesophageal reflux disease D. pancreatitis E.ruptured ectopic pregnancy

The correct answer is: D Pancreatitis occurs from the leakage of pancreatic enzymes into the pancreatic and peripancreatic tissue, usually due to alcoholism or gallstone disease (gallstone can lodge in the common bile duct and prevent efflux of pancreatic enzymes causing backflow of pancreatic enzymes). Other risk factors include: hypercalcemia, hypertriglyceridemia, certain medications (including thiazide diuretics, steroids, etc.), post-ERCP, viral infections (mumps and coxsackie B virus), scorpion bites and many other causes. Signs/symptoms include fever, epigastric pain with radiation to the back, nausea and vomiting, shock, periumbilical discoloration (Cullen's sign) and flank discoloration (Grey Turner sign) which is due to pancreatic retroperitoneal hemorrhage. Diagnosis is made by elevated amylase/lipase and a CT scan which may show "sentinel loop" or "colon cutoff sign" (sentinel loop is an area of localized ileus of the small intestine or colon secondary to nearby inflammation while colon cutoff sign is dilation of the proximal colon with narrowing at the splenic flexure secondary to inflammation from the pancreas extending into the phrenicocolic ligament via the transverse mesocolon). Treatment includes IV fluids and electrolyte replacement, bowel rest, NG suction, pain control, and supportive care. Complications include hypocalcemia, pancreatic pseudocyst formation, renal failure, acute respiratory distress syndrome, pleural effusion, sepsis and death. Answer A: Acute cholecystitis is typically caused by blockage of the cystic duct most commonly by a gallstone. This leads to blockage of the gallbladder which leads to distention which eventually leads to inflammation and infection. Acalculous cholecystitis occurs in the setting of chronically debilitated patients especially in those receiving total parenteral nutrition and also in trauma and burn patients. Symptoms include nausea, vomiting, right upper quadrant abdominal pain, fever/chills. On physical exam there may be right upper quadrant discomfort with the rebound or guarding, epigastric discomfort, fever, scleral icterus and there may be a positive Murphy sign (arrest of inspiration during deep palpation of the right upper quadrant secondary to irritation of the parietal peritoneum touching the inflamed gallbladder). Workup includes a CBC, amylase, lipase, and CMP. The initial imaging choice is an ultrasound of the abdomen which may demonstrate stones in the gallbladder, pericholecystic fluid, gallbladder wall thickening greater than 3 mm, gas in the gallbladder or a sonographic Murphy sign (pain when palpation of the gallbladder under direct visualization of the gallbladder using ultrasound). Consideration of a HIDA scan may be considered if the ultrasound the gallbladder is equivocal. Non-visualization of the gallbladder on HIDA scan suggests acute cholecystitis. Treatment includes admission, surgical consultation for cholecystectomy, IV pain medications, NPO status, antibiotics and repletion of electrolytes and fluids. Consideration of a preoperative ERCP or and intraoperative cholangiogram may be considered to rule out common bile duct stones. Also note that patients with significant medical problems and who are hemodynamically stable can be managed medically with a 4 to 6 week delay in surgical treatment. Complications of acute cholecystitis include gangrenous cholecystitis, gallbladder perforation, gallstone ileus, empyema, sepsis, and abscess formation. Answer B: The initialization of appendicitis begins with obstruction of the lumen of the appendix. This can be secondary to a fecalith, adhesions, enlarged lymph tissue, or food. After obstruction occurs secretion of mucus continues in the intestinal lumen which increases intraluminal pressure and leads to cessation of venous and lymphatic drainage. After this occurs epithelial breakdown ensues with bacterial invasion into the bowel wall leading to inflammation. Eventually intraluminal pressure increases the point that arterial flow stops and this leads to perforation of the appendix with leakage of inflammatory and bacterial contents into the peritoneum. The classic symptom of appendicitis is. Umbilical pain with eventual migration to the right lower quadrant followed by anorexia, nausea and vomiting. A classic point of pain in the right lower quadrant is described as McBurney's point which is one-third of the distance from the ASIS to the umbilicus. There are other classic signs on examination including the Rovsing sign, obturator sign, and psoas sign. The Rovsing sign occurs when right lower quadrant pain is elicited upon palpation of the left lower quadrant. Obturator sign occurs when pain is elicited when the physician passively flexes the right hip and knee and internally rotates the hip. This occurs secondary to irritation of the appendix from the obturator internus muscle. Lastly, the psoas sign is elicited by placing the patient in the left lateral decubitus position in extending the right leg and hip-joint. This causes pain by stretching the psoas muscle which may be irritated by the adjacent appendix. On exam, there may also be involuntary guarding, rebound tenderness and tenderness and rectal exam. The differential diagnosis for right lower quadrant pain is broad and other considerations include mesenteric lymphadenitis, public inflammatory disease, acute gastroenteritis, cholecystitis, perforated ulcer, diverticulitis, pancreatitis, small bowel obstruction, renal calculus, pyelonephritis, and over and pathologies such as torsion, abscess, or ectopic pregnancy. Workup for suspected appendicitis includes CBC, urinalysis, and CT vs. ultrasound. CT scan is considered the best choice for initial evaluation secondary to its excellent sensitivity and specificity. However, ultrasound is the test of choice in children and pregnant patients. Treatment of appendicitis includes appendectomy and a surgical consult. However, clinical observation with serial abdominal exams may be warranted if the diagnosis is unclear. Patients should receive IV fluids, IV antibiotics and remain NPO. Complications of appendicitis include peritonitis, periappendiceal abscess, and perforation. Answer C: Gastroesophageal reflux disease affects approximately 20% of adults. Typical symptoms include retrosternal burning sensation which starts in the epigastrium and radiates upward into the chest. It typically occurs within one hour of eating during exercise or when lying recumbent. It may partially be relieved by antacids. Other symptoms include throat fullness, excessive salivation, bitter taste, painful swallowing, difficulty swallowing, halitosis and ear pain. Other symptoms include atypical chest pain, asthma, hoarse voice, and nocturnal cough. Exam is often normal but the patient may present with poor dentition and/or wheezing. Considerations on the differential diagnosis include peptic ulcer disease, biliary colic, angina, esophageal dysmotility, pill esophagitis, and infectious esophagitis. Diagnosis is clinical and impaired therapy with a trial of proton pump inhibitors can be initiated for 4 to 6 weeks. Response to PPI's diagnostic. If the patient is unresponsive to therapy or has alarm symptoms (weight loss, anemia, long-standing symptoms, blood in the stool, age greater than 50, odynophagia) then a GI consultation is required. GI workup may include upper endoscopy, barium esophagography, or ambulatory esophageal pH monitoring. Treatment is first with behavior modification including elevating the head of the bed during sleep, discontinuing alcohol and tobacco use, advising patients to eat smaller more frequent meals, reduce fat intake, weight loss, avoiding recumbent positioning after eating, and avoiding certain foods including chocolates, coffee, tea, carbonated beverages, citrus, and tomato juice. Other therapies include H-2 blockers, antacids, proton pump inhibitors, bismuth, sucralfate, and H. Pylori eradication. Answer E: Ectopic pregnancy occurs when the fertilized egg implants outside of the uterine cavity. Most implant in the fallopian tube but other sites include the abdominal cavity, ovary, and cervix. Risk factors include pelvic inflammatory disease, tubal ligation, previous pelvic surgery, use of an IUD, previous ectopic pregnancy, elective abortion, and fertility treatments. Symptoms include vaginal bleeding, abdominal pain, and lightheadedness or syncope. On exam the patient may be hypotensive/in shock (secondary to hemorrhage) or vital signs may be normal. The pelvic/abdominal exam may be normal or the may be localized tenderness including cervical motion tenderness. Also, adnexal tenderness may or may not be present on exam. The differential diagnosis is very broad and includes: appendicitis/cholecystitis,ovarian torsion, ovarian cyst, pelvic inflammatory disease, endometriosis, renal colic, UTI, inflammatory bowel disease, mittelschmerz, intrauterine pregnancy, threatened abortion, inevitable abortion, Molar pregnancy, heterotopic pregnancy, corpus luteum cyst, and others. Note that the classic triad for ectopic pregnancy is a positive pregnancy test, vaginal bleeding, and abdominal pain. Diagnosis is by a positive pregnancy test, and ultrasound. On ultrasound the finding of a yolk sac, double decidual sign, or fuel cardiac activity inside the uterus is diagnostic of an intrauterine pregnancy. Note that a gestation sac is not diagnostic for an intrauterine pregnancy. Beta-hCG facilitates ultrasound interpretation. There is a "discriminatory zone" which is between 1000-1500 IU/mL for transvaginal ultrasound and 4000-6500 IU/mL four transabdominal ultrasound. In other words, above the discriminatory zone it should be possible to identify an intrauterine pregnancy and failure to do so mandates an OB/GYN consult for presumed ectopic pregnancy. A beta hCG below the discriminatory zone and an ultrasound that fails to demonstrate an intrauterine pregnancy is consistent either with an early intrauterine pregnancy or in ectopic pregnancy. Management is aimed at having a repeat quantitative beta hCG drawn by the patient's OB/GYN in 48 hours if the patient is stable. Note that the quantitative beta hCG should double within 48 hours in viable pregnancies. A lack of doubling requires further evaluation. Management of unstable patient includes administration of crystalloid, and/or packed red blood cells to maintain blood pressure and an immediate OB/GYN consult should occur to consider an emergent laparotomy. Stable patients with an ectopic pregnancy may consist of a laparotomy or medical management with methotrexate per the recommendation of the OB/GYN consult. Bottom Line: Consider pancreatitis in patients with nausea, vomiting, epigastric pain, Cullen's sign, and Grey Turner sign

A 7-year-old male is brought to the emergency department by his parents. History reveals the child has asthma and his nebulized albuterol is not helping. Vital signs reveal a heart rate of 145/min, respirations of 40/min, blood pressure of 110/65 mmHg, oxygen saturation of 82% on room air. Physical exam reveals a child in severe respiratory distress who is exhibiting extreme work of breathing. Auscultation of the chest is negative for wheezing and breath sounds are extremely diminished. You note paravertebral hypertonicity from T2-6 and T3FRRSR. The decision is made to perform rapid sequence intubation with succinylcholine. What is the most common complication associated with this medication? A. hypercalcemia B. hypokalemia C. hypotension D.increased secretions E. rhabdomyolysis

The correct answer is: E Rhabdomyolysis with associated hyperkalemia is a well known and well documented complication of giving pediatric patients succinylcholine. This happens when apparently healthy children are intubated using succinylcholine. Workup reveals that the child has an underlying skeletal muscle myopathy and in combination with the succinylcholine causes breakdown of the skeletal muscle. The most common myopathy is Duchenne's muscular dystrophy. Rhabdomyolysis occurs immediately after the initial dose and requires prompt correction of the hyperkalemia. Rhabdomyolysis is checked by ordering a serum creatine kinase (CK, a breakdown product of skeletal muscle). In high enough concentrations this can be toxic to the kidneys and cause acute renal failure. Treatment for this is copious IV fluid hydration. If damage is done to the kidney it is typically seen 3-5 days after the initial rise in CK. The contraindications to succinylcholine are as follows; burns, crush injuries, renal failure, immobilization for >48 hours, narrow angle glaucoma and malignant hyperthermia. In the cases of severe burns, crush injuries and renal failure, hyperkalemia is a major concern as all three of these are extremely likely to result in hyperkalemia. This would only worsen with administration of the succinylcholine. In patients who have been immobilized for greater than 48 hours, the same principle as the children would apply. A patient being immobile for that period of time would certainly have muscle breakdown and hence elevated CK. The succinylcholine would only worsen and increase the rhabdomyolysis. Narrow angle glaucoma would be exacerbated by the anticholinergic effect of the succinylcholine. Patients already exhibiting malignant hyperthermia absolutely should not be given succinylcholine as it will exacerbate their symptoms. In this patient you see TART changes and thoracic somatic dysfunction which are likely representative of viscerosomatic dysfunction from the lungs. This is very common with asthma and treating rib and thoracic dysfunction with OMT is commonly employed in a clinic setting to help manage asthma in children. In the acute setting with respiratory distress, however, OMT would not be appropriate until the patient is medically stable. Answer A: Hypercalcemia is not typically seen with succinylcholine use. Hypercalcemia most often results from malignancy or primary hyperparathyroidism. Less often it can be seen as a result of granulomatous disease, thiazide diuretic usage or increased absorption/intake. Hypercalcemia can cause depression, kidney stones, constipation or brittle bones (after being leached out). The first line of treatment would be IV fluids. Answer B: Hypokalemia is a side effect of insulin, albuterol and sodium bicarbonate and as such are used to treat hyperkalemia. Succinylcholine can cause hyperkalemia and is especially at risk for causing hyperkalemia in patients suffering from burns, crush syndrome, severe infections, preexisting myopathies and pre-existing hyperkalemia. In the patient with hyperkalemia you may see peaked T-waves on ECG. This however may not always be true, a patient may have elevated serum potassium without ECG changes. The opposite is always true. Peaked T-waves always correlate to elevated serum potassium levels. Hyperkalemia can lead to dysrhythmias and must be addressed rapidly. Calcium gluconate (through a peripheral line) or calcium chloride (through a central line) must be given and are cardioprotective against developing a dysrhythmia. Calcium is then followed by the other medications: albuterol, sodium bicarbonate or insulin/dextrose. Answer C: Hypotension in this case would likely be caused by the sedating medication. Propofol decreases sympathetic activity causing myocardial depression and peripheral vascular dilation. In addition, if hemodynamically stable propofol can cause airway relaxation and may be beneficial in the asthmatic patient. Propofol works directly on the GABA receptors in the brain and produces sedation and amnesia. It is highly lipophilic and easily crosses the blood-brain barrier. Answer D: Increased secretions are a known side effect of using ketamine as a sedative agent. Ketamine has been well researched and is indicated in the use of sedation for pediatric patients as well as adult patients. It has the side effect of causing increased secretions so physicians should be prepared to deal with them. Also ketamine causes bronchodilation and hypertension through sympathetic stimulation. It is however the most hemodynamically stable of all the sedating agents making it a popular choice for hypotensive patients. Ketamine will also preserve respiratory drive making it the ideal agent if an "awake" intubation is to be performed (generally done so when concern about a difficult airway). Bottom Line: Rhabdomyolysis is a known complication of using succinylcholine in children with undiagnosed skeletal muscle myopathies.

A 14-year-old female is brought into the emergency department by her parents after being struck in the head with a baseball bat at her softball game. She is currently conscious with a Glasgow coma scale of 12/15. Vital signs reveal a heart rate of 95/min, respirations of 22/min, blood pressure of 98/50 mmHg, and an oxygen saturation of 99% on room air. Direct observation demonstrates a 1.5 cm laceration on the left frontal bone with surrounding erythema extending 2 cm above the underlying skin at its peak. After assessing airway, breathing and circulation, the most appropriate step in management is A. CT scan of the head and neck B.IV insertion with maintenance fluids C. neuromuscular testing D. observation E. placement of cervical collar

The correct answer is: E Stabilization of the cervical spine with a c-collar in any head or neck injury should always be a first priority. She should immediately be placed in a c-collar to prevent her cervical spine from doing any more possible damage until a CT scan can be performed to rule out any fractures of the cervical spine. Again this should be first priority to prevent any further damage. There are useful screening tools that may be used to assess if the C-spine may be cleared. These are the NEXUS criteria and the Canadian C-Spine Rules. These include criteria such as: neurological deficits, spinal tenderness, altered level of consciousness, intoxication, and any other injuries. Answer A: The patient should be sent for a CT scan to rule out any intracranial hemorrhages or possible cervical vertebral fractures. This would be the best most definitive tool in ruling out a C-spine fracture or any intracranial abnormalities such as an epidural or subdural bleed. Due to the time frame of obtaining a CT scan other interventions such as c-collar placement, IV placement and you initial assessment must come first. Answer B: IV placement should only take a few moments and in the unstable patient would be a must in resuscitation. This can be done after the c-collar placement by the emergency department nurses while you are doing you initial history and physical. Answer C: Next in the initial assessment would be the physical exam, which should include thorough neuromuscular testing to assess for any neurological deficits. If the patient is unconscious you must rely on reflexes for you assessment. If they are awake a full neurological assessment must be done, including sensation, strength, touch, and fine motor movements. Answer D: Observation would absolutely be contraindicated in this patient. Due to the mechanism of her injury she could have a potential intracranial hemorrhage, which if left without intervention may prove fatal. If a patient were to present with concussion type symptoms and was alert, able to answer questions, only complaining of a mild headache, should observation be considered. If nausea and vomiting are present, these are suggestive of increased pressure and should prompt further work-up including a CT scan of the head. Pediatric head injury cases regarding the need to observe versus perform a CT scan of the head is a very controversial issue and would be practice dependent. Bottom Line: For any trauma involving the head and neck a c-collar should be placed for cervical spine stabilization until it can be cleared. COMBANK Insight : Be cognizant of the final question wording for management items. Most of these questions will specify the initial step vs. next step vs. final step. Other terminology used is the confirmatory, definitive, or most important step. Typically these items refer to a gold-standard management algorithm or rule-out criteria.

An 10-year-old male presents to the emergency department with unilateral testicular pain for the past thirty minutes. Examination reveals that the pain is more intense near the head of the epididymis and testes. You note paravertebral spasms from T11-L2. Palpation reveals bilateral descended testes with an isolated tender nodule at the superior aspect of the left testes. A small blue reflection is seen when the nodule is brought close to the scrotal skin and a light shines on it. The most likely diagnosis is A. carcinoma of testes B. epididymitis C. orchitis D.testicular torsion E. torsion of the appendix testis

The correct answer is: E The appendix testis is located at the superior aspect of the testes in a groove between the testis and epididymis. This commonly becomes torsed in children. The classic "blue dot sign", a small palpable tender nodule on the superior testis that reflects blue with light through the scrotum as described in this case, is almost pathognomonic for this condition. Treatment is directly at pain management as the torsion does not affect testicular function. You may note viscerosomatic reflexes from T11-L2 as a result of the testicular pathology. Answer A: While testicular cancer can cause pain in the testis, it is rare before the age of 20. Germ cell tumors are the most common type of testicular carcinoma with seminoma being the most common overall. Neoplasm of the testis is usually noticed as a lump on the testis which can be either painless or painful, additional signs include feelings of scrotal fullness and sharp pains in the groin. Children can have an increased risk of developing testicular cancer in the setting of undescended testis. Answer B: Epididymitis is an inflammation or infection of the epididymis which lies adjacent and connects to the testis. Epididymitis typically presents with unilateral testicular pain that is additionally tender to palpation and fever. An ultrasound is typically ordered to rule out testicular torsion as the clinical presentation can be similar. Treatment is antibiotics. Answer C: Orchitis is an inflammation/infection of the testis. Like epididymitis, the clinical presentation can be very similar to testicular torsion with symptoms such as unilateral scrotal pain that is very tender to palpation, scrotal swelling, and potentially hematuria. Orchitis can be related to STDs such as gonorrhea/chlamydia or childhood infectious diseases like mumps. Treatment is with oral antibiotics and pain control. Answer D: Testicular torsion is the main concern for emergency rooms when a patient presents with acute onset scrotal pain. Studies suggest that the timeline is up to 6 hours to reestablish adequate blood supply to the testis before tissue death occurs. Ultrasound is the imaging of choice to rule out torsion, but the diagnosis is often made clinically. Immediate surgical de-torsion is the typical treatment. Bottom Line: The classic "blue dot sign" with testicular pain indicates torsion of the appendix testis.

A 62-year-old male presents to the emergency department after vomiting a large amount of bright red blood twice just prior to arrival. History is significant for end-stage liver disease secondary to chronic alcohol abuse with a MELD score of 32. He reports his only complaint at this time is nausea and a cough because he is gagging on the vomitus. He suddenly leans forward and vomits 500 cc of bright red blood. What immediate action needs to be taken at this time? A. emergent esophagogastroduodenoscopy B. IV fluids C. IV octreotide D. NG tube placement E. rapid sequence intubation

The correct answer is: E The initial management of patients with acute variceal hemorrhage should be airway management, hemodynamic stabilization, and treating coagulopathies. These patients need emergent upper endoscopy and should be prepped for the procedure as much as possible in the ED. Resuscitation should include IV fluids or blood products for massive hemorrhage as well as continual Octreotide. Coagulopathy should be reversed if patient is on Coumadin with vitamin K and FFP. Initial antibiotics given prophylactically in the ED, should be either a fluoroquinolone or a third generation cephalosporin. If variceal ligation results in rebleeding, the patient will need to go for a TIPS procedure which will help decompress the portal circulation. A Model for end-stage liver disease (MELD) score is an estimated rate of 3-month survival in those with cirrhosis. It takes the following variables into account: Creatinine, bilirubin, INR, dialysis. Answer A: This is appropriate in severe GI bleeding, however, the patient needs to be intubated first to protect his airway. Answer B: IV fluids and blood products should be administered to maintain hemodynamic stability, however, the main priority at this time is establishing an airway since the patient is choking on his own blood. We are also not provided any information about the patient's hemodynamic status (e.g., blood pressure, heart rate, hydration status). Answer C: Octreotide is appropriate to administer in variceal bleeding, however, this patient is not protecting his airway and needs to be intubated to prevent aspiration. Answer D: An NG tube will not protect the patient's airway and it may worsen a variceal bleed! Always remember your A,B,C's when stabilizing a patient. Bottom Line: This is the most important step at this time since the patient is choking on his own blood. COMBANK Insight : Remember that no matter what is going on in the scenario, you need to break down the situation to its basics. AIRWAY should always be first, because without an airway, breathing and circulation can't continue. This patient will need an airway established, IV fluids and likely blood products if he continues to bleed profusely.

Central Renal Vein Occlusion

A central retinal vein occlusion is caused by atherosclerosis boating up and causing thrombosis and occlusion of the central retinal vein. Risk factors include hypertension, hyperlipidemia, and diabetes. Symptoms include gradual painless monocular vision loss. Note that central retinal vein occlusion occurs gradually whereas central retinal artery occlusion occurs with a sudden onset. Upon examination of the retina there is evidence of optic disc edema, numerous retinal hemorrhages, cotton wool spots in macular edema, and dilated congested veins (blood and thunder appearance of the retina). Treatment is urgent ophthalmology consult but there is no acute treatment in the emergency department. Consider initiating aspirin therapy after discussion with the ophthalmology consult.

Acute Angle Gluacoma

A: In acute angle closure glaucoma the patient displays sudden onset painful vision loss. Pathophysiology: Occlusion of the trabecular meshwork occurs when the pupil dilates preventing aqueous humor from exiting. This increases intraocular pressure and can be triggered by dim light or mydriatics (agents that dilate the eye). Risk factors include Eskimo/Asian ancestry, women, and large cataracts. Symptoms include blurry vision with halos around lights, severe unilateral eye pain, headache, and nausea and vomiting. On exam the patient has a fixed and dilated pupil which is nonreactive to light or sluggish to light, and injected conjunctiva, a steamy or hazy cornea, and elevated intraocular pressure (usually 40 to 70 mmHg). Treatment includes blocking the production of aqueous humor or increasing the outflow. Beta blockers and alpha antagonists are used to block production of aqueous humor. Note that beta blockers are contraindicated in patients with COPD/asthma or congestive heart failure. Alpha antagonists also increase aqueous humor outflow. Other agents include: carbonic anhydrase inhibitors (decrease aqueous humor production-avoid in sulfa-allergic patients), miotics/cholinergics (constricts the pupil and allows outflow), and prostaglandin agonists (increases outflow of aqueous humor). Emergent ophthalmology consult is also warranted.

A 42-year-old male presents to the emergency department after he was discovered unconscious lying outside on a park bench. Vital signs on site reveal a heart rate of 52/min, a blood pressure of 90/48 mmHg, and respiratory rate of 14/min. Physical examination reveals a lethargic and disheveled male with pupils that are 4 mm, equal, round, and reactive bilaterally. An electrocardiogram is obtained and displays sinus bradycardia with first-degree atrioventricular block and diffuse J waves. Laboratory values indicate a normal anion gap. Question 1 of 2 in this set The most likely cause is A. cocaine overdose B. hypothermia C.methanol intoxication D. opioid overdose E. Wellens syndrome

The correct answer is: B This patient is exhibiting signs of hypothermia. Hypothermia is defined by a core temperature of 35.0 °C or (95.0 °F). The causes of hypothermia are vast and include conditions such as environmental exposures, metabolic disorders including hypoglycemia and hypothyroidism, sepsis, head trauma and drug use. The electrocardiographic finding of "J" waves also known as Osborne waves alludes to a diagnosis of hypothermia. J waves are characteristically seen with but are not pathognomonic with hypothermia and the height of the J wave is roughly proportional to the degree of hypothermia. The initial response to hypothermia is the body's physiologic attempts to retain heat thus leading to an initial excitatory stage, which includes increased cardiac output, pulse and blood pressure. This is followed by an adynamic stage in which there is a progressive slowing of metabolism, leading to bradycardia, hypotension and decreasing cardiac output. Characteristic electrocardiogram changes that occur include Osborn or J waves, T wave inversions and prolongation of PR, QRS and QT intervals, followed by dysrhythmias. Answer A: Cocaine acts as a central nervous system stimulant and blocks the pre-synaptic re-uptake of neurotransmitters such as norepinephrine, dopamine and serotonin. The excess neurotransmitters accumulation leads to increased sympathetic activation. Some of the clinical effects include hyperthermia, diaphoresis, euphoria, hyperactivity, agitation, convulsions, dilated pupils, rhinorrhea, tachycardia, hypertension, coronary artery spasm, pulmonary edema, and ischemic bowel. The physical exam findings in the above scenario are inconsistent with findings of cocaine intoxication. Answer C: Methanol is a toxic alcohol. An anion gap metabolic acidosis with an underlying elevated osmolar gap will point to an ingestion and toxicity with this substance. Methanol is substance used in gas line antifreeze, windshield washer fluid as well as "sterno" fuel. Although an initial signs of toxicity may be CNS sedation, a patient may be initially asymptomatic but may later develop abdominal pain, nausea, and vomiting and blurred vision. On physical exam, one may find an increased respiratory rate, as the body attempts to get exhale CO2 and thus ameliorate the underlying acidemia. Laboratory findings will reveal a decreased pH and bicarbonate and an elevated anion gap. This patient has a normal anion gap. Refer to causes of anion gap metabolic acidosis link. Answer D: Typically with opioid intoxication the patient's physical exam will reveal pinpoint pupils and decreased respiratory rate. The main concern with opioid overdose is a decreased respiratory drive, which can lead to respiratory failure. Osborn waves or J waves are not typical findings with opioid use. If the question stem mentioned sequelae of opioid overdose such as hypoventilation with a respiratory rate less than 12, pinpoint pupils or track marks on the extremities, this would be a plausible answer choice. Answer E: Wellens syndrome is a critical stenosis of the left anterior descending coronary artery (LAD) and refers to specific electrocardiogram abnormalities in the precordial leads (V1-V6). It is imperative to recognize this syndrome because it represents a pre-infraction stage of coronary artery disease. The electrocardiographic findings include either deep symmetrical T wave inversions or biphasic T wave changes in anterior precordial leads. Patients will typically present with symptoms consistent with coronary artery disease (chest pain, shortness of breath, diaphoresis, nausea, vomiting) although the classic electrocardiographic findings are typical during pain-free periods. The findings of "J" or "Osborne waves are not associated with Wellens syndrome. Bottom Line: Hypothermia is a decreased core body temperature and can manifest as decreasing metabolic activity including bradycardia, hypotension, and dysrhythmias. "J" or Osborne waves are associated with hypothermia. The treatment for hypothermia includes specific warming techniques and supportive care. One of the first measures should be removal of the patient from the cold environment, removal of cold wet clothing and drying the patient to prevent further heat dissipation. There are numerous methods of rewarming and there are no established superior rewarming modalities. Some re-warming methods include warm water immersion, application of heating blankets, radiant heat, breathing in warmed oxygen, heated intravenous fluids as well as more invasive measures such as gastrointestinal, bladder and peritoneal lavage of warmed fluids. Answer A: Administering cooled intravenous fluids would further exacerbate hypothermia, which can eventually lead to a fatal dysrhythmia. Cooled intravenous fluid administration would be a plausible therapy for hyperthermia. Osborne or "J" waves are not associated with hyperthermia and these electrocardiographic findings should hinder choosing cooled fluids. Answer B: The questions stem is inconsistent with an opiate overdose. The findings of decreased respiratory rate < 12, coma, miosis and sequelae of drug use (finding needles, drug paraphernalia, needle marks) would be indicative of opioid overdose. Respiratory depression is a major cause of morbidity with opiate use, thus ensuring adequate ventilation and oxygenation is of most importance. Naloxone is a competitive antagonist at all opioid receptors and thus reversed opioid effects including respiratory depression, analgesia and miosis. Answer C: Percutaneous coronary intervention (PCI) or an angioplasty is non-surgical approach to treat coronary artery stenosis or occlusion. During a PCI, a cardiologist will introduce a balloon catheter via a femoral vessel under fluoroscopic guidance to the area of coronary artery blockage. Following this the catheter is guided at the blockage and the balloon is inflated to open the site of stenosis, which is followed by placement of a stent. Since Wellens syndrome represent a severe preinfarction stenosis of left anterior descending artery with potential for a fatal anterior wall myocardial infarction these patients should undergo angiography to determine the need for PCA or coronary artery bypass graft (CABG). PCA is not however indicated in the treatment of hypothermia. Answer D: There is no indication to administer sodium bicarbonate for either opiate, cocaine or methanol ingestion or overdose. Sodium bicarbonate can be is used in selected circumstances such as overdose with tricyclic antidepressants overdose, hyperkalemia or salicylate poisoning with the goal of alkalinizing the urine but has no indication for hypothermia. Sodium bicarbonate administration has significant adverse effects such as metabolic alkalosis; cerebral edema due to increasing sodium loads, congestive heart failure and hypernatremia.

An 80-year-old male presents with severe, diffuse abdominal pain that started today and is associated with bloody stools. Past medical history is pertinent for atrial fibrillation for which he takes metoprolol. Physical examination reveals markedly diminished bowel sounds and obvious rigidity. Laboratory studies reveal normal lipase level. The most likely diagnosis is A. biliary obstruction B. colorectal cancer C. diverticulitis D. ischemic colitis E.ulcerative colitis

The correct answer is: D As the name implies, ischemic colitis is a lack of blood supply to some portion of the colon, which results in associated tissue death. This condition is commonly seen in elderly patients and is most commonly due to either low blood pressure or a thromboembolic event. It is important to note that the superior mesenteric artery supplies the ascending and transverse colon while the inferior mesenteric artery supplies the descending colon and proximal rectum. Additionally, the watershed zone at highest risk for ischemic events due to low blood pressure is at the splenic flexure. The location and segments involved in the ischemic event assist in discovering the etiology. In this case, the etiology is unclear. The patient may have thrown a clot due to his atrial fibrillation or he may have decreased blood flow due to the beta blocker medication lowering his blood pressure. These patients will classically present with "pain out of proportion to exam", bloody stools, and some vascular risk. Answer A: Biliary obstruction is most commonly due to gallstones occluding the cystic or common bile duct. Biliary obstruction leads to a classic colicky, sharp pain in the right upper quadrant that commonly radiates to the shoulder. This pain occurs right after a fatty meal as the gallbladder contracts to secrete bile, but cannot as it is obstructed. The most common risk factors for gallbladder disease are the 4 F's: Female, Forty, Fertile (pregnant) and Fat (overweight). Answer B: Colorectal cancer is the third most common type of cancer for both men and women and typically affects patients over the age of 50. Cases can present with vague or dull mild abdominal pain and bloody stools. However, these signs are usually not acute in onset and can be present for years in minimal amounts before patient's seek medical attention for them. Additional symptoms include, nausea, vomiting, and weight loss. Answer C: Chronic constipation and increased strain with defecation are risk factors that lead to the development of small diverticula involving the mucosal wall. Bacteria can accumulated in these pouches and cause infection/inflammation, which is called diverticulitis. Diverticulitis typically presents with left lower quadrant abdominal pain that is tender to palpation, fever, and occasionally blood per rectum. Bowel rest for a time and then a change in fiber intake is typical for treatment, with antibiotics also beings used for moderate to severe cases. Multiple episodes within a short period of time or acute perforation/necrosis are indications for surgical consult. Answer E: Ulcerative colitis falls under the umbrella of inflammatory bowel diseases and characterized by predominately colonic and rectal involvement with relative perianal sparing, pseudopolyps, continuous inflammation, and shallow mucosal ulcerations. This disease is often compared and contrasted to Crohn's disease. There is a bimodal age distribution for ulcerative colitis with it commonly affecting adolescents in their late teens to early twenties and then also occurring in 50-60 year olds. While the above case is similar to a ulcerative colitis presentation, the age is not consistent and given the history ischemic colitis is much more likely. Bottom Line: Keep ischemic colitis in your differential for patients over the age of 60, with vascular risk factors , acute abdominal pain out of proportion to exam, and hematochezia.

A 42-year-old male presents with right lower extremity pain and is found to have a broken tibia. The patient is admitted to the hospital, but the next day, has worsening leg pain. Physical examination reveals the right lower extremity is ecchymotic, cool in temperature, very hard to palpation, and without a pulse appreciated on the dorsalis pedis. The definitive management for the patient's condition is A. alteplase infusion B. angiography C. angioplasty D. fasciotomy E. heparin infusion

The correct answer is: D Compartment syndrome describes the increased pressure within a limited space within the body that compromises circulation and function of tissues within that space. Prolonged elevation of pressure can cause nerve damage and tissue death. The normal compartment pressure is < 10 mm Hg. Tissue pressures in excess of 30 mm Hg for prolonged periods have been classically associated with poor outcomes. There is data, however, that supports "delta pressure" as a better determinant of potential damage. This value is obtained by subtracting intracompartmental pressure from diastolic pressure. The critical level has been found between 10 mm Hg and 35 mm Hg. Historically the symptoms of compartment syndrome were associated with five P's - pain, paraesthesia, pallor, pulselessness, and poikilothermia. These symptoms may not be present at the same time. Clinically the compartment is often swollen, tense, and tender to squeezing. Common causes of compartment syndrome include fractures, typically involving tibia and forearm, hemorrhage, constrictive casts, crush injuries, IV drug injection, and burns. Fasciotomy should be performed as soon as a diagnosis is confirmed with compartment pressure or "delta pressure" measurements. Permanent damage can occur with greater than eight hours of ischemia. Impairment is less likely if treated within six hours of onset. Answer A: Thrombolytic therapy is indicated for initial treatment of massive pulmonary embolism and certain cases of STEMI where treatment with percutaneous coronary intervention is not possible or indicated. It is not involved in the management of compartment syndrome. Answer B: Angiography as well as consultation with a vascular surgeon can be used to confirm a diagnosis of occlusive arterial disease. It is not involved in the management of compartment syndrome. Answer C: Percutaneous transluminal angioplasty with a vascular surgeon is definitive treatment for occlusive arterial disease not compartment syndrome. Answer E: Heparin infusion may be indicated in cases of deep venous thrombosis or occlusive arterial disease. However, the case above describes compartment syndrome as it follows directly after extremity injury. Bottom Line: Fasciotomy is the definitive treatment for compartment syndrome.

Myocardial infarction

Lateral wall myocardial infarctions involve ST-segment elevation in leads V5,V6, I and AVL. With these type of MI's, there can be dysfunction of the left ventricle, causing congestive heart failure. Treatment of myocardial infarctions includes immediate consultation with an interventional cardiologist for percutaneous coronary intervention (PCI) so that the affected vessel can be opened up (angioplasty) and stented open. Other therapy includes, oxygenation, aspirin, and discussion with the cardiologist in terms of other medications that they would like administered (such as heparin, ADP receptor antagonists (clopidogrel or ticlopidine),beta blockers, statins, GP IIb/IIIa inhibitors (abciximab, eptifibatide, and tirofiban), and ACE inhibitors. In this case you see TART changes from T1-4 which are representative of viscerosomatic reflexes from cardiac pathology. Although this does not help you determine the location of the infarction, it helps you confirm your diagnosis. Answer A: Anterior wall myocardial infarctions cause ST-segment elevation in leads V3 and V4 (can include V1 and V2 which indicate a septal region of infarction as well) with associated reciprocal changes in the inferior leads (II, III and AVF). Answer B: Anterolateral wall myocardial infarctions involve ST-segment elevation in leads V1-V6, and occasionally I, and AVL. Answer C: An inferior wall STEMI will display ST-segment elevation in leads II, III, and AVF indicating an inferior wall myocardial infarction. There will also be reciprocal (electrically opposite area of heart) ST-segment flattening in the anterior leads (V1-V3). With inferior myocardial infarctions anticipate a right ventricular or posterior infarct. To determine if there is a right-sided infarct, lead V4 can be switched to the right side of the chest and the EKG repeated. If there is ST elevation in RV4, then there is likely to be a right ventricular infarct and the patient is therefore pre-load dependent and requires IV fluids to maintain cardiac output. Administration of nitroglycerin is cautioned as this can cause a drop in the preload and therefore a drop in the total cardiac output. Answer D: Inferolateral wall myocardial infarctions involve ST-segment elevation in leads V4-V6 and II, III, and AVF.

A 5-year-old male presents to the emergency department by ambulance after being found face down in the swimming pool by his nanny. A bystander was performing CPR when EMS arrived who subsequently intubated with achievement of ROSC in the field. Vital signs reveal a blood pressure of 105/70 mmHg, heart rate of 140/min, respirations at 12/min, and temperature of 37.2ºC (99ºF). Current ventilator settings are as follows: Ventilation assist control tidal volume 200 mL Respiratory rate 12/min FiO2 100% I:E ratio 1:2 PEEP 5 An arterial blood gas is performed revealing the following: pH 7.28 PaO2 400 mmHg PaCO2 55 mmHg HCO3 22 mEq Which of the following is the most appropriate modification to the patient's ventilator settings? A. decrease FiO2 and increase respiratory rate B. decrease respiratory rate C. increase I:E ratio to 1:4 D. increase PEEP and increase respiratory rate E. increase tidal volume

The correct answer is: A When managing a patient on the ventilator, it is important to remember that the tidal volume has to do with protecting the alveoli. A lower tidal volume is more protective of alveoli whereas a higher tidal volume can cause barotrauma by alveolar rupture. The respiratory rate will determine the CO2 level and subsequent pH. A lower respiratory rate causes hypoventilation, a lower pH, and respiratory acidosis whereas a higher respiratory rate causes hyperventilation, a higher pH, and respiratory alkalosis. The I:E ratio is important especially in asthmatics and COPD patients because they need a longer expiratory time, otherwise they will start breath stacking on the vent. When this happens, the patient does not fully exhale before the vent gives the patient another breath. This causes increased intrathoracic pressure and can cause hypotension. When this occurs, the first thing that needs to be done is for the patient to be disconnected from the vent completely and for the chest to be compressed bilaterally to get all of the air out of the lungs. Once this is done, the patient can be reconnected to the vent. This is why asthmatics and COPD patients need a 1:4 I:E ratio. PEEP is used to keep the airways and alveoli open during exhalation and it is physiologic. Sometimes patients may be hypotensive and it may be necessary to decrease PEEP or turn it off completely until the blood pressure can be improved with IV fluids or pressors. Most patients will tolerate 5 mmHg of PEEP, and this is especially important in drownings because the PEEP will help to keep alveoli open as well as recruit more alveoli that are collapsed. The FIO2 is the percentage of oxygen given to the patient. Most patients who get intubated start at 100%, but once the PaO2 comes back, this can be decreased, especially when the PaO2 is elevated. Most patients are on 40-60% FIO2. Answer B: Decreasing respiratory rate will only increase the CO2 level and worsen the patient's acidosis. The respiratory rate needs to be increased to blow off excess CO2 and decrease the respiratory acidosis. Answer C: The I:E ratio represents the inspiration to expiration time ratio. This patient is not an asthmatic therefore increasing the I:E ratio is not necessary. An I:E ratio of 1:2 is adequate for this patient, and, importantly, patient expiratory time may be shorter or longer than the amount of time allotted. Answer D: PEEP means positive end-expiratory pressure. Increasing PEEP will only increase oxygenation, and this patient's O2 is already above what it needs to be. Normal PaO2 is 60-90 mmHg. Answer E: Increasing the tidal volume would only increase the risk of barotrauma. It would not improve the respiratory acidosis. Bottom Line: In this scenario, the ABG shows respiratory acidosis with elevated PaCO2 level and low pH. It also shows that the patient's FIO2 needs to be decreased so that he does not develop oxygen toxicity. COMBANK Insight : You do not need to understand the complexities of ventilator management for the COMAT, however, you should apply your general knowledge of physiology and acid-base mechanics when answering this question. The normal reference ranges for an ABG are available on the Lab Document.

An 38-year-old male is brought to the emergency department by police. He is unconscious with a blood alcohol concentration of 0.36. After fluid resuscitation he awakens and requests to be discharged immediately. Examination reveals an unsteady gait requiring assistance. Shortly thereafter he becomes belligerent and no longer cooperates. It is most appropriate to A. admit the patient for continued observation B. allow the patient to leave against medical advice C. discharge to home D. have police place a medical hold on the patient E. observe the patient in the emergency department until sober

The correct answer is: E This patient can be restrained in the ED until he is safe for discharge home. Unfortunately the patient does not have the capacity to make the right choices for himself, so as his doctor, you must assume that responsibility. At this time, the patient needs to be restrained and continued to be monitored in the ED until he is clinically sober and safe for discharge. If the patient leaves, he is a danger to himself and is at risk for injury. Answer A: You do not have to admit the patient if he does not meed admission criteria, but you also cannot discharge him home if he is unsafe for discharge. Answer B: This patient does not have the capacity to make a decision regarding discharge home. Answer C: This patient cannot be discharged home because he is clinically intoxicated and is unsafe for discharge considering that he is too unstable to ambulate on his own. Answer D: Since the patient is already in the ED, there is no need for the police to place a medical hold. Police can escort patients to the ED if a call from a worried neighbor or someone in the community is witnessing an individual who is in danger to himself or another individual. The police can then bring the patient to the ED where they will be evaluated and potentially held for 48 hours. Answer E: You cannot observe the patient if he becomes violent and storms out of the emergency department, therefore, a medical hold needs to be placed on the patient because he is a danger to himself. This way, the patient can be physically restrained if he tries to leave. Bottom Line: If a patient is a danger to himself in the ED then he must stay under medical supervision until the alcohol wears off and he can be discharged home.

A 46-year-old female presents to the emergency department for evaluation of lower abdominal pressure due to urinary retention for the past 6 hours. History reveals she was about to board a cruise ship for a weekend getaway with her husband, but had to come to the hospital because of her discomfort. Vitals reveal a blood pressure of 155/89 mmHg, heart rate of 121/min, and a temperature of 39.4ºC (103ºF). Abdominal exam reveals a distended bladder without peritoneal signs noted. The skin is flushed and dry. The most likely causative agent is A. anticholinergic B. carbon monoxide C. cholinergic D. opioids E. serotonin

The correct answer is: A Anticholinergic syndrome is characterized by tachycardia, dry skin and mucous membranes, mydriasis, flushed skin, urinary retention, hyperthermia, and altered mentation/agitation. The phrase "hot as a hare (hyperthermia), blind as a bat (mydriasis), dry as a bone (dry skin/mucous membranes), red as a beet (flushed), and mad as a hatter (altered mentation)" captures the major symptoms seen with this syndrome. The patient in this scenario was tachycardic, had dry skin, and had urinary retention. She likely used a scopolamine patch in preparation for her cruise, which can cause anticholinergic syndrome. Other causes include Diphenhydramine as well as Tricyclic antidepressants. Treatment is mostly supportive care with IV fluids and Benzodiazepines as needed, however, Physostigmine should be used if the patient has severe symptoms. Answer B: Carbon monoxide poisoning can present variably with symptoms of altered mentation, loss of consciousness, chest pain, headache, nausea, or vomiting. It is most common in the winter months and usually has to do with poor ventilation while an engine is running or a poorly ventilated heating system. Answer C: Cholinergic syndrome is usually due to organophosphates (insecticide), and symptoms follow the pneumonic "DUMBBELS": Diarrhea, Urination, Myosis/Muscle weakness, Bronchorrhea, Bradycardia, Emesis, Lacrimation, Salivation/Sweating. Treatment includes atropine and pralidoxime. Answer D: Opiate overdose usually presents with CNS and respiratory depression, miosis, and hypothermia. Answer E: Serotonin syndrome usually presents with symptoms of anxiety/agitation/delirium, tachycardia, hypertension, vomiting, diarrhea, hypertonia, hyperthermia, myoclonus ,and hyperreflexia. It is usually medication related, and treatment is based on how severe the symptoms are. Most patients only need to stop the offending agent and receive supportive care with oxygen, IV fluids, and benzodiazepines. If the patient's symptoms are more severe, cyproheptadine can be used. Bottom Line: Anticholinergic syndrome should be at the top of the differential for a patient presenting with tachycardia, urinary retention, and flushed/dry skin. This is the correct answer. COMBANK Insight : Recognizing toxidromes and knowing their treatments is high yield for the Emergency Medicine COMAT. Consult a review book and solidify this information to memory.

A 45-year-old male is brought to the emergency department by EMS after he was rescued from his burning house. He complains of mild shortness of breath at this time with a headache, nausea, and vomiting. When you ask him his name or where he is, he is unable to respond correctly and appear altered. Observation reveals his face appears flushed with evidence of soot around his nostrils and mouth. Which of the following will confirm the most likely diagnosis? A. arterial blood gas B. basic metabolic panel C. electrocardiogram D.plain film radiography of the chest E. pulse oximetry

The correct answer is: A Carbon monoxide poisoning is most common during the winter months in cold climates. Symptoms include headache, nausea, vomiting, and altered mental status. The most common symptom is headache, and many patients may feel like they have a viral illness. ABG is the diagnostic test for carbon monoxide poisoning to measure carboxyhemoglobin levels. A level >15% in smokers and >3% in non-smokers with the appropriate clinical presentation is diagnostic of carbon monoxide poisoning. Treatment is with high flow oxygen. Hyperbaric oxygen is necessary if CO levels are greater than 20%. Answer B: Electrolytes may reveal hypokalemia and hyperglycemia with severe intoxication. Creatinine may be indicative of an acute renal failure secondary to myoglobinuria. Answer C: Acute coronary syndrome is common in the setting of carbon monoxide poisoning, but it will not diagnose CO toxicity. Answer D: A patient who presents from a house fire with altered mental status should be worked up for carbon monoxide poisoning, which is done by measuring the level of carboxyhemoglobin on an ABG. Answer E: Standard pulse oximetry does not differentiate carboxyhemoglobin from oxyhemoglobin. Oxygenation may still remain falsely elevated. Bottom Line: ABG is the diagnostic test for carbon monoxide poisoning to measure carboxyhemoglobin levels. A level >15% in smokers and >3% in non-smokers with the appropriate clinical presentation is diagnostic of carbon monoxide poisoning.

A 9-year-old male presents to the emergency department unconscious after getting hit in the head with a metal baseball bat at the park. Physical examination reveals periorbital ecchymosis. He does not open his eyes to pain, is only moaning incomprehensible words, and withdraws extremities to pain. Analysis of the patient's nasal fluid reveals the presence of cerebrospinal fluid. The most likely diagnosis is A. basilar skull fracture B. epidural hematoma C. nasal bone fracture D.subarachnoid hemorrhage E. subdural hematoma

The correct answer is: A In basilar skull fracture, there is a fracture through the base of the skull involving the sphenoid bone, ethmoid bone, temporal bone, and/or the occipital bone. These type of fractures require a high mechanism of injury and can cause tearing of the meninges which can cause leaking of cerebrospinal fluid. The cerebrospinal fluid can build up behind the tympanic membrane and leak out the eustachian tube resulting in CSF rhinorrhea. If there is a rupture of the tympanic membrane, the patient will display CSF otorrhea. The patient can also have CSF rhinorrhea if there is a fracture of anterior skull. One can tell if the ear or the nose are leaking CSF fluid by placing a drop of the fluid on a paper towel. The blood should coalesce in the center and former ring of clear fluid on the periphery indicating CSF fluid. Other signs of basilar skull fracture include: Seventh cranial nerve palsy, hemotympanum (visual blood behind the tympanic membrane), periorbital ecchymosis (raccoon eyes), mastoid ecchymosis (Battle's sign), vertigo. Management includes performing a CT scan of the head, cervical spine, and facial bones to look for other pathology, consultation with a neurosurgeon and airway management if necessary. Consider discussion of prophylactic antibiotics with the neurosurgeon. Answer B: An epidural hematoma is caused by significant blunt trauma to the head causing accumulation of blood between the inside of the skull and the dura mater. Trauma is usually located over the temporal or temporoparietal region of the patient's skull where the middle meningeal artery is located. This can lead to high pressure arterial bleeding and can lead to brain herniation within hours after the injury. The classic history usually involves blunt trauma to the head with loss of consciousness or altered mental status followed by a lucid interval, then followed by rapid neurologic deterioration. Diagnosis is by history and physical exam, and CT scan. On CT scan and epidural hematoma has a classic by convex or football shaped appearance and is usually located in the temporal region. Management includes early neurosurgeon consultation as evacuation of the epidural hematoma prior to brain herniation reduces morbidity and mortality. If no brain herniation or neurologic deficits occur, a full recovery after hematoma evacuation can be expected as underlying brain parenchymal injury is usually absent. Answer C: Physical exam findings of nasal fractures include: nasal deformity asymmetry or swelling, epistaxis, palpable deformity, septal hematoma. Diagnostic tests include nasal radiographs or CT scan of the facial bones. Note that CT scan of the facial bones is the test of choice to detect other facial bone fractures, skull fractures, and to detect nasofrontoethmoid injuries. Treatment includes dressing of wounds, packing of the nose if the epistaxis cannot be controlled, and management of septal hematomas. Patients should be referred to otolaryngologists for further management and prophylactic antibiotics should be prescribed if nasal packing is required. Answer D: A subarachnoid hemorrhage is most commonly due to a rupture of a cerebral arterial aneurysm. Aneurysm rupture causes an extravasation of blood directly into the cerebrospinal fluid, which increases intracranial pressure. This can result in a cerebral herniation, coma, or death. Other causes of subarachnoid hemorrhage include ruptured arteriovenous malformations, bleeding diathesis, trauma, amyloid deposition, and illicit drug use. A ruptured congenital berry aneurysm most commonly occurs at the junction of the anterior communicating branch and the anterior cerebral artery. Symptoms of a subarachnoid hemorrhage begin abruptly, with sudden onset of a severe thunderclap headache which is commonly described as the "worst headache of my life." The onset of the headache may or may not be associated with a brief loss of consciousness, seizure, nausea, vomiting, focal neurologic deficits, or stiff neck. The stiff neck can present similar to meningitis because blood is accumulating in the subarachnoid space, and is very irritating to the meninges. A positive Brudzinki or Kernig's sign might be present on physical examination. First step in management is a non-contrast brain CT scan. If this is equivocal, a lumbar puncture can be performed which would show the presence of xanthochromia and red blood cells in the fluid. Answer E: A subdural hematoma is a type of intracranial hemorrhage in trauma that is more common in the elderly and alcoholics. This is because of the increased atrophy of the brain in this population which leads to increased movement of the brain causing shearing forces on the bridging veins. The bridging veins are in between the arachnoid mater and dura mater-- hence, subdural hematoma. Diagnosis of a subdural hematoma is by noncontrast CT scan. He subdural hematoma appearance at CT scan is crescent-shaped and the hematoma can cross suture lines unlike epidural hematomas. Management includes airway/oxygenation management, blood pressure management (keep systolic pressure >90 mmHg in head-injured patients), increased intracranial pressure management, and consultation of neurosurgeon. Surgical intervention is usually required for acute (less than 24 hours) and subacute (less than 2 weeks) hemorrhages, and any bleeds associated with a significant midline shift or altered mental status. Bottom Line: Basilar skull fractures should be suspected in patients with periorbital ecchymosis, otorrhea, rhinorrhea, battle's signs. Although the above injuries can occur when a patient has a basilar skull fracture, none of the other injuries will cause findings of battle's signs, periorbital ecchymosis, etc.

A 32-year-old male presents with recurrent episodes of acute low back pain resulting in the inability to stand. This is his fourth episode in the past two years. History reveals the pain radiates into his sacroiliac joints and left buttocks. The pain is worse in the morning and improves through the day. Structural examination reveals a negative straight leg raise test bilaterally and both PSIS are equal heights. The most likely diagnosis is A. acute psoas spasm B. fibromyalgia C. larsen syndrome D.lumbar disc herniation E. short leg syndrome

The correct answer is: A Just as it sounds, acute psoas spasm is a flexion contracture of the iliopsoas muscle which causes unilateral torso flexion and side-bending. Patients typically complain of lumbar pain, specifically at L1-L2, and increase pain with standing straight and walking that radiates to the groin or buttocks. For clinical purposes, the main thing to remember is not to stretch an acute spasm as it can make the contracture worse. Counterstrain and Muscle Energy techniques are very helpful. Answer B: Fibromyalgia is a condition involving chronic musculoskeletal pain that has a relatively unknown etiology, but is believed to be caused by a mix of psychologic and genetic factors. This disease is diagnosed with bilateral pain affecting the multiple locations that lasts for at least 3 months time. There are 18 designated painful tender points classically involved. The low back and posterior upper thigh/buttock area are two places commonly affected in Fibromyalgia, however this scenario does not detail constant pain for 3 months and the pain described is not widespread. Answer C: Larsen Syndrome is a congenital disorder due to a genetic defect in actin associated proteins. These patients typically present either in infancy or early childhood with several cardiac, facial, and musculoskeletal anomalies including dislocation of knees, hips, and shoulders. Answer D: Disc herniation is a condition involving the nucleus pulposus bulging out of the intervertebral disc. This common affects the lumbar spine and herniations are typically posterior and can potentially compress the exiting spinal nerves. Patient's will often complain of low back pain with associated numbness/tingling, particularly along the sciatic nerve running posterior down the leg. This patient's symptoms are very similar to lumbar herniation, however in disc herniation the straight leg raise test is typically positive. Answer E: Short leg syndrome refers to a discrepancy in leg lengths. This is often diagnosed in children due to a congenital deformity or as a sequelae to trauma causing asymmetric growth. Additional, a functionally shortened leg can be seen in adults secondary to posterior innominate rotation and can be a cause of low back pain. For this case, you would expect the PSIS to be inferior on the side of the shortened leg. Bottom Line: Acute psoas spasm causes torso flexion and presents as low back pain radiating to the pelvic area and increased pain with standing.

A 33-year-old male presents to the emergency department with shoulder pain after falling on an outstretched hand while playing basketball. He denies any associated numbness or tingling. Physical examination reveals a patient in moderate discomfort who is holding the affected extremity in a slightly abducted and externally rotated position. There is a palpable depression just below the acromion. Range of motion and muscle strength testing are limited due to the pain associated. A plain film radiograph of the shoulder is obtained as shown in the exhibit. The most likely injured structure associated with this type of injury is A. axillary nerve B. musculocutaneous nerve C. radial nerve D.subclavian artery E. ulnar nerve

The correct answer is: A Patients can present to the emergency department with dislocated shoulders after trauma to the shoulder. Clinically, the patient is unable to move the shoulder, has a palpable dip just below the acromion (sulcus sign) and usually holds the shoulder in a abducted and slightly externally rotated position. Obtaining neurovascular (i.e. checking for capillary refill and checking the sensation and strength of muscles distal to the injured joint) status prior to any treatment is paramount. It is imperative to obtain a two view x-ray of the shoulder including an AP view and an axillary or Y-view to determine the position of the humeral head in relationship to the glenoid fossa. Treatment includes reduction (in which there are many methods), applying a shoulder sling, and orthopedic follow-up. Complications of a shoulder dislocation include a Bankart lesion (a tear of the labrum), a Hill-Sachs deformity (a defect on the humeral head secondary to the injury), and an axillary nerve injury. An axillary nerve injury presents with loss of sensation on the lateral aspect of the affected shoulder. Note that axillary nerve injuries are as high as 40% in shoulder dislocations. Answer B: The musculocutaneous nerve is involved in flexion of the elbow and shoulder joint and supination of the forearm. The musculocutaneous nerve gives rise to the lateral cutaneous nerve which provides sensation to the lateral aspect of the forearm. Injury to this nerve is not associated with shoulder dislocations. This is a highly protected nerve and the most common cause of an injury to this nerve is a stab wound to the axilla. Answer C: Radial nerve injuries are associated with mid shaft humerus fractures due to the coursing of the nerve close to the shaft of the humerus. Signs and symptoms include wrist drop (loss of extension of the wrist and finger extensors), loss of the forearm supinators, and can affect strength of the tricep muscles as well. Numbness occurs over the dorsoradial aspect of the radial three and a half digits and the dorsoradial aspect of the hand. Answer D: The subclavian artery is protected by the clavicle and the large upper extremity muscle girdle. Injury to the subclavian artery is rare and can occur from major trauma associated with severe clavicular fractures, deceleration injuries, and first and second rib fractures. Clinically, one should possibly see swelling and ecchymosis along the course of the artery and decreased or absent pulses of the affected extremity. Answer E: The ulnar nerve motor functions are: ulnar deviation of the wrist, flexion of the wrist (the ulnar side), flexes part of the fingers (ulnar half of flexor digitorum profundus), finger abduction and adduction (interossei muscles), adduction of the thumb (adductor pollicis). Sensation includes the ulnar and dorsal aspect of the fifth and ulnar half of the fourth digit. This nerve can be injured by fracturing the medial at the condyle at the elbow joint. Injury to this nerve is not commonly associated with shoulder dislocations. Bottom Line: Shoulder dislocations can occur after trauma to the shoulder. Checking neurovascular status prior to reduction is imperative. The most commonly injured structure associated with shoulder dislocations is the axillary nerve and injury can cause anesthesia over the lateral deltoid muscle of the affected shoulder.

A 45-year-male presents to the emergency department after being bitten by a dog approximately 90 minutes ago. The man states he was accidentally bitten by his own dog after trying stop a fight between it and another dog in the park. He states his dog is current on all of its vaccinations. History reveals no medical conditions with allergies to IV contrast dye and sulfa drugs. His vital signs are heart rate 89/minute, blood pressure 135/82 mmHg, respirations are 14/min and oxygen saturation is 98% on room air. Physical examination reveals 4 small lacerations on the right forearm without maceration. You note painful muscle spasms of the paravertebral musculature from T1-4. What is the most appropriate pharmacological management? A. amoxicillin-clavulanic acid B.cefuroxime C. doxycycline D. levofloxacin E. trimethoprim-sulfamethoxazole

The correct answer is: A The most common pathogens incurred in mammalian bites are important to know. Dog bites have Pasteurella, Capnocytophaga canimorsus, streptococcal, and staphylococcal as the most common organisms. Amoxicillin-clavulanic acid is the treatment of choice with these conditions and should be prescribed for a 5-7 day course. It is prudent to treat all infected wounds and to prescribe antibiotics for high risk infections. High risk infections include: Cat or human bites, livestock, monkey bites, deep puncture wounds, hand/foot wounds, bites in immunosuppressed patients. The TART changes present in this vignette are due to viscerosomatic changes from the upper extremity which will present at T1-4. Answer B: Cefuroxime is a second generation cephalosporin that is an acceptable alternative to the patient who has an allergy to the amoxicillin-clavulanic acid. There is a small amount of cross-reactivity between patients who have allergies to the penicillin class of antibiotics and to cephalosporins. This cross-reactivity in an anaphylactic reaction ranges from 1:1000 to 1:1,000,000. In general in patients with a minor reactions such as minor urticaria or GI upset with penicillins there is very little chance of any interactions. Answer C: Doxycycline is a tetracycline and is also indicated for use in the patient with an allergy to the drug of choice, amoxicillin-clavulanic acid. This is the drug of choice for several tick born illnesses such as Rocky Mountain spotted fever, Babieosis and Erlichiosis. Doxycycline should be used with caution as it may cause discoloration of the teeth and can cause a hypersensitivity to the sun. Answer D: Levofloxacin is a fluoroquinolone that may be used for a dog bite but may not be used in monotherapy. Levofloxacin is frequently prescribed with clindamycin for patients with severe (anaphylaxis) type reactions to penicillins and cephalosporins. Monotherapy is always preferable to polytherapy as compliance is always higher. Fluoroquinolones must be used with caution as they can cause tendon rupture. Tendon ruptures most commonly occur in middle age men at the Achilles tendon. Answer E: Sulfamethoxazole-trimethoprim would be a reasonable alternative for some, however, this patient has an allergy to sulfa drugs and therefore would be contraindicated in this patient. Bottom Line: Amoxicillin-clavulanic acid is the first line treatment for a dog bite.

A 49-year-old male presents to the emergency department with abdominal pain, nausea, and vomiting with associated anorexia of 3 days' duration. Social history is significant for 3-6 beers per day. Physical exam reveals a disheveled male. Laboratory studies reveal: pH 7.2 sodium 140 mEq/L chloride 102 mEq/L BUN 22 mg/dL bicarbonate 12 mEq/L creatinine 1.5 mg/dL glucose 130 mg/dL ketones, serum large Which of the following should be provided in addition to thiamine? A. glucagon B. glucose and normal saline C. insulin and sodium bicarbonate D. kayexalate E. N-acetylcysteine

The correct answer is: B Alcoholic ketoacidosis most often occurs in patients with a history of chronic alcohol abuse and malnutrition. Most episodes are secondary to binge drinking and patients will usually develop symptoms of abdominal pain, nausea, and vomiting. Secondary to these symptoms, patients will not be able to continue drinking which precipitates alcohol withdrawal and further exacerbates their symptoms. By the time patients present to the hospital, it is usually 2-3 days after onset of their symptoms. Alcohol is converted in the liver to acetic acid, which is used by the body as an energy source, so despite malnutrition, the body continues to use the acetic acid for energy. The acetic acid can either be converted to a fatty acid or can be broken down into one of 3 ketone bodies (acetoacetic acid, beta-hydroxybutyric acid, or acetone). When alcohol ingestion ceases, the body needs an alternative energy source, so it starts breaking down fatty acids in order to use ketones as energy. Low levels of insulin and high levels of glucagon are the triggers that cause the body to start fatty acid breakdown. Alcoholic ketoacidosis is diagnosed by the presence of ketones in the blood and urine as well as an elevated anion gap. Unlike diabetic ketoacidosis, blood glucose levels are not elevated. Noted electrolyte abnormalities typically associated with alcoholic ketoacidosis includes hypophosphatemia, hypokalemia, and hypomagnesemia. In alcoholic patients, 100 mg of thiamine should be given via the intravenous or intramuscular route. It has classically been taught that Thiamine should be given prior to glucose in alcoholic patients to decrease exacerbation of Wernicke's encephalopathy, although this theory is still controversial. The underlying pathophysiology is thought to be related to the fact that glucose oxidation in the brain requires thiamine, which causes already low levels of thiamine in the circulation to be driven intracellularly and further lowers the available levels of thiamine in the circulation. This is thought to precipitate neurologic injury. Thiamine deficiency in long-term alcoholics, is thought to be secondary to malnutrition which causes decreased intestinal thiamine absorption by approximately 70%. Treatment of alcoholic ketoacidosis includes giving thiamine first, followed by glucose administration along with IV fluids to rehydrate the patient. Giving glucose will increase insulin secretion and reduce glucagon secretion which will trigger cessation of fatty acid breakdown. Electrolyte abnormalities should be corrected with administration of phosphate, potassium, and magnesium. Answer A: Glucagon administration would worsen diabetic ketoacidosis, since low levels of insulin and HIGH levels of glucagon stimulate the liver to start fatty acid breakdown. Answer C: Insulin is usually not given in alcoholic ketoacidosis since most of the time the patient is not hyperglycemic. Despite the metabolic acidosis, sodium bicarbonate is not part of the treatment of alcoholic ketoacidosis. Giving glucose and normal saline is enough to stop fatty acid breakdown and inhibit ketone formation, thereby promoting resolution of anion gap metabolic acidosis. Answer D: Kayexalate is given for hyperkalemia. Alcoholic ketoacidosis is usually associated with HYPOkalemia. Answer E: N-acetylcysteine is the antidote for Tylenol poisoning, and should not be given in alcoholic ketoacidosis. Bottom Line: Giving glucose and IV fluids will allow the body to stop using fatty acids for energy thereby decreasing formation of ketones and resolving the ketoacidosis.

A 36-year-old male has a seizure while at work and is transported to the emergency department. A fellow employee is present and he describes that the patient slumped to the ground and had a 30 second episode of "whole body shaking." After this occurred, the patient appeared to be alert and was able to answer questions, however the patient states that he does not remember the episode nor the ambulance ride. He denies a history of epilepsy or other significant medical history and denies taking any medications or using illicit drugs. The physical examination is interrupted when the patient slumps back into the bed and starts shaking his upper and lower extremities. During the episode, his eyes are open, he is yelling profanities. This occurs for approximately 2 minutes. Afterward, he is alert and oriented to person, place, and time, and he is at his baseline mental status per his fellow employee. The patient denies loss of bowel or bladder incontinence during these episodes. Which of the following is the most appropriate management at this time? A. CT scan of the head B. electroencephalogram C.lorazepam D. lumbar puncture E. serum phenytoin level

The correct answer is: A The patient in this history likely has not experienced a real generalized epileptic seizure. There are factors in the history that are not indicative of a real seizure including the fact that he had no evidence of a post-ictal period. When the patient "seized" during the exam, he had other factors that were indicative of a psychogenic non-epileptic seizure such as the fact that he was able to speak during the episode, he had no loss of bowel and bladder, and he had no post-ictal period. However, it is imperative to fully evaluate the patient for a possible epileptic seizure to rule one out. Management includes CT brain in the emergency department followed by admission/observation with electroencephalogram (EEG) and neurology consultation. Patients with psychogenic non-epileptic seizures (PNES) are misdiagnosed 25% of the time and are usually resistant to antiepileptic therapy. Emotional triggers such as stress are common in patients with PNES and patients with PNES typically have audiences when they have a non-epileptic seizure. They typically do not have self-injured patterns such as tongue biting or head trauma as seen in those with epileptic seizures. There can be a history of psychiatric disorders, fibromyalgia, and sexual abuse. They are diagnosed by video EEG monitoring, which may even trigger a fake seizure episode. Treatment includes psychotherapy and adjunctive medications to treat coexisting depression and anxiety. Answer A: A CT scan of the head is required in patients with first time seizures, or if there is concern for a structural brain lesion (focal seizure - i.e. involving one body part or one side of the body). The patient may be "faking" their seizures, but should still be systematically evaluated. PNES should be a diagnosis of exclusion and should never be made in the emergency department. Answer B: EEG is important when trying to rule out seizures, however, this would not be done in the emergency department, rather, after admission to the floor. Answer C: This patient has factors that are not consistent with a real generalized seizure. This is likely a psychogenic non-epileptic seizure and the treatment does not involve benzodiazepines such as lorazepam. Also, the patient's "seizure" has abated on it's own and therefore, lorazepam would not be beneficial. Answer D: A lumbar puncture may be indicated if the patient has a history of findings suggestive of a subarachnoid hemorrhage or meningitis based off of exam or history. Meningitis may have a history of fever, sick contacts, headache, photophobia, vomiting, neck pain and stiffness and on exam the patient may have an altered mental status, petechial or purpuric rash (Neisseria Meningitidis), and nuchal rigidity (neck stiffness). In a patient with a possible subarachnoid hemorrhage they may be complaining of a sudden onset headache described as "the worst headache of their life" with maximum pain at initial onset and may also have nuchal rigidity and altered mental status on exam. Answer E: Checking anti-epileptic therapies are unlikely to be helpful for this patient especially since there is no hx of Phenytoin use. Bottom Line: Regardless of what you think the patient may have, you still need to rule out life threatening etiologies, which in the emergency department would include ordering a CT scan of the brain.

A 16-year-old female is in a motor vehicle collision and EMS is present on the scene. Vitals reveal a heart rate of 165/min, respiratory rate of 45/min, blood pressure of 78/42 mmHg, and oxygen saturation of 86% on room air. Physical examination reveals absent breath sounds on the right with tracheal deviation to the left. The parents are not present and cannot be contacted to provide consent to therapy at this time. Which of the following is the most appropriate course of action? A. needle decompression B. plain film chest radiography C. rapid sequence intubation D.stabilize and wait for the parents to consent to therapy E. tube thoracostomy

The correct answer is: A The patient in this scenario is in extremis and requires immediate intervention to prevent her from progressing to death. She is displaying signs of a tension pneumothorax which includes: Hypotension, tachycardia, tracheal deviation away from the pneumothorax, decreased breath sounds on the side of the pneumothorax, cyanosis, and cardiovascular collapse. The pathophysiology of a tension pneumothorax is that air enters the pleural space through a bronchoaveolar disruption and becomes trapped via a ball-valve effect. This leads to continuous increasing air within the pleural space and thus the mediastinum shifts to the uninvolved side (which is seen as tracheal deviation on exam) and interferes with right heart filling. This leads to decreased left heart filling which leads to decreased cardiac output (and thus hypotension). Treatment is immediate needle decompression by placing a 14 to 18-gauge angiocatheter in the second intercostal space at the mid-clavicular line. A successful needle decompression will be confirmed by hearing air come out of the angiocatheter and improvement in the patient's vital signs. Subsequent tube thoracostomy is required as there is a defect in the pleura. Answer B: A tension pneumothorax is a clinical diagnosis and immediate intervention should be initiated as soon as there is clinical suspicion. It is imperative that the clinician does not wait for chest x-ray confirmation as patients with tension pneumothoraces require immediate intervention and are too unstable to wait for a chest x-ray. Answer C: The patient in this example has respiratory decompensation. However, this is from a tension pneumothorax. Intubation in a patient who has an untreated tension pneumothorax will lead to increased air within the pleural space due to a bronchoaveolar disruption and will likely cause loss of vital signs without appropriate treatment. Answer D: The patient in this example is under age 18 (a minor) and has an emergent medical condition which requires emergent treatment under common US law. Therefore, parental consent is not required in an emergent condition that is limb or life-threatening that this patient is displaying. Answer E: A tension pneumothorax requires immediate decompression with needle thoracostomy followed by chest tube placement. A simple pneumothorax is treated with a chest tube. Those with simple pneumothoraces may be either asymptomatic or present with symptoms such as chest pain and dyspnea. In addition, hyperresonance to percussion may be noted on the affected side. Tension pneumothoraces present in shock with severe respiratory distress and tracheal deviation to the unaffected side. Bottom Line: In a minor with an emergent medical condition parental consent is not required to treat. If a patient has signs and symptoms suggestive of a tension pneumothorax, immediate needle decompression is required.

An 85-year-old female presents to the emergency department with abdominal pain. She admits to recently fracturing her left wrist and is currently taking oxycodone/acetaminophen as needed for pain. Vitals reveal a blood pressure of 119/72 mmHg, heart rate of 67/min, and respirations of 16/min. Physical examination reveals a distended, non-tender abdomen with hypoactive bowel sounds in all 4 quadrants. Rectal exam reveals hemoccult negative stool without rectal masses and there is no stool in the rectal vault. A plain film radiograph of the abdomen is obtained as shown in the exhibit. Which of the following is the most appropriate management? A. bisacodyl B. colonoscopy C. exploratory laparotomy D. loperamide E. nasogastric tube insertion

The correct answer is: A The x-ray displays an abdomen film that shows stool in the right colon which signifies constipation. The patient does not have abdominal tenderness. Therefore, an acute abdomen is unlikely. Also, the patient does not have stool in the rectal vault on exam so disimpaction is unlikely to help. Treatment is centered around exercise, increasing non-caffeinated fluids, increasing dietary fiber intake, and changing medications that cause constipation. The patient in this question likely has constipation related to the fact that she is on opioids for wrist fracture. Medications that can help include enemas, fiber supplements, osmotic laxatives (which draw increased water into the colon to increase the water content of the stool and thus aid in transport), stimulant laxatives (which stimulate intestinal transport), stool softeners, and suppositories. Bisacodyl works by increasing intestinal fluid accumulation by altering water and electrolyte secretion. It also mildly irritates the intestinal smooth musculature thereby increasing peristalsis. Answer B: Emergent consultation of gastroenterology is not indicated in this patient as the likely cause of her constipation is recent opioid use. However, symptoms suggestive of an organic cause of the patient's constipation include acute onset, weight loss, rectal bleeding/melena, rectal pain, fever, and changes in the stool caliber. These findings may warrant further investigation including possible CT scan of the abdomen/pelvis to look for an abdominal mass. Outpatient referral may be helpful in patients with chronic constipation. Answer C: Exploratory laparotomy is performed in those with unexplained abdominal pain and trauma situations in order to obtain additional information that is not available through other diagnostic methods. This patient does not have signs and symptoms consistent with an acute abdomen requiring emergent surgical intervention. Answer D: Loperamide is an antidiarrheal medication that slows intestinal motility through opioid receptors on circular and longitudinal muscles. This is obviously contraindicated in those with constipation. Answer E: The abdominal x-ray in this case shows constipation. If there was dilated intestine and air/fluid levels a small or large bowel obstruction would be suspected. Management of a small and large bowel obstruction includes insertion of a nasogastric tube for decompression of the proximal bowel, NPO status, and a surgical consult along with admission for further management. Bottom Line: Constipation is treated increasing water content, exercise, increasing fiber content, discontinue medications that cause constipation and by various laxatives, enemas, and suppositories.

An 82-year-old male presents to the emergency department with epigastric pain that started 2 hours prior to arrival. The pain is moderate, constant without radiation, and there are no alleviating or aggravating factors. History reveals that he is currently on warfarin for chronic atrial fibrillation and has a history of chronic gastritis. Vitals reveal: Blood pressure: 89/45 mmHg Temperature: 36.6ºC (97.8ºF) Pulse: 105/min Pulse oxygenation: 98% on 2L nasal cannula Physical examination reveals an ill-appearing male with pale conjunctiva and dry mucous membranes. The heart is irregularly irregular and tachycardic without murmurs, rubs or gallops. The lungs are clear to auscultation bilaterally. The abdomen is soft, non-distended, and tender to palpation in the epigastric region without rebound or guarding. Rectal exam reveals no masses but is hemoccult positive. A complete blood count with coagulation studies is obtained and reveals the following: White blood cell count: 9 x 109L Hemoglobin: 9.2 g/dL Hematocrit: 29.1% Platelets: 155 x 109/L INR: 8.5 PT: 65 seconds PTT: 35 seconds The patient is stabilized with IV fluids and vitamin K. Which of the following is the most appropriate additional A. fresh frozen plasma B. heparin C. iron D. packed red blood cells E. platelets

The correct answer is: A This patient has hypotension, supratherapeutic INR, hemoccult positive stool, and epigastric pain. He has an upper gastrointestinal bleed until proven otherwise. Initial management includes supportive care (airway, breathing, and circulation). The patient in this example is taking warfarin which inhibits vitamin K-dependent factors II, VII, IX, X, protein C and S. The INR/PT reflect the changes in the extrinsic clotting cascade and inhibiting vitamin K dependent factors inhibits most of the factors in the extrinsic pathway thus leading to the prolongation of PT/INR. Fresh frozen plasma (FFP) which contains clotting factors, is indicated when there is life-threatening bleeding caused by warfarin, regardless of the patient's INR. FFP contains clotting factors and therefore is the fastest remedy to prevent further life threatening bleeding. Since this patient likely has a GI bleed with associated hypotension, then FFP is indicated. Vitamin K is given as well to increase the production of these factors. However, this takes time and administration of FFP will achieve immediate hemostasis while Vitamin K will take longer as the factors must be synthesized. Answer B: Heparin is an anticoagulant that at low doses acts to bind to antithrombin III and accelerate its activity, thus inhibiting thrombin and factory Xa in the coagulation cascade. At high doses, it inactivates factors IX, X, XI, and XII and thrombin and inhibits conversion of fibrinogen to fibrin. It therefore inhibits many of the factors in the intrinsic coagulation pathway and causes prolongation of PTT. An anticoagulant is the opposite effect which one would want in a patient with an acute life-threatening bleed. Giving an anticoagulant such as heparin in a patient with an upper gastrointestinal bleeding would likely lead to increased bleeding and increased risk of death. Therefore, administering heparin is not indicated in this patient. Answer C: Iron is an important mineral used in the bone marrow to make hemoglobin, myoglobin, and enzymes. It is also utilized in hemoglobin to transport oxygen. Although iron is useful to help promote hemoglobin production in patients with iron deficiency anemia, it is not indicated in the acute setting of a life threatening bleed such as a gastrointestinal bleed. Answer D: This patient is experiencing hypovolemic shock secondary to acute blood loss anemia likely from an upper gastrointestinal bleed. In patients with hypovolemic shock it is necessary to first tend to the airway and breathing, then concentrate on the circulation/ hypotension. Initially, it is imperative to monitor blood pressures, place the patient on a cardiac monitor, place high flow oxygen and placed two large-bore intravenous lines. Infusion of two liters of IV fluids at a rapid pace should occur next. If the patient remains hypotensive despite infusion of crystalloid, then administration of blood products should occur. If the patient has not been typed in screened, then O negative blood should be transfused. If the patient has a type and screen, type specific blood should be transfused. Indications for blood transfusion depend on patient circumstances and clinical picture. Answer E: Platelets act at the end of the coagulation cascade to cross-link with fibrinogen and von Willebrand factor to stabilize a clot. They come in a pool of 6 whole blood derived platelets or one apheresis platelet. This should increase the platelets in a 70 kilogram patient by 30,000-60,000. Initially platelets are not indicated in acute blood loss anemia, however a massive transfusion protocol may be initiated if a patient continues to bleed until definitive control of bleeding can occur. Massive transfusion is defined as transfusion of 10 units of packed red blood cells, or the approximate amount of 1 adult blood volume. Once a patient has received this amount of blood, they have lost a large amount of other components of blood that they have lost from bleeding including platelets and clotting factors. Therefore, it is necessary to replace these factors by administration of packed red blood cells, platelets and clotting factors (in the form of FFP) in a 1:1:1 ratio. Bottom Line: In patients with supratherapeutic INR/PT and acute life-threatening hemorrhage, administration of FFP and vitamin K is indicated.

A 49-year-old female with a history of poorly controlled type 1 diabetes mellitus presents to the emergency department with abdominal pain, nausea, vomiting, and altered mental status since yesterday. Her husband states she is not adherent with her insulin and reports blood sugars of over 500 mg/dL at home. Laboratory studies reveal an anion gap of 18, glucose of 547 mg/dL, and moderate to severe ketones in her urine. What is the most appropriate initial treatment at this time? A. IV fluids B. IV insulin C. IV phosphate D.IV rocephin and vancomycin E. subcutaneous insulin

The correct answer is: A Treatment of DKA starts with volume expansion with IV fluids. The pathophysiology of DKA is as follows: Glucose is the preferred energy source of the body, but without insulin it cannot get into the cells. As a result, the cells use the alternative energy source, fatty acids. Ketones are breakdown products of fatty acids and cause an increase in the anion gap. By placing the patient on an insulin drip, glucose is once again brought into the cells and breakdown of fatty acids stops. This reverses the anion gap and lowers the blood glucose levels, allowing the body to regain homeostasis. Insulin also causes potassium to go into the cells, which is why it is important for the potassium level to be checked prior to starting insulin. Depending upon the level of hyponatremia and glucose, the starting fluids would typically be 0.45% NaCl or 0.9% NaCl at 250-500 mL/hour. Once the potassium level is checked and replaced as needed, an insulin drip will need to be started at 0.1 unit/kg/hour. We also typically check the patient's serum pH level to determine if bicarbonate is needed in the fluids. Note that those in DKA are typically bolused anywhere from 2-4 liters of fluid prior to starting IV insulin. After the serum glucose is at a reasonable level (protocols differ between hospitals, but 200 mg/dL is a relatively standard cutoff), the fluids should be changed to D5 0.45% NaCl and the insulin drip is weaned. Answer B: An insulin drip is part of the treatment algorithm for DKA, however, the IMMEDIATE treatment necessary for this patient is IV fluids. Answer C: Phosphate replacement should be considered if hypophosphatemia occurs which can precipitate arrhythmias or respiratory failure. Answer D: DKA is often precipitated by infection, however, we have limited history and findings at this time to indicate a source of infection. Answer E: A bolus of insulin can be given, but these patients will need to be on an insulin drip until their anion gap resolves. The initial treatment should always be IV fluids, BEFORE the insulin is given. Bottom Line: The first step in the treatment of DKA is infusion of isotonic saline to expand intravascular volume. This patient is showing signs of shock with tachycardia and hypotension. She needs the intravascular volume to be repleted in order to be stabilized. The IV fluids will lower the patient's blood glucose level, but she will still need insulin AFTER IV fluids have been started.

A 35-year-old male presents to the emergency department with a left hand wound after getting involved in a bar fight 3 days ago. The patient states his last tetanus shot was 15 years ago. Examination reveals a 1 cm laceration to the 3rd metacarpophalangeal joint when he punched someone in the mouth, and it is now red and swollen to the dorsum of the hand. Laboratory studies reveal a WBC of 21 x 103/mcL. What would be the most appropriate antibiotic in the treatment of this patient? A. ampicillin-sulbactam B. azithromycin C. gentamycin D.metronidazole E. nitrofurantoin

The correct answer is: A Treatment of cellulitis secondary to human bite wounds is targeted at treatment of both aerobic and anaerobic bacteria which colonize the oral mucosa. The most common aerobic bacteria includes Strep, Staph, and Eikenella species, while the most common anaerobic bacteria includes Prevotela and Fusobacterium species. Human saliva is known to have as many as 50 different species of bacteria, so antibiotic coverage needs to be broad and include both aerobic and anaerobic coverage. Most simple human bites can be managed with PO antibiotics, usually Amoxicillin/Clavulanate or Ciprofloxacin and Metronidazole in combination. When someone punches another person and cuts the skin overlying an MCP joint or damages the integrity of the MCP joint, there is significant risk for septic arthritis of that joint along with cellulitis. No matter how minor the wound may look, these patients need to be admitted for IV antibiotics and for orthopedic washout of the affected joint in the OR. Most antibiotics used to treat "fight bite" in the hospital include Ampicillin/Sulbactam, Pipercillin/Tazobactam, Cefoxitin, Ceftriaxone and Metronidazole, or Clindamycin and Ciprofloxacin. These antibiotic choices allow for broad coverage of both aerobic and anaerobic oral flora. Answer B: Azithromycin is not considered first line for treatment of cellulitis. It is usually used for treating ENT and pulmonary infections. It has coverage against Staph and Strep, however, it is not the best antibiotic for treatment of cellulitis. Answer C: Gentamicin offers gram negative coverage only and would not be appropriate in the treatment of cellulitis by itself. Answer D: Metronidazole offers good anaerobic coverage, but should not be used by itself for the treatment of cellulitis. It must be used in conjunction with either a 3rd generation cephalosporin or a fluoroquinolone. Answer E: Nitrofurantoin is usually used for urinary tract infections and is not useful in treatment of cellulitis or skin infections. Bottom Line: Ampicillin/Sulbactam offers good gram positive, gram negative, and anaerobic coverage for soft tissue infections and would be the antibiotic of choice.

A healthy 3-year-old male presents to the emergency room having swallowed a foreign object per the parents. He is currently asymptomatic. Ingestion of which of the following requires immediate consultation for endoscopy? A. battery in the esophagus B. closed safety pin in the intestines C. coin in the stomach D. lead in the small intestines E.small toy in the rectum

The correct answer is: A Young children often put anything and everything in their mouths and have a tendency to swallow or aspirate small items. Items that are aspirated can cause serious side effects, while typically swallowed items will pass through the GI system without any symptoms. However, there swallowed batteries are one of the exceptions to this rule. Batteries are particularly concerning as some of the internal acid may cause erosions in the esophagus, stomach, or intestinal lining. When a chest x-ray shows a battery within the esophagus it is considered standard treatment to move forward with retrieval as endoscopy can easily reach the object within the esophagus. If the battery is located in the small intestines, watchful waiting is an acceptable option as the battery may not be causing erosions and retrieval from this location would require surgery. Another swallowed foreign body that can have serious GI complications are multiple magnets. Answer B: A safety-pin in the intestines is of minimal concern, especially since it is closed. Risk vs reward must be calculated for every treatment option and this item has a high probability of passing through the rest of the intestines without causing any damage. Answer C: Coins are one of the common swallowed foreign bodies in the pediatric population. While very large coins can cause some obstructive issues in certain portions of the intestines, they usually pass without problems. Coins will sometimes lodge in the esophagus at narrow sections including the area adjacent to the aortic arch and the gastroesophageal (GE) junction. It is reasonable to retrieve a coin lodged within the esophagus and causing symptoms. In the above scenario, an asymptomatic swallowed coin that has already passed to the stomach should just be monitored. Answer D: Lead is another special situation in pediatric populations as prolonged exposure leads to increased absorption and can potential cause lead toxicity. If a known lead based foreign body is within the esophagus or stomach, retrieval is preferred. However, once the lead has passed through the stomach and entered the intestines, the case becomes surgical and the child should be monitored for symptoms before opting for surgical retrieval. Answer E: Most objects that have made it to the rectum will almost certainly pass without issue. However, parents should monitor the child's stool to confirm passage of the foreign body. Additional consideration may be given to retrieval if the foreign body was inserted rectally and appears lodged or is causing obstruction, as can be seen in adults with psychiatric issues. Bottom Line: Batteries located within the esophagus or stomach should be immediately retrieved due to risk of erosive esophagitis/gastritis.

A 45-year-old male presents to the emergency department for evaluation of severe left eye pain and photophobia after he was hit in the eye by a broom stick. Examination reveals ciliary flush and miosis. The most likely diagnosis is A. acute angle closure glaucoma B. acute iritis C. conjunctivitis D. corneal abrasion E.hyphema

The correct answer is: B Acute iritis or Anterior uveitis is an inflammation of the iris. In this scenario, it is likely secondary to trauma, but there are many other causes. The most common presenting symptoms are eye pain and photophobia. Exam will reveal ciliary flush which is a redness/irritation around the iris itself as well as miosis. The eye is trying to constrict the iris to decrease the amount of light entering the eye because of the photophobia. Treatment includes cycloplegics and steroid eye drops. Answer A: Acute angle closure glaucoma usually presents with decreased vision, headache, eye pain, nausea/vomiting, and halos around lights. Physical exam reveals a MID DILATED pupil that reacts poorly to light, a shallow anterior chamber on slit lamp exam, conjunctival injection, and corneal edema/cloudiness. Intraocular pressure will be elevated. Treatment includes acetazolamide, timolol, apraclonidine, pilocarpine, and steroid eye drops depending on the severity of increased intraocular pressure. Answer C: Conjunctivitis is an inflammation of the conjunctiva. Patient's usually present with conjunctival injection, but usually do not have pain. Bacterial conjunctivitis usually presents with thick yellow discharge whereas viral or allergic conjunctivitis is usually watery. Antibiotic eye drops should be given if bacterial conjunctivitis is suspected. Answer D: Corneal abrasions are caused by trauma to the cornea. They are well visualized with fluorescein staining. Antibiotic eye drops are usually given to prevent infection. Make sure you cover for pseudomonas in patients who wear contacts. Answer E: A hyphema is blood in the anterior chamber. Most commonly it is due to trauma. Treatment is mostly supportive, with patient needing to sleep upright at night so that the blood settles at the bottom of the anterior chamber and does not obstruct vision. Sometimes surgery is necessary however to prevent vision loss. Bottom Line: "Ciliary flush and miosis" is iritis until proven otherwise.

A 16-year-old male presents to the emergency department after being bitten by his neighbor's dog six hours ago. Physical exam reveals a 3 cm left hand laceration into the superficial adipose tissue. The most appropriate management is: A. Irrigation and azithromycin B. Irrigation, amoxicillin-clavulanate and secondary closure C. Irrigation, suture repair and azithromycin D. Irrigation, suture repair and topical mupirocin E.Surgical wound debridement and repair

The correct answer is: B All animal bites should be irrigated. There is controversy as to whether animal bite lacerations should be sutured. Classically, wounds that are at high risk for infection (such as those of the hand, as with this patient) are not sutured. Furthermore, Answer B is the only choice in which all treatments presented are appropriate. Prophylactic antibiotics are not indicated for routine dog bite wounds except those of the hand. In bites to the hand and/or those that are infected, coverage for P. multocida is recommended with amoxicillin-clavulanate or the second or third generation cephalosporins. Rabies and tetanus prophylaxis should also be considered for all bite wounds. All dog bite wounds can be closed after appropriate cleansing with the exception of the following: bite wounds to the hands and lower extremities, with a delay in presentation (>8-12 hours old), or in immunocompromised hosts. These generally should be left open or treated by delayed primary closure. Answer A: Azithromycin is generally not used for the prophylactic treatment of dog bites. Coverage for P. multocida with amoxicillin-clavulanate and a second or third generation cephalosporin is indicated. Answer C: Suturing animal bite wounds is controversial and usually done for cosmesis. In patients with bites to the hand or those at high risk for intention, the wound is usually left open and allowed to heal by secondary intention. Furthermore, azithromycin is not typically used for prophylactic treatment of dog bite wounds. Answer D: Suturing animal bite wounds is controversial and usually done for cosmesis. In patients with bites to the hand or who are at high risk for intention, the wound is usually left open and allowed to heal by secondary intention. Furthermore, topical mupirocin is not indicated and antibiotic coverage for P. multocida should be with amoxicillin-clavulanate or a second or third generation cephalosporin. Answer E: There is no evidence of necrotic tissue that would require surgical debridement. This wound can be managed in the emergency department. Bottom Line: Animal bite wounds to the hand or that are at risk for infection are generally left open to heal by secondary intention. In such patients, coverage for P. multocida is recommended with amoxicillin-clavulanate or a second or third generation cephalosporin. For patients with otherwise clean wounds that aren't to the hands, antibiotics are not indicated.

A 32-year-old male presents to the emergency department after sustaining a laceration two days ago while preparing dinner. The patient states that he accidentally cut himself with his knife, which was clean and unused. He thought the bleeding was well controlled but woke up this morning with blood-soaked bandages. Past medical history reveals that he received a TDaP vaccination approximately 5 years ago. Physical exam reveals a 6 cm laceration to the left forearm that extends through the dermal layers. Wound edges appear to be linear with no maceration. The wound continues to slowly ooze sanguineous drainage. Which of the following is the most appropriate next step in management? A. apply silver-impregnated bandages B. irrigation with normal saline C. psychiatric consultation D.simple suture closure E. Td booster

The correct answer is: B As this wound occurred more than 6 hours ago (the evening prior) this wound cannot be primarily closed. Primary closure must happen within 6 hours of sustaining the wound. As this patient presents the next day after sustaining his injury, primary closure with sutures, glue or skin tape would not be indicated due to high risk of infection. To further assist in infection prevention the patient must undergo thorough irrigation to remove any bacteria and potential foreign debris. While there is no set amount of irrigation, lacerations need approximately 60 mL per centimeter of laceration at a high pressure. This can easily be attained by using a large syringe with an IV catheter at the tip to create the amount of pressure necessary to properly flush the wound. As this patient will undergo a delayed closure they will need to followup with either their primary care physician or return to the ER to have the wound closed. After 48 hours the risk of infection decreases to nearly zero and as such the wound can be closed by delayed primary closure. Delayed primary closure means that after a certain period of time (usually 2-4 days) the wound is closed by traditional methods after the risk of infection has gone. Secondary closure means to allow the wound to slowly close on its own over time without medical intervention. This is uncomfortable for patients and can be very stressful to have an open wound for that great period of time. Answer A: Patients with acute lacerations and wounds would not need bandages impregnated with silver alginate. This is a technique commonly employed by wound care physicians who are working with patients whom have chronic non-healing wounds. The silver alginate helps to prohibit bacterial growth by the silver ion being released into the wound and prohibiting bacterial growth. The silver ions are inert and in the concentrations found in these bandages would in no way would cause harm to the patient by being absorbed if the bandages are used correctly. Answer C: Although the wound described in somewhat unusual for an accidental laceration in the kitchen, the wound should be managed first. The patient should be evaluated by obtaining additional history before ordering a psychiatry consultation such as signs of depression, suicidal ideation, and recent life events. Answer D: Immediate closure with simple sutures is an inappropriate response. All wounds must be irrigated and explored prior to closure to rule out any foreign bodies and contaminants. In addition, given the time frame of the patient's presentation, a primary closure would be contraindicated due to the high risk of infection. Lacerations sustained on the face or scalp are in highly vascular areas and as such can undergo primary closure up to 24 hours out from when the wound was sustained. Lacerations elsewhere may only be closed if presented within 6 hours of the wound occurring. Answer E: Administering the Td booster in this patient would not be necessary. He has stated that his last vaccination was 5 years ago. The Td booster is indicated in adults who have either a dirty, contaminated wound or have gone for more than 10 years since their last vaccination or booster shot. As this patient has a clean, non-contaminated wound and has had a booster within the last 10 years, a tetanus shot is not indicated for this patient. If the patient's wound had been a dirty wound, he would need a booster as the limit on this is a booster within the last 5 years. For patients who are simply uncertain of when their last vaccination was, it is always good practice to administer the booster. Bottom Line: Lacerations that have presented greater than 6 hours from the time of incident may not undergo

A 22-year-old male presents to the emergency department after found wandering in the street by a good samaritan who felt he was rather agitated. Vitals reveal a heart rate of 130/min, respiratory rate of 30/min, blood pressure of 180/95 mmHg, and temperature of 38.8ºC (102ºF). Examination reveals dilated pupils, coughing with black sputum, and reactive mydriasis. The most likely causative agent is A. alcohol B. cocaine C. marijuana D.opiate use E. phencyclidine

The correct answer is: B Cocaine use causes a sympathomimetic toxidrome which causes symptoms of tachycardia, tachypnea, mydriasis, hyperthermia, agitation, altered mentation, and seizures. Patients who snort cocaine can develop nasal septum perforation over time. Up to 40% of patients may develop (if smoked) black sputum production and hypersensitivity pneumonitis known as "crack lung." Treatment is usually symptomatic with IV fluids, antipyretics, and benzodiazepines as needed for agitation. Always check an EKG as cocaine can cause coronary vasoconstriction and enhanced thrombus formation leading to MI. Answer A: Alcohol intoxication is usually obvious because the patient will have alcohol on his breath, have slurred speech, and impaired coordination. Answer C: Marijuana use is usually characterized by increased appetite and conjunctival injection. Answer D: Opiate overdose/use usually presents with CNS and respiratory depression as well as pupillary constriction. Answer E: Symptoms of PCP toxicity include altered mental status (from comatose to profound psychomotor agitation), hallucinations, hypertension, tachycardia, and multi-directional nystagmus. Bottom Line: Cocaine use causes a sympathomimetic toxidrome and is consistent with the patient's symptoms of tachycardia, hypertension, tachypnea, hyperthermia, and mydriasis. The nasal perforation should be a clue that the patient has been snorting cocaine, which can cause necrosis and eventual perforation of the nasal septum.

A 20-year-old male presents with diarrhea that started 24 hours ago after starting a new diet rich with raw vegetables and salads. History reveals frequent large, bloody bowel movements associated with abdominal cramping. Stool studies reveal increased fecal leukocytes. Question 1 of 2 in this set The most likely cause is A. C. difficile B. E. coli O157:H7 C. S. aureus D. Yersinia E.Norwalk

The correct answer is: B E. coli O157:H7 infection causes severe, acute hemorrhagic diarrhea and abdominal cramping. Duration of illness can be five to ten days. Inciting agent is usually undercooked beef (hamburger), unpasteurized milk, juices, raw fruits and vegetables. Treatment is supportive. Antibiotics should be avoided due to risk of developing hemolytic uremic syndrome. Answer A: C. difficile is suggested by the administration of antibiotics or within 2 weeks of their discontinuation. Broad-spectrum antibiotics such as clindamycin, cephalosporins, ampicillin/amoxicillin, and fluoroquinolones have been commonly implicated while almost any antibiotic and even chemotherapeutic agents, proton pump inhibitors, and antiviral agents can cause pseudomembranous colitis. C. difficile is characterized by profuse, diarrhea, abdominal pain, fever, leukocytosis, dehydration, and hypovolemia. Answer C: Staphylococcus aureus is associated with sudden onset (within one to eight hours) severe nausea, vomiting, diarrhea, and fever. Duration of illness is one to two days. Inciting agents are refrigerated meats, potato or egg salad, or pastries. Treatment is supportive. Answer D: Yersinia is associated with pseudoappendicitis, fever, abdominal pain, vomiting, diarrhea and rash. Duration of illness is one to three weeks. Inciting agents are undercooked pork products, tofu, contaminated water. Treatment is supportive care and antibiotics are not usually required. Answer E: Norwalk virus is commonly associated with contaminated drinking water. Classically, epidemics caused by the Norwalk virus occur on cruise ships. Norwalk virus causes a self-limited gastroenteritis with nausea, vomiting, and non-bloody diarrhea. Bottom Line: E. coli O157:H7 is acquired through consumption of undercooked beef, unpasteurized milk, juices, raw fruits and vegetables and causes severe, acute hemorrhagic diarrhea and abdominal cramping. This is the parasympathetic viscerosomatic reflex for several body regions: descending colon, rectum, distal ureter, bladder, urethra. Treatment will alleviate parasympathetic overdrive resulting in alleviation of diarrhea.

A 60-year-old male presents to the emergency department with darkened stools. History reveals microcytic anemia that has been treated with iron supplementation. Physical examination reveals a comfortable patient in no acute distress with a cardiopulmonary and vascular physical exam. A complete blood count is obtained revealing a hemoglobin and hematocrit of 12.8 g/dL and 34 %, respectfully and a MCV of 70 fL. A digital rectal examination is performed and reveals normal sphincter tone, no internal hemorrhoids, no stool in the vault, a normal prostate, and no blood on the gloved figure. You note a tender nodule on the right mid-thigh, just anterior to the iliotibial band. A fecal occult blood test is obtained and is read as positive. Which of the following can cause a false positive result? A. amoxicillin B. beets C. ferrous sulfate D.poppy seeds E. systemic lupus erythematosus

The correct answer is: B Fecal blood is tested in the setting of a complaint or physical exam finding concerning for blood loss through the gastrointestinal tract. It is also a useful tool for screening for colon cancer. The common point of care testing is the guaiac test (gFOBT) which tests for the presence of heme, whether as a free protein or as still attached to hemoglobin. Heme has active pseudoperoxidase activity. In the presence of the developer which is hydrogen peroxide, guaiac acid is oxidized providing a blue indicating hue. False positives are possible with this test if there is an elevated concentration of heme from a recent diet of red meat, or if there is an elevated peroxidase which can be found in some fruits and vegetables, including beets. This question also provides you with a tender thigh nodule which may represent a chapman reflex point from the ascending colon. Although there can be false positives based on diet, do not ignore other signs of potential colon cancer. Answer A: It has been a common in the lay literature that the antibiotic amoxicillin can be the cause of a false positive on urine drug screens for cocaine. However, this has been proven false by a obtaining urine drug tests from thirty three patients receiving amoxicillin treatment. There have been no documented cases of amoxicillin and false positive rates of gFOBT. Answer C: This patient presents with a microcytic anemia treated with iron supplementation. Although diet can produce false positives on gFOBT, iron supplementation does not as the gFOBT is a test of heme and peroxidase activity and is not effected by iron. The presence of iron, however, can darken the stool leading to a melena-appearing stool that tests negative on gFOBT. Answer D: Poppy seed is derived from an opium plant. During processing of the seed for consumption, morphine can adversely contaminate the seed and, as a result, lead to a false urine drug screen of morphine and opioid use. This has significantly decreased over the past few years with a report of 90% resolution of contaminate making poppy seed consumption as an excuse for opioid positive screening less valid. Answer E: Syphilis testing is done initially by the non-Treponemal testing for screening purposes, which would be followed by a Treponemal test if found positive on screening to rule out other causes of a positive test. Because the non-Treponemal tests looks for humoral immune response to infection, a false positive RPR can be caused by autoimmune diseases such as systemic lupus erythematosus (SLE). SLE has not been correlated to fecal occult blood tests. Bottom Line: Because of cost and availability, gFOBT is widely used for point of care testing and colon cancer screening. Because of the mechanism of action of this test, it is possible to have a false positive from diet. Knowing and considering this idea could prevent unnecessary treatments.

A 46-year-old female presents to the emergency department with a chief complaint of shortness of breath of one hours' duration. She stated that it started suddenly and was associated right upper chest pain. History reveals that she just got back from a long flight yesterday. Review of her past medical history is positive for hypertension, oral contraceptive pills, and a 20 pack-year smoking history. Vitals reveal a blood pressure of 126/77 mmHg with an oxygen saturation of 98% on 2L nasal cannula. A contrast-enhanced CT scan of the chest is obtained and shows multiple small segmental pulmonary emboli in the right lung. The most appropriate medication to administer would be A. aspirin B. clopidogrel C. enoxaparin D.tPA E. warfarin

The correct answer is: C A pulmonary embolism is a thrombus formed in the venous system that mobilizes to the pulmonary arteries which leads to acute obstruction, ischemia/infarction of the lung parenchyma. Large emboli can cause obstruction of right ventricular outflow and cardiovascular collapse. The patient in this question is diagnosed with a pulmonary embolism via the test of choice, CT angiography. Her vital signs are stable and she had a small segmental pulmonary emboli rather than a large saddle embolism. Treatment includes immediate anticoagulation with either unfractionated heparin or low molecular weight heparin such as enoxaparin. These agents act to bind to antithrombin III and accelerate its activity, thus inhibiting thrombin and factory Xa in the coagulation cascade. This prevents further clot formation and the body will eventually dissolve the clot. There are extensively studied prediction rules for pulmonary embolism such as the Wells score and the PERC rule. The Wells score includes: 1) clinically suspected DVT (3 pts); 2) alternative diagnosis is less likely than pulmonary embolism (3 pts); 3) tachycardia greater than 100 bpm (1.5 pts); 4) immobilization for greater than three days or surgery in the previous four weeks (1.5 pts); 5) history of DVT or PE (1.5 pts); 6) hemoptysis (1 pt); and 7) malignancy with treatment within six months or palliative (1 pt). A score greater than six indicates a high probability of pulmonary embolism. A score between two and six indicates a moderate probability of pulmonary embolism. And a score of less than two indicates a low probability of pulmonary embolism. The typical recommendations include performing a D-dimer with a low to moderate score. If the D dimer is negative then a pulmonary embolism is highly unlikely. If the D dimer is positive proceed to CT angiography. If the patient has a score of greater than six with Wells criteria proceed directly to CT angiography. Another useful tool for evaluating patients for pulmonary embolism is the PERC rule (mnemonic for pulmonary embolism rule out criteria). If all of the criteria are negative, this effectively rules out pulmonary embolism with a pretest probability less than 15%. If any of the criteria are positive one must consider a PE and order a D-dimer test or if clinical suspicion is high enough a CT pulmonary angiogram. Note that a negative D-dimer effectively rules out pulmonary embolism. The PERC criteria include: age greater than 50, heart rate greater than or equal to 100, oxygen saturation of room air less than 95%, prior history of DVT/PE, recent trauma or surgery, hemoptysis, exogenous estrogen, or unilateral leg swelling. Utilizing the Charlotte diagnostic algorithm for PE if one has a clinical Gestalt of less than 15% and the has one positive on the PERC rule then they should order a quantitative D dimer test. If the quantitative D dimer test is positive they should order imaging. If clinical Gestalt is moderate (15-40%), one should bypass the PERC criteria and order a quantitative D dimer. Lastly, if the clinical Gestalt is high (>40%), one should order a CT pulmonary angiogram. Answer A: Aspirin as a non-selective cyclooxygenase inhibitor. It leads to inhibition of formation of thromboxane A2 antiprostaglandins which leads to reduction of platelet aggregation. It is not indicated in the treatment of pulmonary embolism. Its use is indicated an acute coronary syndrome, pain/fever, MI prevention, TIA/thromboembolic stroke prevention, arthritis, and rheumatic fever. Answer B: Clopidogrel is not indicated in the treatment of pulmonary embolism because it does not prevent further clot formation. Clopidogrel binds to ADP receptors reducing platelet activation and aggregation. It is indicated in acute coronary syndrome, and thrombotic event prevention. Answer D: A massive PE can lead to cardiac arrest, hypotension, and right heart strain. If these signs are evident, thrombolysis is indicated with TPA being the preferred agent. TPA acts by binding to fibrin and converts tissue plasminogen to plasmin, promoting fibrinolysis. There is no evidence that mortality is lowered, however this will improve right heart function. Contraindications to thrombolysis include suspected aortic dissection or pericarditis, active internal bleeding, known intracranial neoplasm, ischemic CVA in past year or previous hemorrhagic stroke at anytime. Answer E: Warfarin is drug that inhibits factors two, seven, nine, ten, protein C and S in the coagulation cascade. It leads prevents further clot formation. Warfarin cannot be used as the sole anticoagulant because initially during its early use it can lead to a hypercoagulable state. It must therefore be used in conjunction with a heparin agent. Therapy is overlapped with a heparin agent until the INR (international normalized ratio) is between 2-3. Bottom Line: Heparin/enoxaparin is indicated in the treatment of a stable patient with a pulmonary embolism.

A 3-year-old female presents to the emergency department with a fever and vomiting. History reveals that the patient has had 3 days of runny nose, dry cough, and fussiness. Vitals reveal a weight of 15 kg (33 lb), temperature of 38.4ºC (101.2ºF), heart rate of 125/min, respiratory rate of 24/min, and pulse oxygenation of 99% on room air. Physical examination reveals a well-appearing child who is interactive with her parents and the practitioner. There are no lesions or rashes on skin examination. Otoscopic examination reveals the left tympanic membrane to be injected and dull appearing. Later, the patient is observed to have tolerated oral liquids and solids and is subsequently discharged with outpatient antibiotics and antipyretics. Which of the following is the most appropriate dosage of acetaminophen for this patient? A. 112.5 mg B. 225.0 mg C.300.0 mg D. 375.0 mg E. 75.0 mg

The correct answer is: B The appropriate acetaminophen dosage for children is 10-15 mg/kg. Since this patient weighs 15 kilograms, the prescription should be for 225 mg (15 x 15 = 225 mg) or 150 mg (10 x 15 = 150 mg). Since 225.0mg is the only credible answer choice listed, this should be the correct answer. The appropriate frequency of dosing is every 6 hours not to exceed a maximum daily dose of 75 mg/kg up to 4 grams daily (the maximum daily dosage for a child of 15 kg would be 1,125 mg in this example). When weight is unknown the following guidelines for this age group can be used: Age 2-3 years (weight, 24-35 lbs [10.9-16.3 kg]) Dose: 160 mg/dose Oral suspension (160 mg/5 mL): 5 mL Chewable tablets (80 mg/tab): 2 tablets Chewable tablets (160 mg/tab): 1 tablet Another useful pain reliever/fever reducer is ibuprofen, but it can only be used in children greater than 6 months of age. The dosing of ibuprofen is 10 mg/kg every 6 hours not to exceed a daily dosage of 40 mg/kg (up to 3200 mg).24-35 lbs (10.9-16.3 kg) gets 160mg per dose.

A 20-year-old female presents with recurrent, pulsating, unilateral headaches that last for four to six hours, are moderate in intensity, and are accompanied by nausea or photophobia. She reports the symptoms are exacerbated by physical activity. The most likely diagnosis is A. cluster headache B. migraine headache C. rebound headache D. sinus headache E.tension headache

The correct answer is: B The classic descriptors for a migraine headache includes unilateral, pulsating, nausea/vomiting, sensitivity to light and sound, with preceding visual or sensory aura. No specific etiology is known to cause migraines but it commonly runs in families and usually corresponds with hormonal changes such as puberty and menopause. Migraines commonly have a first onset in adolescent to young adult females. The age and description in this case make migraine the best option. Answer A: A cluster headache is a type of trigeminal autonomic cephalagia and is commonly described as excruciatingly severe ipsilateral pain around or behind the eye that lasts from 30 minutes to 3 hours with a rapid onset. Occasionally associated symptoms such as ptosis and mitosis occur. Treatment involves just giving oxygen by mask which usually helps reduce the duration of symptoms to < 30 minutes. As in migraines, triptans can also be used with calcium channel blockers for long-term prevention. Answer C: A rebound headache is due to overuse of pain relief medications, typically taken to relieve headaches in the first place. This phenomenon occurs commonly with overuse of triptans and opioids. The treatment is to cease or taper the offending agent without causing overt withdrawal symptoms. Answer D: Sinus headaches occur from acute or chronic sinusitis causing pain and pressure in the cheeks and/or forehead. This headache is often frontal in location and is associated with classic sinus symptoms such as fatigue and congestion. As with most headaches, treatment consists of NSAIDS and Triptans for pain relief. This is often misdiagnosed as migraine headaches and a CT scan can confirm sinus involvement if needed. Answer E: Tension headache is the most common type of headache overall. It is typically described as bilateral squeezing pain, often originating from the posterior neck and migrating upwards to involve both sides of the head. These headaches can be induced by stress, lack of sleep, and eye strain and are typically though to be caused by chronic neck muscle tension, hence the name. Amitriptyline has been shown to be the most effective treatment for these type headaches. Bottom Line: Migraine headaches are often described as unilateral, pulsating pain with associated aura occurring in young females.

A 55-year-old male patient presents to the emergency department by EMS for acute altered mental status. The patient's wife is at bedside and informs you he has a history of poorly controlled hypertension. She states he was fine that morning, but started complaining of a sudden "bad headache" later that day, localized to the left side of his head. She denies any recent trauma. Approximately 1 hour prior to arrival, he became confused, disoriented, and incoherent which prompted her to call 911. Vital signs reveal a blood pressure of 190/70 mmHg and a respiratory rate of 8/min with periods of apnea. On physical exam, the patient has no evidence of trauma and only makes incoherent sounds with a sternal rub while reaching for the examiner's hand. Given the patient's history and presentation, what is the most likely additional finding expected for this patient? A. biconvex hyperdensity on CT of the brain B. bradycardia C.hypotension D. leukocytosis E. temporal bone fracture

The correct answer is: B The diagnosis in this patient is subarachnoid hemorrhage with cushing's triad. This patient presented with a known history of hypertension with an acute mental status change after a rapid onset headache without history of trauma. His glascow coma scale can be calculated as 8 (Eyes:1, Motor:5, Verbal:2). Given the low GCS and his history, this patient would most likely be emergently intubated and taken to CT for a scan of his brain, which would most likely show subarachnoid hemorrhage with increased intracranial pressure (compressed ventricles) and mass effect. The patient's vital signs on presentation demonstrate hypertension and a low respiratory rate with incidences of apnea which follow cushing's triad. The most likely additional finding in a patient with cushing's reflex would be bradycardia. As the intracranial hemorrhage increases producing a mass effect within a fixed environment (cranium), the intracranial pressure (ICP) increases. The calculation for this is cerebral perfusion pressure (CPP)= mean arterial pressure (MAP) - intracranial pressure (ICP). As the intracranial pressure increases due to the subarachnoid hemorrhage, the MAP has to increase in order to maintain the same CPP which is why patients will present with hypertension. As the blood pressure continues to elevate, carotid baroreceptors cause stimulation of the vagus nerve which induces bradycardia. This theory is still somewhat controversial and some think the actual compression of the brainstem is what actually causes cushing's triad. Either way, it is a sign of impending brain herniation and should be cause for concern if recognized early on. Answer A: A biconvex hyperdensity seen on CT of the brain is indicative of an epidural hematoma, which is unlikely in this patient because there is no history of trauma. Epidural hematomas do not form spontaneously. Given the patient's history of hypertension and sudden headache followed by altered mental status, subarachnoid hemorrhage is more likely. Answer B: The correct answer is bradycardia as it is the third vital sign in the cushing's triad. Answer C: A diastolic blood pressure ≥100 mm Hg or systolic blood pressure ≥160 mm Hg is associated with SAH. As blood pressure increases and there is increased vagal nerve stimulation by carotid baroreceptors, the tachycardia will progress to bradycardia which is what is actually noted in the cushing's triad. Answer D: Leukocytosis would be expected in a patient who presents with fever, neck stiffness, and headache, and should cause concern for possible meningitis. There is no indication of infectious process in the question stem, however, it should always be part of your thought process in a patient who presents with confusion and altered mental status. Answer E: Temporal bone fracture would be a finding suggestive of epidural hematoma secondary to traumatic rupture of the middle meningeal artery. There is no indication of possible trauma in this patient, although, an epidural hematoma can certainly cause increased intracranial pressure and cushing's triad if left untreated. Bottom Line: A history of poorly controlled hypertension with acute onset of headache followed by mental status changes makes subarachnoid hemorrhage a likely diagnosis. Cushing's reflex explains why this patient would present with hypertension, bradycardia, and irregular respirations.

A 28-year-old male presents to the emergency department after a motor vehicle accident. History reveals the patient was rear-ended from behind and sustained a whiplash injury, hitting the back of his head against the headrest. He smells of alcohol and is currently complaining of nausea and a headache. Question 1 of 2 in this set The most appropriate next step in management is A. apply cervical collar for immobilization B. assess airway, breathing, and circulation C. elicit history and perform a physical examination D.obtain head CT scan E. obtain plain film radiographs of the head

The correct answer is: B The first principle in the management of trauma patients is the primary survey. The mnemonic for the primary survey is ABCDE and stands for: A = airway, B = breathing, C = circulation, D = disability, and E = exposure. Therefore, in the management of trauma patients a physician must assess that the patient has a patent airway, bilateral breath sounds, pulses in all 4 extremities, bilateral equal pupils with equal response to light along with assessment of strength and sensation in all 4 extremities, and completely undressing the patient to expose any hidden injuries while making it a priority to keep them warm. After assessment of the Primary survey comes assessment of the secondary survey. The secondary is a detailed physical examination of the entire body from head to toe. Answer A: A spinal column injury should be suspected in any trauma victim, especially in the setting of motor vehicle collisions. However, the first step in any emergent setting is an assessment of a primary survey consisting of the ABC's. The patient should then be immediately placed into a cervical collar and undergo evaluation with both the NEXUS low-risk criteria and the Canadian C-spine rules. Answer C: Performing a history and physical examination is imperative in the evaluation of all patients. However, evaluation of trauma patients is different. According to the American college of surgeons and the advanced trauma life support course, performing the primary survey followed by the secondary survey is the order of evaluation. Also, according to advanced trauma life support during evaluation of the primary survey if the physician finds a problem, then they intervene immediately. For example, if a patient presents with an obstructed airway, then the physician is trained to secure the airway first before evaluating breathing air circulation in the primary survey. After completing the primary and secondary survey the physician can elicit a history if the patient is able to. Answer D: The patient in this scenario is likely to require a head CT due to the fact that he sustained a closed head injury with a high mechanism of injury. However, performing the primary and secondary survey and intervening as necessary is the initial step in the management of trauma patients. Answer E: In a trauma patient with a closed head injury, evaluation of the cranium by plain radiography is not the first test of choice. The first test of choice in evaluating a trauma patient with a closed head injury is a non-contrast CT scan of the head. A non-contrast CT scan of the head can evaluate whether or not the patient sustained intracranial hemorrhage, and is more sensitive for evaluation of a fracture of the cranial vault. Bottom Line: When evaluating trauma patients always assess the ABCDE's (primary survey) first. A CT scan of the head is obtained revealing an area of increased intensity in the frontal lobe. This is most likely due to The patient in this scenario suffered a closed head injury with the direct force originating in the posterior aspect of his head. The head CT has findings suggestive of an intraparenchymal hemorrhage in the frontal region of his brain. This is due to contrecoup forces on his brain. In a contrecoup injury an impact occurs on one side of the skull and then through rotational shear forces, the opposite side of the brain gets injured. Answer A: A concussion is a clinical diagnosis and does not have evidence of brain injury on CT scan. A concussion can be associated with transient loss of consciousness, headache, blurred vision, nausea, or amnesia. Symptoms can last months and the individual should refrain from participating in sports or activities that put them at risk for another closed head injury until they are cleared by their primary care physician. Answer C: The patient in this motor vehicle accident suffered a close injury with the direct force occurring at the posterior aspect of his head. Coup injuries are intracerebral contusions that occur by direct force with the transmission of energy travelling into the skull and into the underlying brain parenchymal. If the patient in this motor vehicle accident suffered a coup injury rather than a contrecoup injury, and he would have been intracerebral hemorrhage in the occipital region. Answer D: Shearing mechanisms lead to diffuse axonal injury (DAI), which is visualized both clinically and with imaging studies as multiple small lesions seen within white matter tracts. Those with DAI typically present with profound coma without elevated intracranial pressure. Answer E: A subarachnoid hemorrhage typically presents as a thunderclap headache (usually described as, "the worst headache of my life") that is sudden onset and maximum at onset. They are commonly due to ruptured aneurysms rather than trauma. Imaging will reveal blood localized to the basal cisterns, the sylvian fissure, or the intrahemispheric fissure indicates rupture of a saccular aneurysm. Blood lying over the convexities or within the superficial parenchyma of the brain is indicative of arteriovenous malformation. However, an intraparenchymal hemorrhage may occur with middle communicating artery and posterior communicating artery aneurysms. Bottom Line: A contrecoup injury occurs when a force occurs on the opposite side of the intracerebral contusion. This is caused by indirect rotational shear forces on the brain during a closed head injury.

A 38-year-old male presents to the emergency department with headache described as "the worst headache of my life." History reveals the headache started 30 minutes prior to arrival with a sudden onset while he was lifting weights. He denies associated aura symptomology and a history of headaches. Vital signs reveal a blood pressure of 190/110 mmHg, heart rate of 82/min, and respiratory rate of 16/min. The patient is presently in mild distress and complains of associated neck pain. The most appropriate next step in management is A. blood cultures B. computed tomography scan of the head C.electrocardiogram D. lumbar puncture E. magnetic resonance imaging of the hea

The correct answer is: B The patient in this scenario has findings suggestive of a subarachnoid hemorrhage. A subarachnoid hemorrhage typically presents as a thunderclap headache (usually described as, "the worst headache of my life") that is sudden onset and maximum at onset. The headache commonly develops during exertion such as lifting weights or during sexual intercourse. Other symptoms include: neck pain instead of headache, profuse vomiting, nausea, transient loss of consciousness, and/or buckling of the legs. Signs may include decreased level of consciousness, focal neurologic deficits, ataxia, seizures, nystagmus, nuchal rigidity, or cranial nerve deficits. 85% of non-traumatic subarachnoid hemorrhages are from ruptured cerebral aneurysms most commonly Berry's aneurysms in the circle of Willis. Other causes of a non-traumatic subarachnoid hemorrhage include arteriovenous malformation (AVM), tumor, or hypertensive intracerebral hemorrhage. Risk factors include: polycystic kidney disease, female gender, smoking, hypertension, and heavy drinking. A non-contrast CT scan of the head is the initial test of choice for diagnosis of subarachnoid hemorrhage and is 92-95% sensitive if performed within the first 24 hours of symptoms. Management includes urgent neurosurgery consultation and blood pressure control. Answer A: Blood cultures are indicated if the clinician has a suspicion that the patient has an infectious etiology such as an encephalitis or a meningitis. The patient in this scenario developed sudden onset headache which is atypical for encephalitis or meningitis which has a more gradual onset of headache. Findings suggestive of meningitis include nuchal rigidity, fever, petechial rash (if associated with meningococcal infection) and/or associated infections such as sinusitis, otitis, or pneumonia. Encephalitis is likely to have similar signs and symptoms such as impaired cognitive ability, altered level of consciousness, seizures, focal neurologic deficits, cranial nerve palsies, etc. Answer C: In a patient with symptoms suggestive of a subarachnoid intracranial hemorrhage, an electrocardiogram is not the initial test of choice as diagnosing a subarachnoid hemorrhage promptly is imperative. However, the majority of patients with subarachnoid hemorrhages have cardiac dysrhythmias and EKG patterns suggestive of ischemia or infarction. Examples of such patterns include T wave inversion, U waves, QT prolongation, and occasionally ST depression and/or elevation. Answer D: Non-contrast CT scan of the head is the initial test of choice and is 92 to 95% sensitive for subarachnoid hemorrhage if performed within the first 24 hours of symptom onset. Note that sensitivity goes down for CT scan of the head if performed after 24 hours of symptom onset. Therefore, if a CT scan is negative for subarachnoid hemorrhage, a clinician should perform a lumbar puncture to rule out subarachnoid hemorrhage. A positive result on lumbar puncture for subarachnoid hemorrhage is the presence of xanthochromia or the presence of erythrocytes in a non-traumatic tap. Xanthochromia is determined by spectrophotometry and usually takes between 6-12 hours to develop after the onset of the bleed. Note that xanthochromia is the yellow discoloration of the CSF after the breakdown of heme (from hemoglobin) into bilirubin. One can distinguish a traumatic tap from a subarachnoid hemorrhage by the clearing of CSF with subsequent tubes on the lumbar puncture. Answer E: MRI is a time consuming test with added cost (over CT), less availability, and is less sensitive for detecting subarachnoid hemorrhage. It does however, have the advantage of detecting intracranial aneurysms over non-contrast CT scanning.

An 84-year-old female presents to the emergency department from the nursing home for generalized weakness. Nursing staff were concerned because her urine smelled foul. Vital signs reveal a blood pressure of 105/60 mmHg, heart rate of 115/min, respiratory rate of 22/min, oxygen saturation of 98% on room air, and temperature of 39ºC. Physical examination reveals dry mucous membranes but is otherwise unremarkable. Urinalysis is positive for leukocyte esterase and nitrite with >100 WBC/hpf, 3+ bacteria, and 0-4 epithelial cells. A CBC and CMP is normal including lactic acid. On exam you note suprapubic tenderness and a tender nodule adjacent to the umbilicus. The most appropriate diagnosis is A. generalized weakness and positive SIRS criteria B. sepsis secondary to urinary tract infection C. septic shock D.severe sepsis secondary to urinary tract infection E. urinary tract infection

The correct answer is: B This patient's history and laboratory studies are indicative a urinary tract infection. Her vital signs meet SIRS criteria (two or more of the following). Thus, because we have a source of infection, she meets sepsis criteria as sepsis is defined as SIRS with the presence of a source of infection. Her sepsis is secondary to a UTI. She does not meet severe sepsis criteria because she does not show signs of end-organ failure on her CBC and CMP, nor lactic acid. SIRS criteria: - Temp >38°C (100.4°F) or < 36°C (96.8°F) - Heart Rate > 90 - Respiratory Rate > 20 or PaCO2 < 32 mm Hg - WBC > 12,000/mm3, < 4,000/mm3, or > 10% bands Sepsis: SIRS + a source or suspected source Severe sepsis: Sepsis + organ dysfunction - Sepsis-induced hypotension - Lactate above upper limits of laboratory normal - Urine output < 0.5 mL/kg/hr for >2 hours despite adequate fluid resuscitation - Acute lung injury with PaO2/FIO2 < 250 in the absence of pneumonia or < 200 in the presence of pneumonia - Creatinine >2 mg/dL - Bilirubin >4 mg/dL - Platelet count < 100,000/microL - Signs of coagulopathy (INR >1.5) Septic shock: sepsis-induced hypotension persisting despite adequate fluid resuscitation. With any bladder infection you may note a chapman reflex point surrounding the umbilicus. If the infection reaches the kidneys you may also note a tender nodule one inch lateral and superior from the umbilicus which is the anterior chapman point for the kidney. Answer A: This patient meets SIRS criteria and has a known source of infection (UTI), so she meets criteria for sepsis. The patient has otherwise normal lab work and no known organ dysfunction. Answer C: Sepsis-induced hypotension is defined as "the presence of a systolic blood pressure of less than 90 mm Hg or a reduction of more than 40 mm Hg from baseline in the absence of other causes of hypotension." Patients meet the criteria for septic shock if they have persistent hypotension and perfusion abnormalities despite adequate fluid resuscitation. Answer D: Severe sepsis implies some form of organ dysfunction secondary to an infection. This patient's CMP is normal (liver, kidney, electrolytes) as well as lactic acid. Answer E: This patient has more than just a UTI. She meets SIRS criteria AND has a source of infection, so she is septic secondary to UTI. Bottom Line: This patient's history and labs are indicative of a UTI, and she also meets SIRS criteria. Thus, source + SIRS criteria = Sepsis.

A 8-year-old presents with forearm pain after a fall. Plain film radiography reveals a non-angulated buckle fracture of the distal radius and ulna. Which of the following will have the best functional outcome? A. ACE wrap B. long arm cast C. removable splint D. surgical reduction and internal fixation E. thumb spica cast

The correct answer is: C Buckle, or torus fractures describe the bulging of the bony cortex or periosteum, usually involving the metaphysis, following compressive forces. The buckle fracture may be palpable as a ridge over the metaphyseal area of the long bone. It is often very subtle on radiographic imaging. Any asymmetry, bulging, or deviation of the cortical margin indicates a buckle fracture. Buckle fractures are not typically associated with severe angulation, displacement, or rotational abnormalities, so reduction is rarely necessary. Treatment is splinting in a position of function with orthopedic follow-up within one week. Simple torus fractures of the distal radius may need splinting, but orthopedic follow-up is not necessary. Answer A: Muscular strain is typically treated with compression in the form of an ACE wrap. A buckle fracture should be splinted. Answer B: A long arm cast is indicated for fractures of the distal humerus, proximal forearm, radial head and neck, olecranon or severe ligamentous injuries of the elbow. A long arm cast is not indicated for buckle fractures. Answer D: Buckle fractures are not typically associated with severe angulation, displacement, or rotational abnormalities, so reduction is rarely necessary. Answer E: A thumb spica splint is indicated in fractures of the scaphoid, lunate, first metacarpal or injuries to the ulnar collateral ligament. It is not indicated for buckle fractures. Bottom Line: Treatment of buckle fractures consist of splinting in a position of function and orthopedic follow-up.

A 36-year-old male presents to the emergency department with a stab wound to the chest. He is found to have a blood pressure of 75/40 mmHg and a heart rate of 110/min. Physical examinations reveals a 3 cm transversely oriented stab wound immediately left of the sternum in the 4th intercostal space, jugular vein distention 8 cm above the sternal angle, and muffled heart sounds. Immediate chest x-ray is performed and shown in the exhibit. Question 1 of 2 in this set Which of the following is most likely causing his hypotension? A. air trapping in the pleural space B. aortic rupture distal to left subclavian artery C. increased intrapericardial pressure D. reflex bradycardia E. systemic vascular dilation

The correct answer is: C Cardiac tamponade may occur after penetrating or, less commonly, blunt thoracic injuries. The pericardium is relatively resistant to distention. Consequently, as little as 100 mL of acute blood collecting in the pericardial space can significantly compress the heart and decrease venous return and cardiac output. Beck's triad of symptoms incudes hypotension, dilated neck veins and muffled heart sounds. Hypotension is caused by pericardial pressure exceeding right atrial pressure leading to impaired atrial filling. Right ventricular preload decreases as a result and CVP increases. Myocardial blood flow is also reduced which can lead to cardiac ischemia and continued decrease in cardiac output. Management is with emergent pericardiocentesis to remove blood from the space and temporarily restore hemodynamic stability. Most patients then require surgical intervention for diagnosis and repair of cardiac injuries and other bleeding sources. Answer A: Tension pneumothorax is a serious, life-threatening complication of penetrating thoracic trauma. Pulmonary or chest wall disruption creates a one-way valve causing air to be trapped in the pleural space. Progressive build-up of pressure in the pleural space pushes the mediastinum to the opposite hemithorax, and obstructs venous return to the heart. The classic signs of a tension pneumothorax are deviation of the trachea away from the side with the tension, a hyper-expanded chest, an increased percussion note and a hyper-expanded chest that moves little with respiration. This patient's chest x-ray does not show a tension pneumothorax. Answer B: Aortic disruption can be seen with rapid- deceleration injuries, most commonly high-speed frontal impact motor vehicle accidents and falls from heights greater than 10 feet. Disruption most commonly occurs just distal to the left subclavian artery where the aorta is attached by the ligamentum arteriosum. 85% of patients die at the scene due to rapid exsanguination. Patients who survive to the emergency department usually have small tears or partial-thickness tears of the aortic wall with pseudoaneurysm formation. Classic chest x-ray findings include widening of the mediastinum and loss of the aortic knob. Answer D: Typically those with low blood pressure secondary to blood loss will have elevated heart rates to compensate for the lower cardiac output. Hypotension would not be caused by bradycardia, however. Answer E: Trauma induces a sympathetic response which vasodilates the vasculature through release of nitric oxide. This, in combination with endocrine and other acute phase reactions, results in an increased extracellular fluid volume and hypovolemia. This is likely contributing to the patient's hypotension, however, it is not the most correct option available. Bottom Line: Cardiac tamponade results from fluid accumulation in the pericardial sac leading to increased intrapericardial pressure, compromising ventricular filling and decreased cardiac output. Classic physical examination findings include hypotension, distant heart sounds, and jugular venous distention. Focused Assessment with Sonography for Trauma (FAST) examination allows for rapid evaluation of trauma patients with real time images using ultrasonography. The presence of fluid in the pericardial sac would confirm the diagnosis of pericardial tamponade in this patient. Ultrasound guided pericardial drain placement may be done to improve the patient's hemodynamic status with removal of as little as 15 mL of blood. This temporizing measure can prevent myocardial ischemia for long enough to transport the patient to the operating room for definitive treatment. Answer A: A CT scan of the chest may reveal fluid within the pericardial sac, however, it is not usually necessary for evaluating one with cardiac tamponade if ultrasonography is available. Signs consistent with tamponade on a CT include pericardial effusion, distention of the venae cavae, distention of the hepatic veins, bowing of the interventricular septum, deformity and compression of the cardiac chambers, and reflux of contrast into the azygos vein and inferior vena cava. Answer B: Electrical alternans is pathognomonic for cardiac tamponade, however, it would not be confirmatory in diagnosis. Other signs consistent with cardiac tamponade include sinus tachycardia, low voltage QRS complexes, and PR segment depression. Answer D: Chest radiography may reveal cardiomegaly, however, it is never confirmatory in the diagnosis of cardiac tamponade. Other signs likely to be revealed in this patient include chest wall trauma with fluid accumulation elsewhere secondary to blood loss. Answer E: A paracentesis is using a needle to remove fluid from the abdominal cavity. A pericardiocentesis would be most appropriate here, but only after obtaining the appropriate imaging modality (ultrasound) and using this as a guide to complete the procedure. Bottom Line: FAST exam should be used to diagnose pericardial tamponade in a hemodynamically unstable trauma patient.

A 16-year-old male presents with epistaxis that started approximately 10 minutes prior to arrival and did not resolve with direct pressure. Examination reveals active bleeding from the anterior medial portion of the nasal septum. What is the most appropriate initial management at this time? A. balloon tamponade B. electric cauterization C. oxymetazoline D.Silver nitrate E. thrombogenic foam

The correct answer is: C Direct pressure is the best initial management of a nose bleed. When this does not resolve the bleed, oxymetazoline spray may be used, which is an alpha-adrenergic agonist that stimulates alpha-adrenergic receptors and produces vasoconstriction in the arterioles of the nasal mucosa. Most anterior nose bleeds occur at Kiesselbach's plexus and are easily controlled with direct pressure alone, followed by Oxymetazoline if direct pressure fails. A thorough inspection of the nose should be done, and cautery is the third step for continued bleeding. After cautery, nasal packing is considered with a variety of different products, including, gauze, balloon, or thrombogenic foam. Some patients with more severe bleeds or posterior bleeds should follow-up with an ENT within 2-3 days for reevaluation. A posterior nose bleed may need to be admitted for ENT evaluation and posterior packing with either a specialized balloon or a foley catheter. Per uptodate: "Many otolaryngologists recommend initial treatment with two sprays of Oxymetazoline (Afrin) to hasten hemostasis, although little published data exist to support this practice. One small retrospective study found that Oxymetazoline spray stopped bleeding in 65% of patients presenting to the emergency department with epistaxis". Answer A: Balloon tamponade is used if cautery does not work to control bleeding. Answer B: Electric cauterization or cauterization using silver nitrate can both be used if direct pressure does not control bleeding. Answer D: Silver nitrate is used to cauterize bleeding sites and should be used after direct pressure, and Oxymetazoline have failed to control the nosebleed. Answer E: Thrombogenic foam can be used to pack the nose and cause a clot to form, hopefully stopping the hemorrhage. This is usually done when cautery fails. Bottom Line: Direct pressure is the first line treatment for epistaxis, regardless of the severity. Second line typically involves spraying an alpha adrenergic agonist to vasoconstrict the blood vessels.

A 20-year-old female presents to the emergency department in active labor. History reveals that she is unemployed, uninsured, and has not received prenatal care for this pregnancy. Medical stabilization and an appropriate transfer of care is required based upon the A. Affordable Care Act B.Emergency Exception Doctrine C. Emergency Medical Treatment and Active Labor Act D. Healthcare Information Portability and Accountability Act E. the patient's signed informed consent

The correct answer is: C EMTALA is a law that was first enacted in 1986 that discouraged poor and high-risk patients from being transferred from one emergency department to another for financial benefit of an institution. It focuses on performing a medical screening exam and stabilizing patients. Under the act, any patient who presents to the emergency department requires a medical screening exam (MSE). A medical screening exam is exam performed usually by the physician of the emergency department to determine if the patient has an emergency medical condition (EMC) regardless of the patient's ability or willingness to pay for any services performed. An EMC is any condition in which a delay in treatment would cause loss of bodily functions or impairment to organs or limbs. Examples include a pregnant woman who is contracting, a patient with severe pain, psychic disturbances, and/or symptoms of substance abuse. If an EMC is found the emergency department is required to stabilize the condition. If the hospital is unable to stabilize the patient then they are required to transfer the patient to a facility that can. Transfers cannot take place when an emergency medical condition is not yet stabilized unless the patient or patient's representative requests transfer knowing the risks and the hospitals EMTALA obligations, the transferring physician feels that the risks outweigh the benefits, or the transfers medically appropriate. A hospital that makes an EMTALA violation may have to pay up to a $50,000 fine per violation. The emergency medicine physician and the on-call physician who negligently violate EMTALA may also have to pay up to $50,000 per violation. Answer A: The affordable care act (ACA) is a United States federal statute enacted by US president Barack Obama on March 23, 2010. The goal of the act was to increase quality and affordability of health insurance, lower the uninsured rate, and reduce the cost of healthcare for the government and individuals. It requires health insurance companies to cover all applicants regardless of pre-existing conditions or sex. There are reforms in the act which encourage quality over quantity through increased competition, incentives and regulation. Answer B: The emergency exception doctrine is a principal which dictates that emergency medicine physicians may treat patients without consent in certain situations. They may treat patients without consent only if all three of the following conditions are met: no family or other patient representative/decision-maker is immediately available for consent, the patient has an emergency condition which requires immediate intervention, and the patient is unable to express his or her wishes (i.e., the patient is comatose, delirious, demented, intoxicated, etc.). Answer D: The health insurance portability and accountability act was enacted in 1996. There are two parts of HIPAA. Title I protects health insurance coverage for workers and their families when they change or lose jobs. It also requires the coverage of and limits restrictions that a group health plan can place on benefits for pre-existing conditions. Title II of HIPAA defines guidelines, procedures and policies for maintaining the privacy and security of individuals and their identifiable health information or private health information (PHI). It also outlines numerous offenses relating to health care and sets civil and criminal penalties for violations. It was enacted to control fraud and abuse within the healthcare system. The purpose of the system is also to increase the efficiency of the healthcare system by creating standards for the use and dissemination of healthcare information. Of note, any release of personal healthcare information to another health provider (ex. obtaining old EKG's from a patient's primary care physician) must be accompanied by a signed release from the patient. Note the police do not have automatic access to the patient's medical record. Answer E: Informed consent is when a physician or other healthcare provider discusses with a patient or their representative a specific procedure along with the risks and benefits associated with it. Obtaining an informed consent should be considered: when hospital rules require it, when is the community standard, when the procedure carries significant risks, and when the procedure is extraordinary (ie not routine for most patients). In an informed consent the patient or their representative should know the nature of that procedure being performed, the expected outcome, the risks of the procedure, the risks of not performing the procedure (the natural history), and the alternatives to the procedure. The law requires a reasonable physician standard (what a reasonable physician would tell a patient) and the reasonable patient standard (what a reasonable patient would want to know before undergoing the procedure). When obtaining informed consent always obtain a signature of a witness and the patient or the patient's representative if possible and include the risks of: death, permanent neurologic impairment, permanent limp and organ impairment, all seriousness and severe possibilities, and temporary minor adverse results. Bottom Line: EMTALA requires that a medical screening examination be performed on all patients and requires that they be they be stabilized appropriately if an emergency medical condition is found prior to transfer. However, if their EMC is unable to be stabilized then the transferring physician must notify the patient/family of the risks and the risks of transfer to a facility capable of stabilizing the patient must outweigh the benefits.

A 67-year-old male presents to the emergency department with several episodes of hematemesis. History reveals the patient has had 5-6 episodes of blood-tinged emesis throughout the past day. Further history reveals he is a chronic alcoholic and has not seen a physician in years. His vital signs reveal a heart rate of 95/min, respirations of 18/min, and a blood pressure of 150/96 mmHg. Physical exam reveals a disheveled male with pink conjunctiva, capillary refill of 2 seconds, and abdominal obesity with distended veins around the umbilicus. You note a tender nodule just to the right of the sternum in the second intercostal space. A complete blood count is obtained revealing: WBC 8.5 x 103/mcL RBC 4.1 x 106/mcL Hemoglobin 10.9 g/dL Hematocrit 40% MCV 105.0 mcm3 MCHC 40 g/dL The most likely diagnosis is A. aorto-esophageal fistula B. Boerhaave syndrome C. esophageal varices D. Mallory-Weiss tear E. peptic ulcer disease

The correct answer is: C Esophageal varices are very common in patients who suffer from portal hypertension, secondary to chronic alcohol abuse. The triad of "Gut, butt and caput" are an easy way to remember the areas most affected by portal hypertension. The "gut" corresponds to esophageal and gastric antrum varices, the "butt" corresponds to hemorrhoids and the "caput" is in reference to caput medusae in which the venous drainage around the umbilicus becomes engorged as exhibited in this patient. The patient's alcoholism has resulted in alcoholic cirrhosis. This patient is exhibiting hematemesis that is only blood tinged which suggests slowly bleeding varices. Treatment for this in the emergency setting is to ensure the patient is stable and administer IV fluid hydration as well as octreotide, which helps decrease the bleeding by causing vasoconstriction which decreases the flow in the portal system and therefore decrease the bleeding. COMLEX will often provide you with viscerosomatic reflexes or chapmans points that can be helpful in making your diagnosis. In this situation you have a chapman point in the second intercostal space on the right which is classically described as the anterior point for the esophagus. Answer A: An aorto-esophageal fistula is a known complication of a patient who recently underwent an aortic graft, history should be carefully obtained to rule this out as a possibility. Classically they will have a "herald" episode of bleeding which precedes massive hemorrhage. Diagnostic diligence and immediate surgical referral is of utmost importance for these patients as they can deteriorate extremely rapidly. Fluids including blood transfusions should be started immediately if any hematemesis has begun. Answer B: Boerhaave syndrome is a complete esophageal rupture most often from extremely forceful emesis or a sudden rise in intraesophageal pressure. The presentation is sudden excruciating pain after forceful emesis. Hematemesis may or may not be seen initially. On physical exam you may note subcutaneous emphysema within the cervical region from escaped intestinal gas. The patient's complaints may also mimic a myocardial infarction so history and physical exam must be thorough. Treatment is ultimately surgical consultation as these patients can deteriorate very rapidly. Answer D: A Mallory-Weiss (MW) tear happens after several episodes of emesis without blood. The tear occurs longitudinally in the mucosa. Typically hematemesis and pain are not seen with a MW tear and will help distinguish it from a possible Boerhaave syndrome. Treatment depends on the overall clinical picture. If still vomiting blood or if unstable vital signs are present the patient should be admitted and have an esophagogastroduodenoscopy (EGD) performed and be placed on a proton pump inhibitor. If the bleeding has stopped and seems to be self limited and the patient is without significant comorbidities, they may be discharged to follow up with a GI specialist. Answer E: Peptic ulcer disease (PUD) is the most common cause of upper GI bleeding. It can be brought on by alcohol use, excessive or chronic NSAID usage and salicylate usage. These can be very slow and present with dark or tarry stools only or be somewhat more rapid in their bleeding and present with coffee ground emesis or bright red hematemesis. The dark tarry stools and coffee ground emesis both result from partial digestion and exposure of stomach acid to fresh blood. Pharmacologic treatment is with an H2 blocker like ranitidine or a proton pump inhibitor like omeprazole. Bottom Line: Hematemesis can have a wide variety of etiologies and the emergency physician must be discerning in the differential diagnosis.

A 32-year-old female presents with altered mental status and a blood glucose level of 45 mg/dL. She is provided 1 amp of D50 and the patient immediately became arousable with a Glasgow coma scale score of 14. History reveals her husband was unable to awaken her after a 5 hour nap and discovered a suicide note along with an empty bottle of glipizide. The most appropriate next step in management involves scheduled glucose monitoring and A. activated charcoal B. glucagon C. octreotide D.repeat 1 amp of D50 E. whole bowel irrigation

The correct answer is: C The patient presents with hypoglycemia secondary to a sulfonylurea overdose. The effects of sulfonylureas can last between 12-24 hours and can cause repeat episodes of hypoglycemia. This patient needs Octreotide as well as D50 given as needed for hypoglycemia. A dextrose drip alone is not recommended because elevation of the blood sugar will cause increased insulin release which would worsen the patient's symptoms. Octreotide, a somatostatin analog that inhibits insulin release from pancreatic beta-islet cells, is used in conjunction with dextrose at a dose between 50-150 mcg every 6 hours for the first 24 hours. If the patient develops repeat episodes of hypoglycemia during this time, the Octreotide should be continued for another 24 hours. Answer A: Activated charcoal should not be administered more than 2 hours after sulfonylurea overdose as it will not help at that point. Answer B: Glucagon (IM) raises serum glucose levels slightly and may be used as a temporizing measure while IV access is obtained, but it is not a substitute for dextrose. Answer D: There are no signs that the patient is severely hypoglycemic after the initial dose. This patient needs to be admitted for continued monitoring and given Octreotide with D50 given as needed for repeat hypoglycemia. Answer E: Whole bowel irrigation would not be useful for this patient, as she presents already 3 hours from possible ingestion time. Bottom Line: Octreotide is the treatment of choice in sulfonylurea overdose and continued monitoring is necessary due to the long-lasting effects of these medications and the potential for repeat episodes of hypoglycemia.

A 65-year-old male presents with rectal bleeding. History reveals smeared blood on the outside of hard stools. Patient denies abdominal pain, fever, nausea, or vomiting. A digital rectal examination reveals a large stool burden. What is the most likely cause of the patient's symptoms? A. Crohn's disease B. diverticulosis C. internal hemorrhoids D.irritable bowel syndrome E. ulcerative colitis

The correct answer is: C Hemorrhoids are more common as we age. Internal hemorrhoids are usually painless, however, external hemorrhoids are usually painful. Risk factors include constipation with long periods of sitting on the toilet straining to have a bowel movement as well as pregnancy. Treatment is usually conservative with hemorrhoid creams/steroids, but surgical ligation may be necessary in some cases. Answer A: Crohn's disease can cause blood in the stool, but it is usually mixed in with the stool because the blood is coming from within the intestines and is usually associated with abdominal discomfort. There is usually a family history. Answer B: Diverticulosis can cause isolated rectal bleeding without other symptoms, however, the blood would be mixed in with the stool rather than only be on the outside of the stool. Definition of acute diverticulitis is inflammation and/or infection of a diverticulum. It is associated with left lower quadrant abdominal pain, low-grade fever, and nausea, vomiting, constipation, or diarrhea. Answer D: IBS is usually associated with intermittent episodes of abdominal cramping, bloating, and diarrhea. Answer E: Ulcerative colitis usually affects the rectum. It is associated with abdominal discomfort and can have blood mixed in with the stool. There may be a family history. Bottom Line: A history of constipation and painless rectal bleeding with blood noted on the OUTSIDE of the stool is indicative of internal hemorrhoids.

A 7-year-old male presents to the emergency department with blunt force trauma after being involved in a motor vehicle accident. History reveals the child was restrained sitting directly behind the driver of the vehicle, which was struck on the driver's side. The child was with a friend's family member and is accompanied by the friend's parents in the emergency department. Multiple unsuccessful attempts are made to contact the child's parents. His vital signs are found to be within normal limits. Physical exam reveals a gross deformity of the left upper extremity with intact distal pulses and sensation. Plain radiograph of the left upper extremity reveals a midshaft left humeral fracture. The most appropriate step in management at this time is to A. delay care and transfer to a pediatric tertiary facility B. obtain consent from a second physician who will co-sign on all therapy C. obtain informed consent from the adult present D.obtain informed consent from the child E. stabilize the patient and withhold further therapy until the parents can be contacted

The correct answer is: C In this situation a child has presented without a parent or legal guardian capable of providing consent to treat. There are laws and recommendations in place for what to do in this situation. The Emergency Medical Treatment And Labor Act is a federal law stating that any patient that presents to the emergency department regardless of ability to pay, age, or any other discriminating factor is entitled to a Medical Screening Exam (MSE) and treatment for any Emergency Medical Conditions (EMC) identified on the exam. There is also an official recommendation from the American Academy of Pediatrics(AAP) regarding these laws as they apply to children and therefore outlining treatment protocols and how to handle consent issues. These laws state that after appropriate attempts have been made to contact the parent an adult who presents with the child may give consent for treatment. It goes on further to state that if the child is old enough to understand the medical condition and treatment plan they may give consent themselves. They recommend assessing the child's level of understanding and recommend those over the age of 14 as being able to give consent. The emergency physician must perform a MSE upon all patients presenting to the ER and use their best clinical judgement for treatment options necessary for stabilization of and EMC's. Any further treatments consent must be obtained from a parent, legal guardian, adult acting on behalf of the unreachable parent or the child themselves if capable of understanding the medical condition and treatment plan. For this specific fracture, the child's arm should be reduced and placed in a coaptation splint (U-shaped) for two weeks. Surgical repair is indicated in those with open fractures or those with closed fractures when displacement or angulation cannot be otherwise managed. Answer A: Refusal or delay of care with a transfer to a different hospital would be a blatant violation of the laws set forth by EMTALA. If the hospital did not have the capabilities of handling the patients diagnosis, or per patient request, are the only acceptable reasons to transfer a patient to a different facility. If the hospital the patient presents at has the capability of caring for them they must provide the care regardless of extenuating circumstances such as ability to pay, race, creed or religious beliefs. Answer B: Some hospitals may have policies of this nature to determine consent for patients who have no Medical Power of Attorney (MPOA) or next of kin capable of making decisions in their stead. These policies are in place to prevent physicians from performing unreasonable and unnecessary procedures. Consenting with a second physician may prevent a frivolous procedure that is unnecessary. Answer D: In this instance, as the child is only 7-years-old he is not capable of fully understanding everything the doctor would be telling him and as such would not be able to provide informed consent. If the child had presented alone the physician would be protected in treating and stabilizing the child if acting without consent. According the the American Academy of Pediatrics Statement on Consent "Physicians would do well medically and legally to treat the patient as though it is someone you love and care about." This means the physician would be protected in stabilizing and treating any child presenting to the emergency department without a parent or guardian to provide consent. Answer E: This child has an obvious deformity of the humeral shaft and is likely in a great deal of pain. While his pulses and sensation are intact distally it would be inhumane to withhold proper care of the child until his parents can be reached. Laws like EMTALA and the AAP's policy statement on consent exist for situations like this to allow the physician to treat the child without fear of legal ramifications. Also it is suggested that individual institutions develop clear policy statements regarding the care of minors without parental consent. Bottom Line: Consent for a minor can be tricky, knowledge of the laws and individual institution policies is a must for any emergency physician.

A 20-year-old college student driving under the influence of alcohol has a front-on collision with a telephone pole at a speed of 45 mph. The car air bags did not deploy. She experienced a short loss of consciousness, but is awake and answering questions appropriately in the emergency department. She reports to have a headache, chest pain, and right ankle pain. Her neck was stabilized by the paramedics on the scene. Vitals reveal a blood pressure of 101/72 mmHg, pulse of 104/min, respiratory rate of 22/min, and Glasgow coma score of 15. Her pulse oximetry is reading 89% while receiving oxygen at 2 L/min via nasal cannula. Physical examination reveals equal sized pupils that are reactive to light, clear lung fields bilaterally, tenderness to palpation of the chest wall and upper abdomen, and protrusion of the right tibial bone immediately superior to the ankle joint. Neurological examination is normal. Which of the following is the most appropriate next step in management? A. arterial blood gas B. computed tomography scan of the chest C. face mask oxygen D. magnetic resonance imaging of the head E. PA and supine chest radiograph

The correct answer is: C Significant blunt force to the thoracic cavity can result in several devastating, life-threatening conditions. These include rib fractures, myocardial contusion, pulmonary contusion, tracheobronchial disruption, hemothorax, and pneumothorax. Diagnosis of blunt injuries can be difficult and require thorough physical examination and high clinical suspicion. Injuries may present subtly or develop over time, and become life-threatening during the course of a resuscitation. The plain chest radiograph remains the standard initial diagnostic study for the evaluation of chest trauma in a hemodynamically stable patient, however all imaging tests should wait until the patient is stabilized via the ABCs. Airway, breathing and circulation should first be addressed before ordering a xray in this patient. An oxygen mask should be placed on the patient to see if their 02 saturation improves. Answer A: An arterial blood gas will only confirm what you already know clinically, which is that the patient is not saturating well. This patient can become quickly unstable and delaying diagnosis and treatment of the underlying problem would place her at further danger. Answer B: CT scan of the chest may ultimately be necessary to evaluate the complete extent of the patient's injuries. However, CT scan would require sending a patient who could potentially decompensate to radiology and would delay definitive treatment of her underlying source of respiratory distress. Chest radiograph can be performed at the bedside and reported in a timelier manner. Answer D: This patient may have suffered a minor traumatic head injury at time of impact by hitting her head on the windshield or steering wheel because her air bag did not deploy. All patients with a Glasgow Coma Score (GCS) less than 9 should have an emergent CT scan of their head to look for evidence of hemorrhage or increased intracranial pressure. It is also reasonable to get a head CT in patients with GCS score between 10-15 who had a loss of consciousness and are exhibiting symptoms suggestive for brain trauma (e.g., vomiting, headache, disorientation). This patient has a GCS of 15, has normal mental status and is not exhibiting any neurologic or funduscopic abnormalities. Also, MRI is not the initial imaging modality of choice in a trauma patient. Answer E: Once the patient is stabilized on oxygen they should be sent for an immediate PA and supine chest Xray or have a bedside xray ordered. The chest X-ray is a rapid screening examination that will identify significant thoracic problems requiring intervention. Chest radiographs in blunt trauma patients are usually taken in the supine position initially until unstable spinal fractures have been ruled out. Then, it is important to get a PA view to appropriately evaluate for small hemothorax, pneumothorax or diaphragm injury. Addressing the ABCs in a trauma patient should be done immediately before any imaging tests are done. The patient should be placed on an oxygen facemask while waiting for the bedside xray or being sent to xray. Bottom Line: Thoracic injuries secondary to blunt chest trauma may present insidiously and require a high index of suspicion. All stable patients who suffered significant blunt trauma to the chest should get an immediate chest x-ray, including both supine and PA views if possible, to identify and treat potentially life-threatening injuries as early as possible.

A 16-year-old male presents to the emergency department in a panic because he is allergic to peanuts and unknowingly ingested some today. He reports shortness of breath, a rash, and pruritus of the face and chest which started several minutes after ingestion. Examination reveals urticaria of the chest. You note paravertebral spasms from T1-4. Which medication should be immediately administered A. albuterol/ipratropium B. benadryl C. epinephrine D. intubation E. watchful waiting

The correct answer is: C The immediate treatment of anaphylaxis includes administration of IM epinephrine along with H1/H2 blockers and steroids, however, epinephrine should be the FIRST medication administered in a patient with suspected anaphylaxis. A delay in giving Epinephrine could result in rapid decompensation of the patient. Epinephrine 1:1000 concentration is dosed at 0.3-0.5 mL in adults and 0.01 mL/kg in children. It should be given in the anterolateral thigh by IM injection. Patient's should be observed at least 4-8 hours after getting epinephrine and if there is concern for reoccurrence of the patient's symptoms, they should be admitted for observation. All allergic reactions can have a biphasic course and return hours to days later. Patient's should be discharged with continued steroids as well as an Epipen. The patient in the scenario had immediate symptoms of itching and urticaria as well as shortness of breath after eating a cookie which contained peanuts. This qualifies as anaphylaxis and there should not be a delay in his treatment. With inflammation occurring throughout the head and neck as a result of the allergic reaction, you may see viscerosomatic reflexes causing TART changes from T1-4. Answer A: While the patient will eventually need Albuterol/Ipratropium, Epinephrine should be immediately given since the patient's symptoms are concerning for anaphylaxis. Waiting to give Epinephrine can result in rapid decompensation of the patient. Answer B: Benadryl is an appropriate treatment of allergic reactions, however, this patient is exhibiting symptoms of anaphylaxis and needs intramuscular epinephrine injection before his symptoms worsen. Answer D: This patient does not yet need to be intubated as he is protecting his airway at this time. He needs immediate Epinephrine to hopefully prevent the need for intubation. Answer E: This patient is in anaphylaxis, watchful waiting is never the right answer when there is a life threatening emergency occurring with a patient. Bottom Line: Anaphylaxis is highly likely when any ONE of the following criteria is fulfilled: 1. Acute onset of an illness (minutes to several hours) with involvement of the skin, mucosal tissue, or both (eg, generalized hives, pruritus or flushing, swollen lips-tongue-uvula) AND respiratory compromise (eg, dyspnea, wheeze, bronchospasm, stridor, hypoxemia) or hypotension or signs of end-organ dysfunction (eg, hypotonia, collapse, syncope, incontinence). 2. TWO or more of the following: involvement of the skin-mucosal tissue (eg, generalized hives, itch-flush, swollen lips-tongue), respiratory compromise, hypotension, or persistent gastrointestinal symptoms (eg, crampy abdominal pain, vomiting).

A 44-year-old female presents to the emergency department after suffering a bee sting on her finger. She reports that she is severely allergic to bees. History reveals that she injected her son's epinephrine autoinjector directly into the affected digit. Past medical history is positive for a bypass graft of her femoral to popliteal artery for peripheral arterial disease. Vital signs are found to be within normal limits and the patient does not appear to be in distress. Physical exam is remarkable for an anxious woman with a finger that is pale and cool to the touch. Sensation is limited and the finger is non-blanching. The most appropriate management is A. digital massage B. nitroglycerin C. phentolamine D.phenylephrine E. warm water immersion

The correct answer is: C The patient has had an epinephrine injection which would induce vasospasm of the thumb as evidenced by the pallor, and cool feel of the digit. In the event of a small injection from an epinephrine autoinjector you could reasonably observe the patient and discharge to home after color started to return to the digit. There is no clear evidence that active treatment is any better than observation alone. Also treatment with phentolamine is indicated in patients that have decreased flow as in peripheral vascular disease. This woman does suffer from peripheral vascular disease noted by the history of her femoral artery bypass. Phentolamine is a vasodilator by alpha-1 adrenergic blockade and results would be seen within minutes. It is most commonly used in hypertensive crisis from a pheochromocytoma. Recent studies have disproved that epinephrine cannot be used on appendages such as the fingers, nose or penis. Only phentolamine has been shown in published literature to reverse the effects of epinephrine and therefore reverse the digital ischemia. Answer A: Massaging the digit may help to improve blood flow in an otherwise healthy patient but it cannot reverse the effects of vasoconstriction from the epinephrine. In a patient such as this one with peripheral blood flow issues, massaging the digit will not be able to counteract the effects of the epinephrine and the already decreased flow issues due to the existing comorbidities. There is no clear evidence to support this as a treatment option. Answer B: Nitroglycerin paste does cause dilation of the venous circulation, not the arterial system and therefore would not increase blood flow to the affected area. Nitroglycerin paste is formulated to release nitroglycerin in a slower controlled manner rather than rapid dissolving sublingual tablets. Absorption as exhibited in the sublingual formulations is commonly used in patients with angina. Nitroglycerin works to decrease the preload (at lower doses, vasodilator at higher doses), by causing venodilation in these patients, which will decrease the overall cardiac load allowing the heart "rest." By the heart "resting" the demand for blood supply to the myocardium is reduced and therefore the pain of the angina is reduced. Answer D: Phenylephrine is a vasopressor that works by vasoconstricting peripheral vasculature. Phenylephrine is an alpha agonist. It has many clinical uses including acting as a vasopressor, decongestant, detumescent (used in priapism) and mydriatic agent. This and any other vasopressor agents would be contraindicated in this patient as her vital signs are stable and may only further increase ischemia to the affected digit. Answer E: Warm water would cause dilation of the peripheral vasculature however it would not be able to counteract the effects of the vasoconstriction caused by the epinephrine. This may be a palliative option in a milder case or if the patient did not have any issues with peripheral vascular disease. Bottom Line: Only phentolamine can counteract the effects of epinephrine by alpha-1 adrenergic blockade in a patient exhibiting symptoms from the epinephrine injection. Milder cases may be observed and discharged.

A 27-year-old male presents to the emergency department via EMS after a motor vehicle accident. Emergency personnel report the patient admits to severe abdominal pain and has periumbilical ecchymosis. Vitals are obtained and reveal the following: Temperature 36.8ºC (98.2ºF) Heart rate 136/min Blood pressure 67/43 mmHg Respiratory rate 32/min Oxygen saturation 95% on 2L nasal cannula A urinary catheter is inserted revealing a urine output of approximately 10 mL/hour. The most likely class of hemorrhagic shock is A. I B. II C. III D. IV

The correct answer is: C The patient in this scenario is in shock. It is likely hemorrhagic shock secondary to the fact that he was involved in a trauma, has periumbilical ecchymosis (Cullen's sign), and abdominal pain. Cullen's sign is indicative of intra-abdominal hemorrhage which this patient likely has. He is in class III of hemorrhagic shock due to the fact that he has hypotension, heart rate between 120-140, respiratory rate between 30-40, and a urine output of between 5-15 mL/hour. According the American College of Surgeons there are different classes of shock that are taught in standard advanced trauma life support (ATLS) classes. There are four classes of hemorrhagic shock. and there are eight components to determine which class of hemorrhagic shock a patient is in. This components are: blood loss in mL, percent blood volume loss, pulse, blood pressure, pulse pressure, respiratory rate, urine output, and mental status. Please reference the image attached to the explanation for further information about various classes of shock. Bottom Line: Class III of hemorrhagic shock includes: hypotension, heart rate between 120-140/min, respiratory rate between 30-40/min, and a urine output of between 5-15 mL/hour. Please reference the image attached to the explanation for further information.

A 68-year old presents with an INR of 10.0. History reveals a recent discharge diagnosis of atrial fibrillation, during which he was started on warfarin. His INR upon discharge was 2.3. He otherwise feels well, but was recommended to be evaluated in the emergency department. The most appropriate management at this time would be to withhold warfarin and administer vitamin K A. intramuscularly B. intravenously C. orally D.orally and lovenox bridge E. orally and two units FFP

The correct answer is: C There are three main approaches for the reversal of warfarin-induced coagulopathy. The first is to stop warfarin therapy; the second is to administer vitamin K; the third is to administer fresh frozen plasma (FFP), prothrombin complex concentrate, or recombinant Factor VIIa. For INR between 3.5 and 5 without major bleeding, the recommended therapy is to lower the dose or omit a dose. For INR of 5 to 9 without major bleeding, the recommended therapy is to skip the next one or two doses or administer 1 to 2.5 mg of vitamin K orally. For an INR > 10 without major bleeding, the recommended therapy is to hold the next one or two doses and administer 2.5 to 5 mg of vitamin K orally. An elevated INR with life-threatening bleeding should be treated with 10 mg vitamin K IV and typically 3-4 units of FFP. Answers A & B & D & E: For an INR > 10 without major bleeding, the recommended therapy is to hold the next one or two doses and administer 2.5 to 5 mg of vitamin K orally. Bottom Line: For an INR > 10 without significant bleeding, the recommended therapy is to hold the next one or two doses of warfarin and administer 2.5 to 5 mg of vitamin K orally.

A 34-year-old female presents to the emergency department with a 24 hour history of dyspnea, headaches, and weakness. Past medical history reveals she had a kidney transplant 4 years ago but currently has normal renal function. Home medications include tacrolimus, metoprolol, metoclopramide and dapsone. Physical examination reveals gray skin discoloration, pale conjunctiva, and 3/5 muscle strength in all extremities bilaterally. Pulse oximetry shows 85% saturation on room air, however, an arterial blood gas reveals a PaO2 of 96 mmHg. Co-oximetry is performed and confirms the diagnosis. Which of the following is the most appropriate management for this condition? A. high-flow oxygen via non-rebreathing mask B. hyperbaric oxygen therapy C. methylene blue D.observation with supportive care E. sodium nitrite

The correct answer is: C This patient is presenting with methemoglobinemia. The presence of methemoglobin in the blood causes iron to exist in the ferric form (Fe3+) instead of the usual ferrous form (Fe2+) preventing hemoglobin from binding readily to oxygen, leading to decreased oxygen saturation. Symptoms correlate directly with the percent of red blood cells that contain methemoglobin. Initially, patients may be asymptomatic. As methemoglobin levels rise above 15% patients may develop skin and/or blood color changes("chocolate colored venous blood"). If levels are >15% patients may develop cardiac and neurologic symptoms including dyspnea, headache, weakness, confusion, seizure, and death (usually when >70%). Pulse oximetry is usually low but cannot be used to accurately assess hypoxia because it is based on light absorbance, with methemoglobin falsely elevates. Patient may have "saturation gap" where this is a difference between the oxygen saturation measured on pulse oximetry and the oxygen saturation calculated on the basis of ABG results. This is because the partial pressure of oxygen (PO2) value of the ABG measurement reflects plasma oxygen content and does not correspond to the oxygen-carrying capacity of hemoglobin. Therefore, it will remain in the normal range despite a pulse oximetry reading in the low 80's. Diagnosis is made using co-oximetry, an accurate device for measuring methemoglobin. Methemoglobinemia may be congenital or acquired. Oxidant drugs such as dapsone, which may be used for PCP prophylaxis in immunocompromised patients, can cause methemoglobinemia. Other agents include metoclopramide, recreational drugs (phenylamine, cocaine, volatile nitrates), antimalarials (primaquine, chloroquine, sitamaquine), Antineoplastic agents (cyclophosphamide, ifosfamide, flutamide, and 3-aminopyridine-2-carboxaldehyde thiosemicarbazone), and certain analgesics (acetaminophen, phenacetin, celecoxib). Management involves stoppage of the offending agent and administration of methylene blue which increases methemoglobin conversion to hemoglobin. Answer A: High-flow oxygen with a non-rebreather is appropriate for those who have carbon monoxide poisoning. These patients will typically present with similar signs and symptoms to the patient above (headache, dizziness, nausea). It is important to note that standard pulse oximetry does not determine those exposed to carbon monoxide, for the diagnosis is made based upon the history in combination with elevated carboxyhemoglobin levels on an ABG. Answer B: Hyperbaric oxygen therapy can be used to deliver 100% oxygen and is used as an antidote for carbon monoxide poisoning. Hyperbaric oxygen treatment may be used in situations where methylene blue therapy is ineffective or contraindicated. It acts by oxygenating tissue through oxygen dissolved in plasma, rather than through hemoglobin-bound oxygen. Answer D: Supportive care is not the correct answer because without reversal of the methemoglobinemia, symptoms can worsen leading to neurologic impairment, seizures, and arrhythmias which can lead to death. Answer E: Sodium nitrite is a potential cause of methemoglobinemia, not an antidote. However, sodium nitrite can be used as an antidote for cyanide poisoning. The nitrite acts by oxidizing hemoglobin to methemoglobin, which has a high affinity for cyanide. Cyanide binds avidly to methemoglobin, forming cyanmethemoglobin and releasing cyanide from cytochrome oxidase. Then, thiosulfate is commonly administered, which binds cyanmethemoglobin and is renal excreted as thiocyanate. Methylene blue is commonly required as well to reverse the effects of the nitrite. Bottom Line: Methemoglobinemia is congenital or acquired modification of hemoglobin that leads to tissue hypoxia from decreased oxygen saturation and decreased oxygen content. Think of methemoglobinemia in a patient with a low oxygen saturation on pulse oximetry but a normal PaO2 on ABG. Treatment is with methylene blue.

An 86-year-old female with end-stage Alzheimer's disease presents to the emergency department in severe respiratory distress from the nursing home. Vitals reveal a temperature of 39.4ºC (103ºF), a heart rate of 138/min, a blood pressure of 90/50 mmHg, respiratory rate of 32/min, and an oxygen saturation of 84% on a non-rebreather mask. Her Glasgow coma scale score is currently 7. A close friend from the nursing home is present and states that she repeatedly told her that she "did not want to be kept alive by machines." The most appropriate management at this time is A. administer morphine for comfort measures B. bag-valve mask until the family can be contacted C. intubate immediately D.obey the patient's wishes and do nothing E. place on oxygen via a nasal cannula

The correct answer is: C This scenario presents a perplexing dilemma. The physician is faced with an option of either obeying the patient's potential wishes versus attempting to save the patient's life. A competent patient with the capacity to comprehend his or her medical condition has a full right to refuse care and withhold medical treatment. Unfortunately, this patient's mental status precludes her decision-making capacity. A way that a patient may express his or her wishes when one looses capacity to make medical decisions is via an advance directive. The two most common forms of advanced directives are a living will and a health care proxy. A health care proxy is an individual that the patient appoints to make heath care decisions in the event one looses the ability to make decisions for themselves. A health care proxy can then speak on behalf of the patient in regards to consenting to treatments and tests or withholding treatment. The proxy is thus able to indirectly express the patient's heath care wishes and describe what the patient would have wanted if he or she had capacity. A living will is a written document of advanced directives, which delineates the exact type of treatment or interventions, the patient would want to have and which should be withheld. In this scenario, there is no mention of an advanced directive and in this situation; the direction of care a physician should follow is relatively simple and includes interventions to save a patient's life. The patient does not present with any documentation regarding DNR / DNI (Do not resuscitate / Do not intubate), thus the correct management at this time would be to intubate this patient. The patient likely has an underlying pneumonia with sepsis and altered mental status thus requiring aggressive interventions to save her life. Answer A: This is an incorrect answer. Administering morphine to provide comfort measures is incorrect in this scenario. Morphine may provide some relief to respiratory fatigue this aid with comfort measures but it will inadvertently shorted the patients life. Essentially morphine will lead to decreased respiratory drive, worsening hypoxemia and acidemia, leading to respiratory failure. In the situation where patient's advanced directives are unknown, the physician ethical duties to save a patient's life should not be breached. Answer B: This is an incorrect answer. Although the family members may know exactly what the patient would have wanted, and be in unanimous agreement as to what interventions the patient would prefer, this is not always the case. In some circumstances, reaching the family can take hours or days and in others family members will disagree as to what should be done. If no health care proxy, living will, nor documentation regarding end of life care (DNR / DNI) is present, it is imperative to perform whatever interventions are necessary to save the patient's life. Answer D: This is an incorrect answer. Obeying the patient's wishes and doing nothing would lead to patient deterioration. With no evidence of advanced directives we do not know exactly what the patient would have wanted to be done in this situation. Unfortunately we do not know what interventions the patient would have wanted in this situation, such as antibiotics, intravenous fluids or endotracheal intubation to aid with her breathing. Answer E: This is an incorrect answer. Placing the patient on nasal cannula would lead to further hypoxia. The patient presents hypoxic on an oxygen non-rebreather mask. A non-rebreather oxygen mask provides an oxygen concentration of approximately 60-80% O2 whereas a nasal cannula provides an oxygen concentration of anywhere from 25-40% O2. Placing a patient on a nasal cannula would hinder the patient's respiratory status. Bottom Line: Without an advanced directives including a health care proxy or a living will, the physician should presume that a patient would have wanted all interventions necessary to save his or her life and should not withhold any treatment

A 33-year-old female presents to the emergency department complaining of shortness of breath and chest pain of 1 day duration. She denies taking oral contraceptives, history of DVT/PE, recent trauma or surgery, hemoptysis, active cancer, hemoptysis, or leg swelling. The patient reveals that her chest pain is located in the right anterior chest, is sharp and only occurs when she takes a deep breath. Vital signs reveal blood pressure 115/75 mmHg, heart rate 115/ min, respiratory rate 18/min, oxygen saturation 98% Physical examination reveals bilateral equal and clear breath sounds, minimal tachycardia without murmurs, rubs or gallops, and no lower extremity edema. You note bogy, warm soft tissue swelling around the thoracic paravertebral musculature from T2-6 that is tender to palpation. The most appropriate test to order would be: A. arterial blood gas B. chest radiograph C. D-dimer test D. pulmonary angiogram E. spiral CT of the chest

The correct answer is: C When evaluating the patient for pulmonary embolism, both Wells and PERC criteria must be used. The patient should first be evaluated using Wells criteria for risk stratification. If patient is low risk, then PERC criteria can be used. If patient meets all PERC criteria, then PE is essentially ruled out. If patient does not meet PERC criteria or if patient falls into intermediate PE risk by Wells criteria, then D-dimer should be ordered. ***Of note, some patients will fall into intermediate category by Wells criteria, but if your suspicion is high, order a CT angio instead of a dimer***. If patient falls into high risk category based on Wells criteria, then CT angio should be ordered. Remember that you can use somatic dysfunction to help you make/confirm your suspected diagnosis. In this situation, you see acute TART changes from T2-6 which are the result of viscerosomatic changes from the underlying pulmonary pathology. Answer A: In arterial blood gas is not the appropriate initial test order if one is suspicious for pulmonary embolism. However, if one is ordered it may show respiratory alkalosis due to the patient's tachypnea, hypoxemia, and a widened A-a gradient. Note though, that a normal arterial blood gases not exclude a pulmonary embolism. The Alveolar-arterial gradient is measure of the difference between the oxygen in the alveoli and the arterial oxygen content and can help determine the source of the hypoxemia. A high/widened A-a gradient is indicative that the deoxygenated blood from the right side of the heart is being shunted to the left side of the heart without undergoing sufficient oxygenation in the alveoli. Calculation of an A-a gradient utilizing an arterial blood gas (on room air and at sea level): [150 - (PO2 + PaCO2/0.8)] - PaO2. Note that a normal A-a gradient for a young adult non-smoker breathing air is between 5 to 10 mmHg. Normally the A-a gradient increases with age. For every decade a person has lived, the gradient should be expected to increase by 1 mmHg. (Normal Gradient Estimate = (Age/4) + 4). Answer B: A chest x-ray is indicated in any patient with hypoxia. However if the clinical suspicion is high for pulmonary embolism, this is not the initial appropriate test order. Chest x-rays in patients with pulmonary embolism's are usually abnormal but the findings are typically nonspecific including pleural effusion or atelectasis. Classic signs seen in pulmonary embolism's include Hampton's hump and Westermark's sign. A Hampton's hump is a pleural-based wedge-shaped density located on the lateral field of the lung and is indicative of a infarcted lung. A Westermark's sign is rarely seen in patients with pulmonary embolism, but shows decreased vessel markings distal to the pulmonary embolus. Answer D: Pulmonary angiography can identify a clot in the pulmonary artery as a filling defect. It is performed by interventional radiologists and requires a catheter to be inserted into the pulmonary artery by way of the femoral vein. Advantages include the ability to demonstrate small defects in the smaller/distal pulmonary arteries, the ability to measure pulmonary arterial pressures and the ability to treat pulmonary emboli with intra-pulmonary catheter-directed modalities. One can also deploy an inferior vena cava filter to prevent further propagation of DVT's to the pulmonary vasculature. Disadvantages include lack of availability, radiation exposure, cardiac arrhythmias, contrast related complications and cardiac or pulmonary arterial perforation which can lead to death. This is not the correct initial test of choice in this question. Answer E: CT angiography is a study of choice for confirmation of a pulmonary embolism. It has high sensitivity and specificity and can demonstrate a filling defect in the pulmonary artery. In this scenario, it is more cost-effective to order a D-dimer test first as a negative D-dimer would effectively rule out a pulmonary embolism. The acute life-threatening complications from CT scanning include anaphylactoid reaction to contrast, contrast induced nephropathy, and contrast extravasation into a limb which can cause compartment syndrome, secondary thrombophlebitis and pain. Bottom Line: D-dimer is the initial study of choice to order in a low risk patient as part of PE workup.

A 21-year-old male is brought into the emergency room with a left hand wound. History reveals that a few days ago he was intoxicated and involved in an altercation in which he punched someone in the mouth. Physical exam reveals marked edema and erythema just proximal to the 5th metacarpophalangeal joint with a 2 cm laceration on the lateral edge of the metacarpal head. You note swelling of axillary lymph notes, restricted left supraclavicular soft tissue, and paravertebral hypertonicity from T1-4. Which of the following will most likely be found upon culture of this wound? A. Brucella canis B. Capnocytophaga canimorsus C. Eikenella corrodens D. Pasteurella multocida E. Staphylococcus aureus

The correct answer is: C While Eikenella corrodens, staphylococcal and streptococcal bacteria are all commonly associated with human bite infections, most commonly these infections culture Eikenella spp. most often. The pharmacological agent of choice would be amoxicillin-clavulanic acid. Those wounds should never be primarily closed due to the extremely high risk for abscess formation. The only exception to this is a wound on the face or scalp as they are highly vascular and risk of infection is < 10%. For established infections ampicillin-sulbactam, cefoxitin, or piperacillin-tazobactam should be used. No matter the infectious organism, having an infection in the hand can result in various somatic dysfunction that can be helpful in the clinic setting to ensure that you are addressing all aspects of the injury/infection. Inflammation of the arm and axillary lymph notes can occur as the infection spreads. As a result you may see restrictions in the thoracic inlet, shoulder, upper thoracic region that may need to be addressed. TART changes from T1-4 represent viscerosomatic reflex as a result of somatic dysfunction in the upper extremity. Answer A: Brucella canis is a causative agent of brucellosis. This presents with fever, headache, myalgias, and nonspecific laboratory findings. Dogs are the most common vector for transmission of brucellosis however, pigs, cattle and goats have also been implicated in the disease process. Treatment of choice is with doxycycline plus gentamicin or streptomycin or rifampin or trimethoprim-sulfamethoxazole plus gentamicin can be used. Answer B: Capnocytophaga canimorsus is a common etiologic agent elicited from cultures obtained after a patient sustains a dog bite. C. canimorsus is a normal flora for canines and is most often cultured out from wound sustained from a dog bite. In asplenic patients and those with chronic liver disease, infection may result in fatal sepsis. Treatment of choice for this pathogen would be amoxicillin-clavulanic acid. Answer D: P. multocida is a rare isolate of human bites. The organism is a gram-negative, non-motile coccobacillus that commonly causes infections due to cat scratches or dog/cat bites. Penicillin is the first-line therapy with doxycycline, levofloxacin, and cephalosporins as alternative therapy. Answer E: Staphylococcal species are a ubiquitous pathogen found in many places on the body as part of a normal flora. It is possible to elicit Staphylococcal cultures from a wound but may be part of the normal skin flora. Treatment for Staphylococcal infection from a human bite would be with amoxicillin-clavulanic acid. Bottom Line: While "fight bite" or human bite injuries will have a dominant pathogen, most often Eikenella corrodens, must be treated with amoxicillin-clavulanic acid.

A 25-day-old female is brought into the emergency department by her parents who report the neonate has been fussy and had a fever at home of 101.5°F (38.6°C). Her parents state that she has been having increased difficulty breathing but report that she has been feeding well. Vital signs reveal a temperature of 39.4ºC (103ºF), heart rate of 145/min, blood pressure of 75/35 mmHg, respirations of 38/min, and oxygen saturation of 94%. Physical exam reveals a well appearing child with an increased work of breathing, nasal flaring, and grunting. A plain film chest radiograph, complete blood count, and blood cultures have been obtained with results pending. What is the next best step in management? A. CT scan of the chest B. IM lorazepam C. IV ceftriaxone D. lumbar puncture E. stool ova and parasites

The correct answer is: D A lumbar puncture, to obtain CSF for culture and cell count is indicated to determine the culprit for the neonates fever. Any child less than 30 days old presenting to the emergency department with a fever must have the full septic workup performed. A full septic workup includes a chest radiograph, blood cultures, complete blood count, lumbar puncture and urinalysis. The child's presenting symptoms would suggest pneumonia as the cause for the fever and difficulty breathing. As bacteremia and UTI may also be present a full workup still must be performed, even if the child is well appearing. Once older than 30 days a lumbar puncture is no longer indicated in well-appearing infants with a fever of unknown source as the risk for meningitis drops to ~1%. If the child has a clinically diagnosed bacterial infection (otitis media, pneumonia, cellulitis) this is considered a legitimate "source" for fever. However the very young (< 30 days old) may still have concomitant bacteremia or UTI and as such need a full septic work up. If the child were to have meningitis, CSF results may point to the etiology before culture results return. Bacterial meningitis will have low glucose, high protein and high WBC's. Viral meningitis may have mostly normal results but still with high WBC's. It is important to distinguish between real results and elevated WBC's because of a traumatic tap. A ratio of 500-1000:1 RBC's:WBC's is generally considered a traumatic tap. If the WBC count is higher that this ratio, an infectious source must be considered. Urinary tract infection (UTI) should always be in the emergency physician's differential as a source for a fever in a child and considered in the septic algorithm. In neonates (< 30 days old) the risk of UTI is ~10-15% regardless of gender or circumcision status. Once over the age of 30 days the risk drops to about 9% overall (20% uncircumcised boys, 10% in girls and 2% in circumcised boys). When performing the urinalysis the urine should always be collected by catheter if possible; false positives and false negatives with bag collection are very high. Answer A: A CT scan of the chest would not be indicated in this child to determine the source of the fever. If pneumonia was present, a chest radiograph, clinical picture and elevated white blood cell count would all lead to the clinical diagnosis of pneumonia. Furthermore, in children this young it is reasonable and prudent to limit them to such high doses of radiation by using diagnostic measures other than CT scans. Answer B: Prophylaxis is not indicated for febrile seizures. Anticonvulsant therapy is not indicated for febrile seizures. The side effects of these medications may far outweigh the benefit they provide. It is unclear what causes the seizure, the magnitude of the temperature or simply the change in the temperature. Antipyretics such as acetaminophen and ibuprofen are indicated for treatment of fever, however there is no evidence that these agents will prevent any subsequent seizures. Parental education is key for the child that has a febrile seizure. Parents should be informed that only 50% of children <12 months and 30% of children >12 months will have a repeat seizure. Also the risk of a child with a febrile seizure developing epilepsy is the same as the general public, about 1-2%. Answer C: IV Ceftriaxone would be indicated for a child that is older than 30 days old. In general it is not recommended to administer ceftriaxone to a child less than 30 days old as it is known to displace bilirubin resulting in jaundice and kernicterus, dangerously elevated bilirubin that causes brain damage. If there are no other treatment options, or if cultures and sensitivities suggest ceftriaxone as the best treatment option it may be given but careful attention should be paid to the childs serum bilirubin levels. Of note, on any patient it is vital to obtain cultures (urine, CSF and blood) before antibiotics are started, as antibiotic administration may result in false negative cultures. Answer E: Stool ova and parasites is a stool sample that is analyzed by the lab for parasites and their ova. This test is not within the algorithm for the child with a fever of unknown origin. This test should be ordered on patients with suspected infectious diarrhea that is not remitting. This test is generally ordered with a stool culture and possible fecal leukocytes as well. Once etiology of the infectious diarrhea is determined it is important to consider the most appropriate pharmacologic treatment. Generally it is not advisable to give antidiarrheals as this may lead to the potentially fatal problem of toxic megacolon. Bottom Line: Fever of unknown origin in children less than 3 years of age should always prompt a thorough workup including cultures of blood, urine, CSF and a chest radiograph.

A 2-year-old female is brought into the emergency department via ambulance services after ingested an unknown quantity her father's atenolol. Her parents believe this may have happened 2 hours prior to their arrival in the emergency department. Vital signs are blood pressure 95/60 mmHg, heart rate 65/min, respiratory rate 25/min, and an oxygen saturation of 98%. Physical exam reveals a well appearing child who appears fatigued. Auscultation of the heart is negative for murmurs and the lungs are clear bilaterally. You note muscle paravertebral spasms from T1-4. What is the next best step in management? A. activated charcoal B. IV D50 and sterile water C. IV flumazenil at 0.01 mg/kg D. IV glucagon at 0.07 mg/kg E. IV maintenance fluids

The correct answer is: D Glucagon infusion in patients with a suspected beta-blocker overdose and unstable vital signs is indicated. Glucagon works by competitive inhibition of the beta receptors in the heart muscle. The receptors are very close to each other and the glucagon works to inhibit the higher doses of atenolol from reaching the receptors on the heart muscle. In a 2-year-old child a heart rate of 60 is considered bradycardic and intervention should take place. Normal heart rate for a child this age is between 80-140/min. If the glucagon is insufficient and the child remains bradycardic transcutaneous pacing is indicated for management of the patient. Normal vital signs for a 2-year-old are as follows: heart rate 80-140/min, respiratory rate 20-30/min and a blood pressure of 90-105/55-70 mmHg. Somatic dysfunction from T1-4 may result due to the underlying cardiac dysfunction as a result of viscerosomatic reflexes. Answer A: Activated charcoal is a common method of decontamination of the GI tract. It is widely used because of its safety profile amongst children and adults. It works by Van der Waal's forces adsorbing the potential toxin. Activated charcoal should not be used if the following are suspected: esophageal or gastric perforation or a possible endoscopy may be indicated (the charcoal would coat the GI tract clouding visual inspection). The charcoal should ideally be given within 1 hour of a suspected ingestion and studies show that ingestions that have taken place greater than 1 hour, the adsorption may be severely limited as the GI tract may have already absorbed significant quantities of the toxin. The utility for charcoal in adsorbing ionic toxins such as lithium, lead and iron is very limited. These substances do not adsorb well and will remain in the GI tract to be absorbed. This principle also applies to hydrocarbons and toxic alcohols. Elimination is achieved by defecation. Answer B: D50 infusion should be the treatment of choice for a possible insulin or oral hypoglycemic agent overdose. In any patient where a suspicion of either one of these is possible close monitoring of the patients blood sugar should be paramount and symptoms of hypoglycemia closely noted (confusion, dizziness, diaphoresis, shakiness, weakness). The patient's whose blood sugars drop to low levels (<60 mg/dL) should be rapidly corrected initially with D50 to get the patient stabilized. In a patient who took oral hypoglycemics such as glyburide or glipizide whose effects will last longer than the infused D50, somatostatin or octreotide (a synthetic analog to somatostatin) may reverse the lasting effects of the agent and help to stabilize the patients blood sugar. Answer C: Flumazenil is a centrally acting antagonist of benzodiazepine receptors. Its use in someone who has overdosed on benzodiazepines should be used very judiciously. In the case of a patient with known history of seizure disorder, flumazenil is contraindicated as it may precipitate a seizure. Using flumazenil can take the alprazolam off the receptors and potentially cause a seizure. The following is a list of absolute contraindications to using flumazenil: overdose on unknown substances, suspected dependence on benzodiazepines, suspected cyclic antidepressant overdose, co-ingestion of seizure-inducing agents, known seizure disorder,and suspected increase in intracranial pressure. Answer E: Supportive care with IV fluids would be inappropriate for this child. While an adult can easily tolerate a heart rate of 65, in a 2-year-old this is considered bradycardic. The IV fluids would be appropriate in the setting of hypotension. If the child had ingested a smaller amount and was not undergoing beta-1 blockade as evidenced by the decreased heart rate and was only hypotensive, the fluids would have been the next best step. After infusion of the glucagon in attempts to block the beta-blocker effects, fluids would be appropriate to counteract any hypotension. Bottom Line: Careful monitoring of vital signs can mean the difference between observation and acute intervention in the overdose patient. Being familiar with the vital signs in pediatric patients is of the utmost importance. Glucagon can be used to competitively inhibit the actions of ingested beta-blockers.

A 35-year-old unrestrained, intoxicated driver is involved in a single vehicle accident with head on collision into a tree. He is thrown from the vehicle and found approximately 20 feet from the site of impact. His Glasgow coma scale score at the time of arrival of paramedics is 6. He is subsequently intubated for protection of his airway. Upon arrival to the trauma bay he is found to have equal breath sounds bilaterally but has a heart rate of 140/min and blood pressure is 70/40 mmHg. Which of the following would be the most appropriate fluid to initiate resuscitation in this patient? A. 0.45% normal saline solution B. dextrose 10% in water C. fresh frozen plasma D. Lactated Ringer's solution E.packed red blood cells

The correct answer is: D Hemorrhage leading to hypovolemic shock is the most common cause of post injury death in a trauma patient. Rapid fluid resuscitation and controlling the bleeding source are the keys to restoring adequate circulation. All sources of external bleeding should be quickly identified and treated by applying direct pressure. Sources of internal bleeding, including the thorax, abdomen, or into the soft tissues should be considered based on the mechanism of the traumatic event. Fluid resuscitation requires establishing IV access with two 14 or 16 gauge, large-bore catheters in the upper extremities followed by rapid infusion of a salt balanced solution. Lactated Ringers or normal saline are most commonly used due to their ready availability, cost-efficiency, and lack of risk for transmission of blood borne disease. Answer A: 0.45% normal saline, also referred to as 1/2 normal saline, is a hypotonic solution used as a maintenance fluid in patients with water loss. It is commonly used in the treatment of diabetic ketoacidosis and in patients with gastric losses from nasal gastric tube or vomiting. It should not be used in patients with liver disease, trauma patients or burn victims. Answer B: Dextrose 10% in water is a hypertonic solution that is used as a maintenance fluid, specifically in patients who also some nutrition with glucose. It should not be used as a resuscitative fluid in trauma patients. Answers C & E: If adequate response is not obtained through volume repletion with crystalloids (Lactated Ringers, 0.9% normal saline) then fluid can be replaced with blood products to increase oxygen carrying capacity or improve coagulopathy. Blood is replaced using packed red blood cells, either O-negative or type-specific if type is known. Fresh frozen plasma contains proteins and coagulation factors required for the coagulation, fibrinolytic and complement systems. The decision to give blood products usually is not made until after an initial crystalloid challenge. Bottom Line: Crystalloid solutions such as normal saline or a lactated Ringer's solution should be considered as first line agents for resuscitation of the trauma patient in shock.

An 18-year-old female presents with an acute exacerbation of her chronic gastrointestinal condition. Physical examination and laboratory studies are unremarkable. Osteopathic structural examination reveals a primary somatic dysfunction involving T8-T10. This will most likely result in a somatovisceral reflex that will explain which of the following? A. Biliary colic B. Colicky abdominal pain C.Diarrhea D. Ileus E. Nausea and vomiting

The correct answer is: D Ileus refers to the absence of bowel movement, which can be due to obstruction, electrolyte imbalances, post-surgical bowel paralysis, or somatovisceral reflex with hyper-sympathetic activation. T8-T10 specifically corresponds to the sympathetic innervation of the small intestines and since sympathetic innervation causes decreased intestinal motility and secretion, this answer makes the most sense. The other answers can be dismissed as they are all actions of the parasympathetic pathway. Answer A: Biliary colic refers to right upper quadrant pain felt after a meal, which is caused by the gallbladder contracting against an occluded cystic or common bile duct by a gallstone. This effect is visceral in origin and is not caused by a somatic dysfunction. Answer B: Similar to biliary colic, colicky abdominal pain describes the pain felt when any organ is actively contracting against some sort of obstruction or while inflamed. Typically colicky pain occurs with gallbladder or stomach contraction and intestinal peristalsis. Contraction of the gallbladder and stomach and increased motility of the bowels is typically controlled by parasympathetic innervation which corresponds to C1-2 or S2-4 vertebral levels, which is not the case in this scenario. Answer C: Diarrhea can certainly be caused by hyper parasympathetic somatovisceral activity to the bowel causing increased GI tract secretion and motility. However, the parasympathetic associated vertebral levels are C1-C2 or S2-S4 and would not correlate in this scenario. Answer E: Nausea and vomiting can certainly be caused by somatovisceral reflexes and would typically occur with stomach contraction and upper esophageal sphincter relaxation, which are both promoted by parasympathetic nervous responses. Since the patient in this case has a somatic dysfunction corresponding to sympathetic activity, this answer would be unlikely. Bottom Line: Decreased intestinal motility is part of the sympathetic nervous action to the bowel.

A 35-year-old presents with fever, cough, and shortness of breath. Point of care testing is positive for influenza B. Which of the following would NOT provide appropriate coverage against this organism? A. influenza vaccine B. oseltamivir C. peramivir D. rimantadine E. zanamivir

The correct answer is: D Influenza is a highly infectious viral illness caused by the single-stranded RNA virus orthomyxovirus. Peak influenza activity in temperate climates arises between late December and early March. It is transmitted by aerosolized or droplet transmission from the respiratory tract of infected persons. There are two major types of influenza, A and B. The measures used to reduce the impact of influenza are immunoprophylaxis with inactivated vaccine and/or chemoprophylaxis with influenza-specific antiviral agents. A trivalent vaccine is prepared on an annual basis with two strains of influenza A and one strain of influenza B representing the likely influenza viruses to circulate in the upcoming winter. Vaccine efficacy depends on the age and immunocompetence of the vaccine recipient as well as the degree of similarity between the viruses in the vaccine and those in circulation. Four antiviral agents are approved for preventing or treating influenza. Amantadine and rimantadine are approved by the U.S. Food and Drug Administration for both prophylaxis and treatment of influenza type A in persons one year of age and older. Zanamivir is approved for treatment, not prophylaxis, of influenza A or B in persons seven years of age and older who have been symptomatic for no more than two days. Oseltamivir is approved for the treatment of influenza A and B in persons one year of age and older who have been symptomatic for no more than two days. Oseltamivir is approved for prophylaxis of influenza infection among persons thirteen years of age and older and indicated for the treatment of acute, uncomplicated influenza in patients eighteen years and older who have been symptomatic for no more than two days. Peramivir is an effective treatment modality for influenza A and influenza B in uncomplicated influenza in patients eighteen years and older who have been symptomatic for no more than two days. Answer A: The final question asks regarding the treatments coverage rather than whether the answer choice is a treatment. Influenza vaccine is used for prophylaxis against both influenza A and B. Answer B: Oseltamivir is approved for the treatment of influenza A and B in persons one year of age and older who have been symptomatic for no more than two days. Oseltamivir is approved for prophylaxis of influenza infection among persons thirteen years of age and older and indicated for the treatment of acute, uncomplicated influenza in patients eighteen years and older who have been symptomatic for no more than two days. Answer C: Peramivir is an effective treatment modality for influenza A and B in uncomplicated influenza in patients eighteen years and older who have been symptomatic for no more than two days. Answer E: Zanamivir is approved for treatment, not prophylaxis, of influenza A or B in persons seven years of age and older who have been symptomatic for no more than two days. Bottom Line: The influenza vaccine, oseltamivir, peramivir, zanamivir will influenza A and B while rimantadine and amantadine cover influenza A only.

A 19-year-old female is brought to the emergency department by her friends with altered mental status. Her friends stated that she consumed an unknown alcoholic substance more than 12 hours ago. She presents with persistent vomiting, headaches, and reports decreased visual acuity. Vitals are obtained revealing a temperature of 37°C (98.7°F), heart rate of 102/min, blood pressure of 120/90 mm Hg, respirations of 28/min, and a spO2 of 100% on room air. An arterial blood gas is immediately available and reveals: pH 7.14 pCO2 16 mmHg pO2 103 mmHg Bicarbonate 3 mmol/L Physical examination reveals a young female with psychomotor retardation and marked drowsiness. Her cranial nerve exam is significant for decreased pupillary response to light. A basic metabolic panel is obtained and reveals the following: Glucose 65 mg/dL BUN 10 mg/dL Sodium 133 mEq/L Potassium 5 mEq/L Chloride 101 mEq/L Carbon dioxide 16 mmHg Calcium 0.87 The most likely cause is A. alcoholic ketoacidosis B. diabetic ketoacidosis C. ethylene glycol D. methanol E. salicylates

The correct answer is: D Methanol toxicity is a common presentation in the emergency room. Every emergency physician must consider methanol toxicity when a patient presents after a known history of consuming an "alcoholic" beverage leading to acute intoxication followed by stupor and visual disturbances 12 hours after consuming the beverage. Initial diagnostic evaluation will include basic chemistry labs as well as arterial blood gas (ABG). The basic chemistry can demonstrate electrolyte disturbances including sodium and chloride and an ABG can give you an idea of the acid base disturbance including pH, pCO2 as well as bicarbonate concentration. A low pH would prompt you to consider acidemia with classification based on pCO2 and bicarbonate. In this case a low bicarbonate would suggest a metabolic acidosis with an elevated pCO2 which suggests an attempt for respiratory compensation. Subtracting cations from ions according to the following formula: Anion gap = Na - (Cl + HCO3) An anion gap is found to be 29 mEq/L (normal range: 8-16 mEq/L). Differentials for anion gap metabolic acidosis can be considered with help from the mnemonic CAT-MUDPILES (cyanide, alcoholic ketoacidosis, toluene, methanol, uremia, diabetic ketoacidosis, propylene glycol,isoniazid, lactic acidosis, ethylene glycol, salicylates). Serum levels of methanol will help solidify the diagnosis. During initial presentation the physician should attempt to obtain as much information about the offending agent as possible which includes obtaining the original container and interviewing family and friends. An initial physical exam will show CNS depression as well as an afferent pupillary defect, which is a classic finding in methanol toxicity. Vital signs are significant for tachypnea and tachycardia as the body attempts to compensate for the metabolic acidosis by exhaling CO2. This is expected with metabolic acidosis, but helps rule out autonomic system toxicity. The patient's presenting symptoms as well as her history of mental status changes within 12 hours of consuming alcohol followed by the visual disturbances will lead the physician to consider methanol toxicity rapidly. Answer A: Alcohol acidosis is seen in men or women with a history of chronic alcohol use who present after a recent binge drinking as in this case. Pathophysiology includes the conversion of ethanol to acetaldehyde and then acetic acid which is ultimately used for fatty acid production. The high production of fatty acid is used for ketone production which can be clinically significant and lead to a ketoacidosis. Ketoacidosis would cause a gap metabolic acidosis. Clinical presentation, however, can differ from acute methanol toxicity absence of CNS depression and usually present with abdominal pain or tenderness from esophagitis, pancreatitis, or hepatitis given the chronic nature of the alcoholism. Vomiting, which is also classically seen, will lead to a potassium depletion, which is not seen in this patient. Answer B: Patients with a history of diabetes who present with CNS depression are evaluated with a suspicion of diabetic ketoacidosis (DKA) which includes a metabolic panel and arterial blood gas. This condition, unlike methanol toxicity, includes the triad of hyperglycemia, anion gap metabolic acidosis, and ketonemia. In this case, the patient has a normal blood sugar which excludes DKA. Answer C: Due to its sweet taste, and accessibility for suicidal attempts, this is a common toxicity. The mechanism of toxicity involves the alcohol dehydrogenase leading to glycolic acid production which is the cause of the metabolic acidosis with an anion gap. Further down the biochemical pathway oxalate crystals and calcium oxalate which leads to renal insufficiency. Ethylene glycol differs from methanol by the lack of visual disturbances classically seen in methanol poisoning. Answer E: Salicylate overdose acid base disturbance is initially seen as a respiratory alkalosis due to direct stimulation of the respiratory drive centers leading to hyperventilation and subsequent metabolic compensation with an elevated HCO3-. This phase occurs during the first 12 hours. After the first 12 hours, potassium loss from the kidneys leads to paradoxical aciduria with an anion gap, lasting another 12 hours. An anion gap with acidemia should lead to the suspicion of salicylate overdose, however, this patient has a metabolic acidemia without a respiratory compensation leading away from the picture of a mixed acid base disturbance. Bottom Line: Methanol toxicity must be considered in a patient with mental status changes, visual disturbances, and with a blood gas consistent with a wide anion gap metabolic acidosis.

A 26-year-old G5P0222 female at 28 weeks' gestation presents to the emergency department with painless vaginal bleeding. History reveals she smokes cigarettes daily. Sterile speculum exam reveals a large amount of bright red blood oozing from the cervical os which is closed. The most likely etiology of the bleeding is A. ectopic pregnancy B. endometriosis C. inevitable abortion D. placenta previa E. placental abruption

The correct answer is: D Placenta previa occurs when placental tissue is overlying or very close to the internal cervical os. As the uterus enlarges and the cervix dilates, some of the blood vessels near the cervix tear causing vaginal bleeding. >90% of placenta previa seen on ultrasound before 20 weeks does not end up as placenta previa at term. Risk factors include multiparity, prior c-section, multiple gestations, increased maternal age, previous placenta previa, smoking, and assisted fertilization. Placenta previa can be classified as complete, partial, marginal, or low-lying. Painless vaginal bleeding after 20 weeks is placenta previa until proven otherwise. Diagnosis is made by ultrasound. Management depends on stability of the patient and the fetus and can range from monitoring the patient to delivering the patient by C-section. Answer A: Ectopic pregnancy usually presents with abdominal pain and vaginal bleeding. Risks for ectopic include IUD, previous ectopic, pelvic surgery, or PID. Ultrasound is diagnostic test of choice in conjunction with B-hCG level. Answer B: Endometriosis is ectopic endometrial tissue with cyclic hormonal responsiveness occurring primarily on the pelvic peritoneum and ovaries. Answer C: Threatened abortion: vaginal bleeding, closed cervical os, viable intrauterine pregnancy confirmed. Inevitable abortion: vaginal bleeding, open cervical os, POC (products of conception) not yet expelled. Incomplete abortion: vaginal bleeding, cervical os is open with partial passage of POC/some retained POC. Complete abortion: vaginal bleeding, closed cervical os, complete passage of POC, no retained POC. Missed abortion: fetal demise with no uterine activity to expel. Septic abortion: spontaneous abortion complicated by intrauterine infection. Answer E: Signs of placental abruption include vaginal bleeding, abdominal pain, contractions, uterine rigidity and tenderness, and non-reassuring fetal heart rate. Bottom Line: Painless vaginal bleeding in a patient >20 weeks should be concerning for placenta previa, especially with a history of smoking.

A 42-year-old female presents to the emergency department in respiratory distress after being stung by a bee. Her blood pressure is 100/70 mmHg, heart rate is 110/min, respiratory rate is 22/min and oxygen saturation is 88% on room air. Physical examination reveals generalized urticaria, diffuse wheezing with stridor, and swelling of the tongue and conjunctiva. The patient is subsequently prepared for rapid sequence intubation and is given succinylcholine. Which of the following is a CONTRAINDICATION for this medication? A. cataracts B. cirrhosis C. hypokalemia D. myopathy E. renal insufficiency

The correct answer is: D Succinylcholine is a depolarizing neuromuscular drug that is most commonly used to facilitate tracheal intubation. It acts by binding nicotinic receptors for an extended period of time leading to prolonged stimulation of the receptor. Initial depolarization of the receptor causes transient muscle twitching (fasciculations). However, continued binding of the drug prevents impulse transmission from the receptor, resulting in temporary flaccid paralysis. Succinylcholine is an ideal agent for "rapid sequence induction" due to its rapid onset (45-60 sec) and short duration of action (8-10 min), which allows for return to normal breathing before the patient becomes hypoxic. Hyperkalemia is a major side effect of succinylcholine due to increased release of potassium from intracellular stores. Therefore, it is contraindicated in any condition that is associated with hyperkalemia due to unregulated or excessive cellular potassium release. Severe hyperkalemia, occasionally leading to cardiac arrest, has been seen in patients who receive succinylcholine and have major denervation injuries, spinal cord transection, stroke, multiple site trauma/crush injuries, extensive burns, prolonged immobility with disease, rhabdomyolysis, and myopathies associated with elevated CK levels. The exact mechanism of succinylcholine induced hyperkalemia in patients with myotonia and muscular dystrophies is unknown, but may be the result of muscle membrane fragility, leading to excessive potassium release. Other contraindications of succinylcholine include personal or family history of malignant hyperthermia, ocular surgery, penetrating eye injuries, closed-angle glaucoma, and genetically determined disorders of plasma pseudocholinesterase. Answer A: Succinylcholine results in an increase in intraocular pressure (IOP) of 5 to 15 mmHg and, therefore, is contraindicated in any patients who are at increased risk for or have pre-existing IOP elevation. This includes patients who have closed-angle glaucoma, open-eye/ penetrating eye injuries, or recently underwent ocular surgery. Cataracts does not cause increased IOP and is not a contraindication for succinylcholine use. Answer B: Succinylcholine is rapidly metabolized by plasma cholinesterase and excreted in the urine. Cholinesterase is produced in the liver and caution should be used when administering succinylcholine to patients with extensive liver disease. However, most patients with cirrhosis still produce adequate amounts of this enzyme and only experience a slight, clinically unimportant increase in the duration of action of succinylcholine. Therefore, cirrhosis is not considered a contraindication for succinylcholine use. Answer C: Serum potassium increases by approximately .5 mEq/L after injection of succinylcholine in a healthy adult. However, severe hyperkalemia resulting in cardiac arrest has been observed in patients with underlying conditions that predispose them to excessive cellular potassium release. Therefore, hyperkalemia, not hypokalemia, is a contraindication for succinylcholine use. Answer E: Patients at risk for severe succinylcholine induced hyperkalemia are those with underlying conditions that result in excessive or unregulated cellular potassium release. Hyperkalemia seen in patients with renal disease is due to impaired potassium excretion. Patients with renal failure who receive succinylcholine do not have a greater increase in potassium levels and the mild elevation of .5 mEq/L usually does not place their serum levels in the toxic range. Therefore, renal insufficiency is not considered a contraindication for succinylcholine use and it is considered safe in normokalemic renal-failure patients. Bottom Line: Succinylcholine is a depolarizing neuromuscular agent that is used for rapid sequence induction. Major side effects of the drug include malignant hyperthermia, increased intraocular pressure, and hyperkalemia. Therefore, contraindications are ocular surgery, penetrating eye injuries, closed-angle glaucoma, history of acute malignant hyperthermia, myopathies associated with elevated serum creatine kinase and trauma that may result in rhabdomyolysis (e.g., burns, crush injuries).

A 35-year-old male presents to the emergency department with complaints of a rash on his finger. History reveals no exposures to new detergents or soaps. The patient denies any previous episodes and allergies. Vital signs include a heart rate of 72/min, respirations of 14/min, blood pressure of 135/86 mmHg, and oxygen saturation of 99% on room air. Physical exam reveals a group of painful, non-pruritic vesicles on an erythematous base on the distal aspect of the 5th digit. You note paravertebral hypertonicity from T1-4 and from T11-L2 that is tender to palpation. What is the most likely causative agent? A. Aeromonas spp. B. Streptobacillus moniliformis C. drug reaction D. herpes simplex virus E. urushiol inoculation

The correct answer is: D The patient has herpetic whitlow which is a result of inoculation of the herpes simplex virus (HSV) through an open wound of the hand or finger. This can occur most commonly with HSV-1 from autoinoculation from sucking on a finger with an open wound while having a herpes labialis infection. Children are prone to this as well as healthcare workers. This occurs in healthcare workers from handling the oropharyngeal secretions of a patient with herpes labialis without proper personal protection, such as gloves. Another possible means contracting the virus is through a "fight bite" or human bite injury by someone with an active oral herpes infection. In the general adult population HSV-2 is more common from autoinoculation from genital secretions. Incubation time is usually 2-20 days and resolution occurs 10-14 days after presentation of symptoms. Viral shedding continues another 5-7 days and then complete resolution occurs. While the infection is self-limited oral or topical acyclovir may shorten the course. These infections require no further emergent treatment and an incision and drainage would be contraindicated as this is not associated with an abscess. TART changes from T1-4 are representative of viscerosomatic reflexes from the upper extremity in this case. Additionally you see TART changes from T11-L2 which may be viscerosomatic reflexes from the genitals, suggesting that this patient may have active genital herpes as well. Answer A: Aeromonas spp infections are associated with freshwater contamination. This can be from either an open wound exposed to freshwater or an injury sustained from a fish bite. Infection with this agent would result in a local cellulitis, abscess or possible furuncle. Treatment of choice would be with a fluoroquinolone or sulfamethoxazole-trimethoprim. Answer B: Streptobacillus moniliformis is the causative agent responsible for Rat Bite Fever. This can occur after sustaining a bite from a rat, squirrel, mouse or gerbil and symptoms include rigors with fever, migratory polyarthralgia, and a maculopapular petechial or purpuric rash. Treatment of choice is with penicillin or if allergic, doxycycline or tetracycline. Answer C: While this should always be in the differential for a new rash, clues to this in the question stem would have included history revealing that the patient was recently started on some new medication. These can be very frustrating to diagnose as they can have an insidious onset of days to weeks. Rashes can appear nearly anywhere on the body and are typically pruritic. Answer E: Urushiol is the causative agent in Rhus dermatitis or poison ivy dermatitis. The rash would also present as vesicular, however, they would be pruritic and non-painful. Treatment for this would be a course of topical (for milder cases) or oral (for severe cases) of glucocorticosteroids. Desiccating agents such as colloidal oatmeal baths or calamine lotion may provide relief of the pruritus but will likely not shorten the course of the dermatitis as the steroids would. Bottom Line: Herpetic whitlow is caused from inoculation of an open wound on the hand or finger and course may be shortened with topical or oral acyclovir.

A 74-year-old male presents to the emergency department with chief complaint of chest pain. The chest pain started earlier today and is located in his left chest with radiation to the left side of the neck and shoulder. It is described as moderate as a dull pressure. He occasionally gets similar pain with heavy exertion when going upstairs, however, he developed it today by just walking around the first floor of his home. The pain recurred four times and was relieved by rest. He does not currently have any pain, however he did have associated nausea, shortness of breath, lightheadedness and diaphoresis. Later, he called 911 because he was concerned that his chest pain occurred multiple times today with minimal exertion. He takes aspirin on a daily basis due to history of coronary artery disease status post two stents. Past medical history is significant for hypertension, hyperlipidemia, and type II diabetes. Family history reveals that his father died at age 64 secondary to a heart attack. This patient's TIMI score is A. 2 B. 3 C.4 D. 5 E. 6

The correct answer is: D The patient in this example is older than 65, has experienced more than two anginal episodes in the last 24 hours, has taken aspirin in the last seven days, has a history of known coronary artery disease, and has three risk factors for coronary artery disease including hypertension, hyperlipidemia, and type II diabetes. Note that the patient's father had an MI after age 55 so that does not count towards one of his risk factors for coronary artery disease. However, he already has three risk factors for coronary artery disease including hypertension, hyperlipidemia, and diabetes. Therefore the patient in this example has a TIMI score of 5. The TIMI risk score is a prognostication tool used in patients with unstable angina or NSTEMI. It categorizes a patient's risk of death and ischemic events and helps clinicians with therapeutic decision-making. A patient gets a point of one for each of the following: aspirin use in the last seven days ST changes of at least 0.5 mm on EKG elevated serum cardiac biomarkers age greater than 65 known coronary artery disease (coronary stenosis greater than or equal to 50%) at least two angina episodes within the last 24 hours at least three risk factors for coronary artery disease such as: hypertension, diabetes, current cigarette smoker, family history of premature coronary artery disease (coronary artery disease in male first-degree relative or father less than 55, or female first-degree relative or mother less than 65) The higher the TIMI score the higher the risk at 14 days of: all-cause mortality, new or recurrent MI, or severe recurrent ischemia requiring urgent revascularization. A mnemonic for remembering the factors of the TIMI score is AMERICA: Age greater than 65, Markers (elevation of cardiac biomarkers), EKG, Risk factors, Ischemia (two or more anginal events over the past 24 hours), Coronary artery disease (prior coronary stenosis of 50% or more), Aspirin use within the last seven days. Answers A & B & C & E: Please see the correct answer for the calculation. Bottom Line: The TIMI score is a tool used to determine all-cause mortality, new or recurrent MI, or severe recurrent ischemia requiring urgent revascularization. The higher the TIMI score, the higher the risk.

A 22-year-old male presents to the emergency department with shortness of breath and right-sided chest pain after a car accident. Physical examination reveals tachycardia and decreased breath sounds on the right side. He has a seat belt sign across his chest. A portable chest radiograph is obtained as shown in the exhibit. The most likely diagnosis is A. atelectasis B. mediastinitis C. pericarditis D. pneumothorax E. rib fracture

The correct answer is: D The patient in this example suffered a blunt trauma to his chest. His physical exam findings are suggestive of a pneumothorax including decreased breath sounds on the right side of his chest wall, hypoxia, tachypnea, and tachycardia. Evaluation of a trauma patient with a blunt chest injury includes chest x-ray. The chest x-ray pictured here displays a large right-sided pneumothorax. Findings on chest x-ray suggestive of a pneumothorax include: mediastinal shift, subcutaneous air, a low lateral diaphragm on the side of the injury (deep sulcus sign), an absence of lung markings in the periphery of the lung field. Note also that CT scan of the chest may pickup called pneumothoraces. Treatment for small pneumothoraces may be treated with 100% non-rebreather mask with a repeat chest x-ray to document resolution of the pneumothorax. However, large pneumothoraces require insertion of a chest tube. If a patient with a untreated pneumothorax is intubated, the simple pneumothorax can convert to a tension pneumothorax and lead to the patient having cardiovascular collapse. Also note that if the lung does not really expand after insertion of a chest tube consider the possibility of a bronchial injury or a large tear in the lung parenchyma. Disposition of patients with pneumothoraces requires admission and trauma surgeon consult. Answer A: Atelectasis is the collapse of the alveoli of the lungs. It can be caused by obstruction, loss of surfactant, pleural effusion or pneumothorax, a space occupying lesion, cicatrization atelectasis/parenchymal scarring (secondary to necrotizing pneumonia, or granulomatous disease). Atelectasis can cause hypoxia. Treatment depends on the underlying cause, but prevention is key which includes ambulation, deep breathing and use of incentive spirometry. Treatments include positive airway pressure, oxygen supplementation, use of bronchodilators to thin secretions, mucolytics, chest physiotherapy, and bronchoscopy to clear obstructions. Answer B: Mediastinitis is a rare but life-threatening condition that usually occurs after medical procedures such as cardiac surgery, endoscopy, or bronchoscopy. It can also occur from trauma, upper respiratory tract infections or dental infections. Risk factors include factors that cause immunosuppression such as malignancy, autoimmune disorders, diabetes and illicit drug use. The most common ideological agents after cardiac surgery includes staph aureus and staph epidermidis. Other etiologic agents include mixed infections including Pseudomonas and anaerobic organisms which occur from non-iatrogenic infections. Symptoms include chest pain which radiates to the back or neck, shortness of breath, confusion, sore throat or dental pain if otogenic, or drainage from a surgical site. On exam the patient will likely be septic appearing, febrile, have subcutaneous emphysema, there may be a Hamman sign (systolic crunching sound), or erythema/swelling around the surgical site. Mediastinitis is diagnosed best by CT thorax which will show pneumomediastinum, however, there may be air fluid levels in the pre-cervical region and retro pharyngeal region along with para tracheal soft tissue swelling. Treatment includes surgical consult for possible debridement, supportive care, airway management, and broad-spectrum antibiotics for coverage of MRSA, Pseudomonas and oral/G.I. flora. Answer C: There are multiple possible causes of pericarditis which include: idiopathic, viruses, bacteria, tuberculosis, Rickettsia, parasitic, medications (INH, hydralazine, procainamide), systemic diseases (rheumatic fever, scleroderma, SLE), post-MI (Dressler syndrome), post-traumatic, uremia, and malignancy. In pericarditis there is inflammation of the pericardium which leads to pericardial thickening and effusion. Symptoms include substernal chest pain worse with deep inspiration and laying flat, fatigue, intermittent fever. On exam the patient is likely to have a pericardial friction rub which is easier to auscultate when the patient is leaning forward. There are multiple tests used to evaluate a patient in whom you suspect pericarditis including: EKG, chest x-ray, echocardiogram, and various lab tests including ESR, cardiac enzymes, CBC (leukocytosis) and BUN/Cr (elevated in uremic patients). In EKG evaluation of patients with pericarditis there are three stages. Stage I shows within the first hours to days and shows PR depression and diffuse ST elevation with concave up segments. Stage II shows normalization of the ST and PR segments, stage III shows diffuse T-wave inversions, and stage IV shows a normal EKG. An echocardiogram may show pericardial effusion. A chest x-ray is usually nonspecific but may show cardiomegaly if there is a pericardial effusion present. Treatment includes treating the underlying etiology, aspirin or NSAIDs for anti-inflammatory effects and pain control, steroids if infectious agents ruled out, and admission. Answer E: Although rib fractures are associated with blunt chest trauma and with pneumothoraces, the patient here does not display a rib fracture on chest x-ray. Bottom Line: Blunt and penetrating chest trauma can lead to a pneumothorax which will present as chest pain, shortness of breath and diminished breath sounds on the side of the pneumothorax. Diagnosis is mainly by chest x-ray. Treatment includes chest tube insertion.

A 34-year-old male presents to the emergency department after sustaining a seizure. Upon presentation the patient appears to have lost bladder continence and is alert and oriented only to self. Vital signs reveal a temperature of 38.6°C (101.5°F), a heart rate of 115/min, respiratory rate of 26/min, and a blood pressure of 102/68 mmHg. Later, the patient is found to be unresponsive and rigid. Intravenous lorazepam is provided without relief. His symptoms continue for a total of 12 minutes. The most likely diagnosis is A. heat stroke B. hemiballismus C. malingering D. status epilepticus E. Todd's paralysis

The correct answer is: D The patient is found after observation in the emergency department to be in status epilepticus. Status epilepticus can occur in any patient with a known seizure disorder or can present as a first time event. It is defined as continuous or intermittent seizures for more than 5 minutes without recovery of consciousness. The 5 minute mark has been established because after 5 minutes of continuous seizure activity, the seizures will be much less likely to spontaneously terminate, be controlled by antiepileptic drugs and more likely to cause neuronal damage. For any patient a brief history and physical must be obtained to work up potentially treatable causes. The patient should receive 2 large-bore IV's followed immediately by IV benzodiazepines such as lorazepam or diazepam. If this is not successful in breaking the seizure phenytoin or fosphenytoin are indicated next in the treatment algorithm. Should the status continue past 20-30 minutes the patient is in real danger of suffering from major neuronal damage. In addition they may have severe hypoxia, metabolic acidosis, hyperthermia and hypoglycemia. Neurotoxic excitatory mediators and calcium are released which can send the patient into cardiac dysrhythmias, rhabdomyolysis and pulmonary edema. If all other efforts fail after 20-30 minutes anesthesia can be induced to treat refractory status epilepticus. Midazolam or propofol infusions are administered for 12-24 hours. If the patient has tonic-clonic movements and intubation is required neuromuscular blocking agents would be indicated and will facilitate ventilation. Succinylcholine would be a relative contraindication in these patients as rhabdomyolysis and hyperkalemia may be found in these patients. Answer A: Heat stroke is found in the patient who has already been suffering from heat exhaustion; it is the next progression. The cardinal features of heat stroke include hyperthermia (>40°C, > 104°F) and altered mental status. A common misconception is that anhidrosis is a key point in the diagnosis of heat stroke. Many patients will still exhibit the ability to sweat and meet criteria for heat stroke. The central nervous system is especially vulnerable in heat stroke. Virtually any symptomology can occur ranging from ataxia, to confusion to seizure activity. There are two types, exertional and nonexertional and differentiating between the two is clinically irrelevant as all of these patients need rapid cooling with organ system preservation. There are no diagnostic tests to diagnose heat stroke and the diagnosis is made purely on history and clinical presentation. Answer B: Hemiballismus is the involuntary movement of the limbs. It stems from a pathologic process in the subthalamic ganglia found in the basal ganglia. It is a flailing, ballistic undesired reaction of the limbs. An acute cerebrovascular accident is the most common cause for hemiballismus. Other causes include: amyotrophic lateral sclerosis, traumatic brain injury, nonketotic hyperglycemia, vascular malformations (resulting in a stoke), neoplasms and finally as a complication of HIV. Dopaminergic blockade is thus far the most effective means of treating hemiballismus and it is not known the exact mechanism of how this works. Answer C: Malingering is always a consideration and should always be on the differential diagnosis. Patients may have many motivations ranging from drug seeking behavior to attention seeking from a loved one. Due to the popularization of seizures in mainstream media many patients believe they are familiar enough and can fake the seizure; what is depicted on TV is usually tonic-clonic and as such malingering patients will almost always exhibit this type of seizure. The physician should be able to differentiate actual seizures from pseudoseizures. The physician must be able to properly diagnose a seizure and differentiate the symptoms. Bystanders often misjudge what they are seeing in the heat of the moment and history may not always be accurate. Loss of continence is generally a good indication of an actual seizure, as is accidentally biting the tongue. If the patient withdraws from painful stimuli during an active seizure, is talking while having tonic-clonic movements, or has voluntary control over eye movement (for example they look at you when you call for a foley catheter or for them to be intubated) they are most certainly malingering. Answer E: Todd's Paralysis is transient post-ictal paralysis. Patients may suffer from this for up to 36 hours post seizure activity but typically disappears fairly quickly. It is characterized by a focal deficit of unknown etiology. The paralysis is typically unilateral and on average will last up to 15 hours. It may also be bilateral and affect speech and a patients vision. It can be nearly impossible to differentiate from a possible transient ischemic attack. If a patient should present with these symptoms a stroke must be ruled out. Bottom Line: Status epilepticus is defined as seizure activity lasting longer than 5 continuous minutes or seizures that are refractory to anti-epileptics. It must be treated with IV/IM benzodiazepines and if persisting longer than 20 minutes anesthesia and intubation must be considered. I

A 25-year-old female presents to the emergency department with "flu-like symptoms." History reveals that she underwent a surgery two days ago for recurrent sinusitis. Vital signs reveal a rectal temperature of 39.0°C (102.2°F), heart rate 115/min, respirations 24/min, blood pressure 95/70 mmHg, and an oxygen saturation of 98% on room air. Physical exam reveals a female who is disoriented with a diffuse hyperemic blanching rash. She has a surgical dressing around her nares that appears dry. IV crystalloids are promptly started. Which of the following is the most appropriate initial step in management? A. CT scan of the maxillofacial sinuses B. evaporative cooling C. IV metronidazole D. remove the surgical dressing E. serum lactic acid level

The correct answer is: D The patient needs to have her surgical dressing removed to promptly evaluate her nares for any retained nasal packing after her sinusitis surgery. Nasal packing placed after surgery, as in the case of this patient, or potentially to tamponade epistaxis predisposes the patient to developing toxic shock syndrome (TSS). TSS is a toxin mediated process characterized by high fever, profound hypotension, diffuse erythroderma, mucous membrane hyperemia, pharyngitis, diarrhea and constitutional symptoms. This patient is hemodynamically unstable and given her history, Toxic Shock Syndrome is the most likely diagnosis and must be excluded. The nasal packing should be promptly removed as it is the main source of the infection and hence, the toxins. As she is hypotensive crystalloid fluids are indicated in stabilizing her shock and improving her blood pressure. These patients can be profoundly hypotensive due to the vasodilating effects of the toxin. It can rapidly progress into multi-organ system failure resulting in renal failure and shock. In more recent years cases of TSS attributed to tampon usage has greatly decreased due to advances in the manufacturing process. While there is no definitive laboratory marker for diagnosis, a clinical diagnosis must be made based on the patients symptoms and history. The majority of cases (~90%) are female yet males have a 3.3 times greater risk of mortality than women whom have contracted menstration-related TSS. Most cases are associated with S. aureus species producing the Toxic Shock Syndrome Toxin-1 (TSST-1) with a smaller subset of the disease process being attributed to Group A Streptococcal species. Streptococcal TSS is similar in course and presentation to TSS except that it generally arises from a severe soft tissue infection. Answer A: Imaging is rarely the correct response for initial management. This patient is exhibiting signs of shock as she is tachycardic and hypotensive. While imaging may in some cases be needed urgently, this patient will need fluids and to be resuscitated before imaging can be done. CT scan of the maxillofacial region would show the nasal cavity and sinuses and would show any retained nasal packing or any source of infection. However, for patients where Toxic Shock Syndrome is a consideration a quick visual inspection of potential mucus membrane cavities such as the nares and vagina must be inspected to rule out an infectious nidus. Answer B: While this patient is exhibiting a fever of 39°C, evaporative cooling is not yet indicated. For this patient IV fluids to maintain appropriate blood pressure and ensuring hemodynamic stability is the first requirement. IV fluids alone and treatment of the infection will help to reduce the fever in this patient. Evaporative cooling involves the same mechanism the body naturally uses to cool itself. Water mist is sprayed onto the bare skin with a fan blowing air over the mist and allowing the body to release heat by evaporating the water. Evaporative mist cooling is not indicated until the body's core temperature is >40°C. Answer C: IV Metronidazole is indicated to use for anaerobic infections, sexually transmitted diseases (Trichomonas sp), amoebic infections and Giardia infections. The mechanism of action for metronidazole is inhibition of nucleic acid synthesis by disrupting DNA synthesis. As this patients symptoms are being caused by an aerobic organism, Staphylococcus aureus, this agent would not be appropriate for this patient. Only after cultures are quickly obtained should broad spectrum antibiotics be started. Antibiotics do not alter the outcome of the acute illness as it is toxin mediated, they are given to eradicate the toxin producing bacteria. A β-lactamase stable antimicrobial, such as nafcillin, oxacillin or a cephalosporin with β-lactamase stability, should be used as the infection may reoccur. Answer E: Toxic Shock Syndrome (TSS) can cause rapid multi-organ system failure and certain serum markers would help demonstrate this. A serum lactic acid level will help identify the severity of sepsis but should not be performed as an initial treatment but rather as part of an overall thorough workup. Lactic acid levels >4 mmol/L indicate severe sepsis and have a corresponding higher risk of mortality. Normal lactic acid (lactate) levels are between 0.5-1.5 mmol/L. A complete metabolic panel can be used to show blood urea nitrogen and creatinine which may be used to demonstrate acute renal failure. In addition aspartate aminotransferase (AST), alanine aminotransferase (ALT) and alkaline phosphatase (AP) can be used to monitor for hepatic failure. While these may be helpful to obtain initially to assess the patient's overall clinical picture, the patient needs to be hemodynamically stable and have her source of infection addressed. Bottom Line: Toxic Shock Syndrome has more possible presentation than just a retained tampon and other possible sources must be identified and promptly treated.

A patient presents to the emergency department with chest pain and is found to have an acute myocardial infarction. Which of the following would be an indication for thrombolytic therapy? A. frequent unifocal premature ventricular contractions B. new onset 2nd degree AV block Wenckebach C. new onset first degree AV block D. new onset left bundle branch block E.new onset ST depression

The correct answer is: D The standard 12-lead ECG is the single best test to identify patients with AMI upon ED presentation and should be obtained and interpreted within 10 minutes of presentation for patients with symptoms suggestive of myocardial ischemia. Patients with acute myocardial infarction by definition have heart muscle cell death, which is reflected with a rise in cardiac biomarkers. Despite elevated cardiac markers, a patient with acute MI can present with a variety of ECG findings. These include pathologic Q waves, ST elevations/depression or new onset bundle branch blocks. Patients with ST elevation myocardial infarction (STEMI) should receive prompt reperfusion with either percutaneous coronary intervention (PCI) or fibrinolytic (thrombolytic) therapy. Patients without ST elevations, but with elevated cardiac biomarkers (NSTEMI) are typically treated medically (e.g. aspirin, heparin, clopidogrel, statin) without thrombolytics. The exception is with new onset left bundle branch block (LBBB), as new LBBB in the context of chest pain is traditionally considered part of the criteria for thrombolysis. Thrombolytic agents such as streptokinase and alteplase (tPA) work by activating the enzyme plasminogen, which clears the cross-linked fibrin mesh (the backbone of a clot). This makes the clot soluble and subject to further proteolysis by other enzymes, and restores blood flow over occluded blood vessels.

A 17-year-old male presents to the emergency department by ambulance after a motor vehicle accident. EMS reports that there was major damage to the car and that there was evidence of damage to the steering wheel. Vital signs reveal a blood pressure of 73/53 mmHg, a heart rate of 144/min, and a respiratory rate of 35/min. He is presently complaining of chest pain and dyspnea. Physical examination reveals a painful sternum, distant heart tones, and distended neck veins. Which of the following is the most appropriate initial step in management? A. chest tube insertion B. consult cardiothoracic surgery for endovascular repair C. needle decompression D. pericardiocentesis E. transesophageal echocardiogram

The correct answer is: D This patient has a cardiac tamponade and vital signs are unstable after a blunt chest trauma. The signs of cardiac tamponade include Beck's Triad which involve: hypotension, jugular venous distention, and muffled heart sounds (this patient has all 3 components of Beck's triad). In a pericardial tamponade there is accumulation of pericardial fluid in the pericardial space to a point that the pressure in the pericardial sac leads to impairment of ventricular filling and thus cardiac output. Diagnosis can be confirmed by transthoracic echocardiography usually performed at the bedside. A bedside transthoracic echocardiogram will display a pericardial effusion in this patient with collapse in diastole of the right ventricle or atrium. Initial stabilization prior to pericardiocentesis includes IV fluids to improve preload. The ultimate treatment though is a bedside pericardiocentesis which needs be performed to prevent cardiovascular collapse. Using bedside echocardiography, one can insert the needle under direct ultrasound guidance to drain the pericardial effusion. Removing a small amount of fluid can improve the patient's hemodynamic status. Answer A: Insertion of a chest tube is indicated after needle decompression to help re-expand the lung in the setting of a tension pneumothorax. A chest tube is the initial therapy for a simple pneumothorax. Signs and symptoms of a tension pneumothorax include hypotension, hypoxia, absent breath sounds, and tracheal deviation away from the affected side. Answer B: Consultation of a cardiothoracic surgeon for endovascular repair is indicated if the patient has an aortic dissection. A classic symptom of an aortic dissection is intense chest pain with radiation to the back between the shoulder blades. The classic pain is described as a "tearing sensation" between the shoulder blades. Other findings include unequal blood pressure in the upper extremities, stroke like symptoms (if the dissection involves one or both of the carotid arteries), absent pulse with associated pain in an extremity (due to dissection into the vascular supply of an extremity), or renal failure or abdominal pain (due to dissection into a renal artery or major intra-abdominal artery. The most common classification for aortic dissection is the Stanford classification. In this classification, the dissection is either called a type A dissection or a type B dissection. A type A dissection is any dissection that involves the ascending aorta and a type B dissection is a dissection that only involves the descending aorta. Management of an aortic dissection includes urgent blood pressure control with beta blockers such as esmolol or labetalol to decrease the heart rate and thus the shear forces on the dissection to prevent propagation of the dissection and strong vasodilators such as nitroprusside or hydralazine. Note though, that the beta blocker must be given first because giving a strict vasodilator first leads to reflex tachycardia which theoretically can lead to increased shear forces on the aorta and propagation of the dissection. Answer C: A needle thoracostomy (needle decompression) is performed when a tension pneumothorax is suspected. A tension pneumothorax is a clinical diagnosis and one should not wait for chest x-ray to perform needle decompression. In a tension pneumothorax the patient is in an obstructive shock pattern. Signs and symptoms include hypotension, tachycardia, diaphoresis, cyanosis, cardiovascular collapse and tracheal deviation secondary to the buildup of air within the pleural space. A needle thoracostomy involves immediate placement of a 14 or 18-gauge angiocatheter in the second intercostal space at the midclavicular line or fourth/fifth intercostal space at the anterior axillary line on the affected side. After needle thoracostomy tube thoracostomy must be placed. Answer E: A transesophageal echo is a test where the patient is sedated and an ultrasound probe was inserted into the esophagus to view the heart. This is a time-consuming test and usually not readily available in the emergency department. It is also a test that has not indicated when the patient has a tenuous cardiorespiratory status. Bottom Line: When a patient presents with a history of trauma to the chest with distended neck veins, shortness of breath and signs of shock, consider a pericardial tamponade. The treatment is prompt pericardiocentesis.

A 42-year-old male presents with right-sided flank pain. Structural examination reveals tissue texture changes and tenderness on the right side from T11-L2 with an exquisite area of tenderness at L2. Urinalysis reveals positive blood and 2-3 leukocytes. The most likely diagnosis is A. cystitis B. prostatitis C. pyelonephritis D. ureterolithiasis E. urethritis

The correct answer is: D Ureterolithiasis can present in many different ways. Classic symptoms include renal colic and hematuria. Others may be asymptomatic or have atypical symptoms such as vague abdominal pain, acute abdominal or flank pain, nausea, urinary urgency or frequency, difficulty urinating, penile pain, or testicular pain. Typically ureterolithiasis does not present with fever, unless the patient has a septic stone which means they have developed an infection secondary to the obstruction. Urinalysis typically shows blood, but not always. Treatment is symptomatic with pain control and antiemetics. A septic stone needs urgent urologic evaluation. In the above scenario, the patient has tissue texture changes on the right from T11-L2 which indicates renal pathology along with right flank pain and hematuria. Ureterolithiasis is the most likely diagnosis in the above scenario. Answer A: Symptoms of cystitis include dysuria, frequency, urgency, suprapubic pain, and/or hematuria. None of these symptoms are listed in the above scenario. Cystitis would not manifest with flank pain unless the patient had pyelonephritis. Treatment of cystitis in men consists of a fluoroquinolone antibiotic or Trimethoprim-Sulfamethoxazole. Answer B: Symptoms of prostatitis include fever, chills, malaise, myalgia, dysuria, irritative urinary symptoms (frequency, urgency, urge incontinence), pelvic or perineal pain, and cloudy urine. Diagnosis is mainly by palpating an edematous and tender prostate on physical exam along with the history. Treatment includes a fluoroquinolone or Trimethoprim-sulfamethoxazole. Answer C: Symptoms of pyelonephritis include cystitis symptoms- dysuria, frequency, urgency, suprapubic pain, and/or hematuria PLUS fever (>38ºC), chills, flank pain, costovertebral angle tenderness, or nausea/vomiting. Pyelonephritis is diagnosed clinically based on pyuria with other systemic symptoms including fever and flank pain. Treatment depends on the severity of the illness. For mild cases, outpatient treatment with a fluoroquinolone should be adequate. For moderate to severe cases, IV antibiotics with either a fluoroquinolone, third generation cephalosporin, or an extended spectrum penicillin. Answer E: Urethritis typically presents with symptoms of dysuria, pruritus, burning, and discharge at the urethral meatus. Causes are usually secondary to sexually transmitted infections. Treatment is aimed at treatment of Gonorrhea/Chlamydia which are the most common causes. Antibiotics should include Ceftriaxone and Azithromycin or Doxycycline respectively. Bottom Line: Hematuria and flank pain without other given symptoms should be concerning for ureterolithiasis.

A 13-year-old male presents with nausea, vomiting, and abdominal pain around his belly button and right lower quadrant. Physical examination reveals tenderness with rebound in the right lower quadrant with associated guarding, approximately a third of the distance between the right iliac crest and the umbilicus. Laboratory studies reveal a WBC of 20 x103/L. The most appropriate next step in management is A. acute abdominal series B. CT scan of the abdomen without contrast C. KUB D.stool guaiac E. ultrasound of the abdomen

The correct answer is: E The patient described above has classic findings of acute appendicitis. Although largely a clinical diagnosis, imaging is part of a standard workup for appendicitis and provides useful information. The goal of any imaging study should be to establish a prompt diagnosis to avoid a negative appendectomy and perforation, as well as to rule out other causes with minimal radiation and cost. Plain radiography is not helpful. Findings are typically nonspecific but may demonstrate a nonspecific bowel gas pattern or adynamic ileus. Graded compression ultrasonography should be the initial imaging modality of choice given its lack of radiation and minimal cost, especially in pregnant females and children. In most adult males and nonpregnant females for whom the diagnosis of appendicitis is not sufficiently clear, abdominal CT may be obtained. Oral contrast medium is recommended for CT of the abdomen and pelvis when investigating GI etiologies. Many centers recommend CT imaging with both IV and oral contrast, although IV contrast may be contraindicated in patients with renal insufficiency or dye allergy. Stool guaiac testing does not aid in making the diagnosis of, or ruling out, appendicitis. Answer A: The acute abdominal series is a common set of plain film radiographs obtained to evaluate bowel gas. It typically consists of an AP supine view, PA erect view, and PA erect chest radiograph. Answer B: Although CT imaging of the abdomen has a high sensitivity for detecting acute appendicitis, ultrasound is the cheaper and safer modality and should be attempted first. If a normal appendix is not visualized on sonography, CT imagining can be performed WITH contrast. Typically both IV and oral contrast is used, but studies have shown that IV contrast alone is sufficient to rule out appendicitis. Answer C: Plain radiography such as a KUB would not be helpful and would most likely show nonspecific findings. Answer D: Stool guaiac testing does not aid in making the diagnosis of, or ruling out, appendicitis. Bottom Line: When evaluating for acute appendicitis, abdominal ultrasound is the safest and least expensive imaging modality and should be used first. CT imaging is useful and should be performed if ultrasound does not rule out acute appendicitis. COMBANK Insight : Pay attention to the question stem when it asks for the most appropriate next step in management. More than one answer choice provided would help make the diagnosis, but only one is the most appropriate as a next step.

A 54-year-old African-American male presents to the emergency department with swollen lips and difficulty talking. History reveals this has never happened to him before. Physical exam reveals red raised lesions that are itchy, with significant edema noted around the oral cavity with an enlarged tongue to the point it is difficult for the patient to talk. Intramuscular epinephrine is given and moderate improvement is noted. The most appropriate pharmacologic treatment is A. C1 inhibitor concentrate B. cetirizine C. fexofenadine D.loratadine E. methylprednisolone

The correct answer is: E This patient has angioedema. Angioedema may have different causes; such as histamine mediated angioedema, bradykinin mediated angioedema, hereditary angioedema, and acquired angioedema. The type that is secondary to ACE-inhibitors is bradykinin mediated which is not responsive to antihistamines. In this case it is more likely to be histamine mediated as evidence of the urticarial rash noted on the patient. Hereditary angioedema is less likely due to the lack of recurrent history in this 54-year-old. With histamine-mediated the treatment depends on the severity. As this patient is showing signs of airway compromise it is important to use epinephrine to stabilize the patient. Sometimes another dose is appropriate. In this case since the patient had a moderate improvement the next step would be IV medications; such as IV/IM diphenhydramine, and IV steroids (methylprednisolone). The patient may also benefit from treatment with H2 antagonists such as IV famotidine. Giving PO medications in a patient that has signs of airway compromise would not be appropriate. This is noted by the tongue swelling and difficulty speaking. In cases like this be prepared to intubate as it may be necessary for airway protection. Answer A: C1 inhibitor is used for acute attacks of hereditary angioedema (HAE), which is less likely in this patient due to the lack of recurrent episodes. This type of angioedema is also associated with abdominal pain and vomiting. However, this patient should be screened for the condition considering that he has laryngeal edema. C1 inhibitor is one of the serine proteinase inhibitors that plays a role in the complement and intrinsic coagulation pathway. Those with HAE have a deficiency of C1 inhibitor; replacing this is believed to suppress reactions by inactivating plasma kallikrein and factor XIIa, therefore preventing bradykinin production. Answer B: Cetirizine, fexofenadine, and loratadine are second generation antihistamines (H1 antagonist) that may be used in mild cases without signs of airway compromise. Answers C & D: Cetirizine, fexofenadine, and loratadine are second generation antihistamines (H1 antagonist) that may be used in mild cases without signs of airway compromise. Bottom Line: For histamine mediated angioedema, determine the severity. Remember to be concerned about the airway whenever there is tongue involvement. If this is the case epinephrine may be used and IV/IM antihistamines and IV steroids would be given to prevent worsening or recurrence of symptoms.

A father brings his 5-year-old son to the emergency department after a metal polishing acid splashed in the child's face. The child is crying loudly and is rubbing his eye which appears injected. No other lesions or burns are visible on his face. The patient is prepped and irrigation with water is started on the affected orbit. Which of the following is the next best step in management? A. alkaline solution irrigation B. ophthalmology consultation C. patch the affected eye D. pH measurement E. topical ciprofloxacin drops

The correct answer is: D When the eye is exposed to any chemical with corrosive properties, it should be irrigated with COPIOUS amounts of water for 30 minutes as soon as possible. Use pH paper to monitor the acidity of the tears after irrigation and continue irrigating until the pH normalizes to 6.5-7.5. Although this vignette did not indicate the timeline of exposure, irrigation of the eye is of utmost importance and should occur sooner rather than later. Ultimately, we need more information about this child's physical examination including a visual acuity to make more definitive management options (e.g., ophthalmology consultation or antibiotics). Answer A: Neutralization is rarely a treatment option, as chemicals used for neutralization may release heat or cause damage on their own. Answer B: This patient will likely need to follow-up with an ophthalmologist, however, this does not necessarily require an immediate consultation unless there is concern for a ruptured globe, opacification of the cornea or corneal injury, or significantly decreased visual acuity (which we do not yet have on this patient). Answer C: The injured eye should not be patched with the irritant still inside since it can continue to cause damage and significant scarring which can lead to blindness. The acid needs to be irrigated from the eye as quickly as possible. Patching can be helpful if the eye is painful when left open, especially in children. Answer E: Those with signs of corneal injury should be provided topical antibiotic ointment or drops to provide broad coverage against superinfections, especially if an alkali was the causative agent. Bottom Line: Water irrigation is the best choice for chemical burns, especially those to the eye.

A 24-year-old female presents to the emergency department with lower abdominal pain and vaginal bleeding. Patient states she missed her period last month and assumed she was starting her period yesterday, but the flow is worse than normal. History reveals she is G1P1, has an IUD present, and has a history of prior pelvic inflammatory disease. Vital signs include a blood pressure of 90/53 mmHg and a heart rate of 115/min. Physical exam reveals a moderate amount of vaginal bleeding through a closed cervical os and suprapubic tenderness with palpation. Lab work reveals that a hCG level of 10,000, and a transvaginal ultrasound is unremarkable. What is the next best course of action? A. abdominal CT scan B. KUB C. reassurance that a pregnancy is not large enough to be seen at this time D. repeat hCG level in 48 hours E. salpingectomy

The correct answer is: E Abdominal pain and vaginal bleeding are the most common symptoms of ectopic pregnancy, and it should be suspected in any woman of childbearing age with these symptoms. Risk factors include previous ectopic pregnancy, tubal ligation, use of an intrauterine device (IUD), smoking, PID, multiple sexual partners, and early age of intercourse. B-hCG level and transvaginal ultrasound are used in the evaluation for ectopic pregnancy. If ectopic pregnancy is suspected and the patient is hemodynamically unstable or has signs or symptoms of impending or ongoing rupture (eg, pelvic or abdominal pain or evidence of intraperitoneal bleeding), then surgical intervention is warranted. This patient is at risk for an ectopic pregnancy given her history of current IUD use and history of PID. An hCG of 10,000 should show an intrauterine pregnancy on ultrasound if one exists. In this case the patient is pregnant without evidence of intrauterine pregnancy, which means this is an ectopic pregnancy until proven otherwise. This is a surgical emergency and OB/GYN should be called immediately to take the patient to the OR for exploration. Answer A: A CT scan can evaluate where bleeding in originating from or show other reveal masses or cysts, but this is not an appropriate imaging modality in the setting of an ectopic pregnancy. Answer B: A kidney stone should not cause vaginal bleeding, and still does not explain why there is no intrauterine pregnancy at this time. This is an ectopic pregnancy until proven otherwise. Answer C: The discriminatory zone for hCG levels is 1,500-2,000 IU/L when using transvaginal ultrasound and 6,000 IU/L when using transabdominal ultrasound. This patient's hCG level is 10,000, which means an intrauterine pregnancy should be visualized on ultrasound. Answer D: This patient is at risk for ectopic pregnancy and should NOT be sent home until she has been evaluated by OB/GYN! Sometimes low risk patients who's hCG level is too low for ultrasound evaluation will return every 2 days for serial hCG levels until an intrauterine pregnancy can be visualized on ultrasound. Bottom Line: Risk factors for ectopic pregnancy include previous ectopic pregnancy, tubal ligation, use of an intrauterine device (IUD), smoking, PID, multiple sexual partners, and early age of intercourse.

A 25-year-old female presents to the emergency department after fall where she sustained multiple bruises and a possible fractured forearm. She presents with her husband who insists on being present for the interview and entire duration of her care. While interviewing her, he frequently interrupts and does not let her answer. Physical exam is remarkable for multiple bruises all over her arms and torso as well as a very tender right forearm. Plain film radiographs of the affected extremity are ordered. After some time, the husband leaves the room and you again question her on her injuries alone. She states, "he beat me," but adamantly says that she does not want the authorities notified. What is the most appropriate intervention at this time? A. discuss the incident with both parties present B. notify the Division of Child and Family Services C. notify the local police department D. speak to her husband alone regarding the incident E. treat her injuries and admit for observation

The correct answer is: E Emergency room physicians, nurses and staff are all mandatory reporters for pediatric and elder abuse. This information is state dependent and can be found at the Administration on Aging to learn the requirements for individual states. As this patient is an adult, mandatory reporting is not required and would require their permission. In these situations fear of retaliation of the aggressor is quite common and may limit the patients desire to report them. For this it is reasonable and prudent to admit the patient until arrangements can be made for the patient to get the help they will need for safe discharge, without fear of retaliation. The most dangerous times in an abusive relationship such as this are during the time of abuse disclosure (reporting to the ER for injuries) or during attempts to leave the relationship. It is common practice in emergency departments to post abuse/neglect information in the bathrooms or other private areas where the patient may find it without the knowledge of the abuser. Answers A & D: Confronting the aggressor is never a good idea. They may simply deny any abuse, take your actions out on the patient after discharge or even become aggressive with the staff in the emergency department. It is very important to carefully document these cases including all conversations had with all parties. Only with the patients permission should the authorities be notified. The police can then confront the aggressor and do a formal investigation. Answer B: Division of Child and Family Services (DCFS) is a governmental agency, also known as Child Protective Services (CPS), that is the agency responsible for investigating and if necessary removing a child from an abusive home. Physicians are required to notify this agency in any suspected abuse case of anyone under the age of 18. Different states have different laws regarding the specifics of abuse and the extent to which to report. All 50 states do require that a minimum, a verbal report to the police department be made. Answer C: This patient has declined notification of the police and doing so without her permission or knowledge would be violation of this patient's autonomy. During your assessment you must ascertain if the patient is able to think clearly and make sound decisions. If they are then you must respect their ability to make decisions regarding their own care. If they are of altered mind set or not thinking clearly then the patient may be admitted for further workup and possible psychiatric evaluation. In an adult that has mental deficiencies and is not capable of making their own decisions, this would be a case where the emergency physician would again be a mandatory reporter. Bottom Line: Patients have autonomy to make their own decisions for their care. Physician's are only mandatory reporters for child or elder abuse.

A 56-year-old male presents to the emergency department complaining of abdominal cramping with nausea, vomiting, and diarrhea. Vitals reveal blood pressure of 180/96 mmHg, heart rate 106/min, respiratory rate 18/min, oxygen saturation 98%, temperature 37oC (98.7oF). Physical exam reveals a patient that appears restless with notable signs of piloerection, dilated pupils, rhinorrhea, and tenderness throughout the abdomen. This patient is most likely withdrawing from A. alcohol B. barbiturates C. benzodiazepines D. cocaine E. opiates

The correct answer is: E Opiate withdrawal presents with diffuse abdominal cramping due to the hyperactive bowel. Remember that opiates cause constipation, now remove that and the bowels become hyperactive causing cramps with nausea, vomiting, and diarrhea. The patients may present with muscle cramps, rhinorrhea, sneezing, yawning, lacrimation (tearing), leg cramping, piloerection (goose bumps), and dilated pupils. Treatment is long acting opiates with agents such as methadone. Patients may also benefit from clonidine as evident in the vitals with the hypertension. Answer A: Alcohol withdrawal the patients may present with tremulousness to delirium tremens (hallucinations, diaphoresis, agitation, tachycardia, hypertension, febrile, and/or seizures). Patients who are actively going through alcohol withdrawal need to be watched closely depending on the severity of symptoms. Seizures can be a serious consequence and the patient may need to be admitted to the ICU. These patients can be treated with benzodiazepines to prevent delirium tremens. Answers B & C: Barbiturates and benzodiazepines present similarly to alcohol withdrawal. Seizures can be a serious consequence. These patients may need to be monitored closely. Restarting a benzodiazepine or a barbiturate is the treatment option. Answer D: Cocaine withdrawal presents with the patient in a slower state. Think opposite of cocaine toxicity. The patients may be depressed, hungry, or excessive sleep. Treatment options are conservative measures of observation. Bottom Line: The withdrawal symptoms are the opposite of the intoxication. Opiate intoxication pin point pupils, opiate withdrawal dilated pupils. The contrast of the overdose helps determine the withdrawal phase.

A 45-year-old healthy male presents to the emergency department after he cut his left hand on some barbed wire. His last tetanus shot was 11 years ago and he was up to date prior to that time. Physical exam reveals a small 2 cm superficial laceration to the dorsal aspect of his left hand which appears clean without active bleeding. Does this patient require tetanus toxoid? A. No, he has had 3 tetanus vaccinations in the past B. No, he has had a tetanus booster within the last 15 years C. No, the wound is clean without obvious contamination D.Yes, he needs both tetanus toxoid and human tetanus immune globulin E. Yes, his last tetanus was more than 10 years ago

The correct answer is: E Tetanus prophylaxis: - For minor wounds - Give if last vaccination was >10 years ago or if < 3 doses of tetanus toxoid vaccination previously. - For all other wounds - Give if last vaccination was >5 years ago or if < 3 doses of tetanus toxoid vaccination previously. - Give human tetanus immune globulin along with tetanus toxoid if < 3 doses of tetanus toxoid vaccination previously. Answer A: It does not matter how many times you have had the tetanus vaccination, if it has been >10 years for minor wounds, it needs to be re-administered. Answer B: Tetanus prophylaxis should be given if the patient has had a tetanus vaccination >10 years ago for clean and minor wounds or if the last tetanus vaccination was >5 years ago for all other wounds. Answer C: Even clean wounds need a tetanus vaccination if the last one was >10 years ago. Answer D: You do not need to give immune globulin if the wound is clean and minor. In all other wounds, it must be given if the patient has had <3 previous tetanus vaccinations or is not sure. Bottom Line: The rule is to give tetanus prophylaxis for minor wounds if last dose was >10 yrs ago.

An atopic 34-year old male presents to the emergency department via ambulance after the abrupt development of shortness of breath with associated swelling of his hands, feet, and lips. History reveals he was stung multiple times by wasps while doing yard work. Vitals signs reveal a blood pressure of 110/64 mmHg, respiratory rate of 22/min, and oxygen saturation of 96% on 5L nasal cannula. Physical exam reveals oropharyngeal edema and stridor. What is the most appropriate initial step in management for this patient? A. bolus of intravenous crystalloid solution B. emergent cricothyroidotomy C. intravenous epinephrine D. intravenous glucocorticoids and diphenhydramine E. rapid sequence intubation

The correct answer is: E This patient is presenting in anaphylaxis, most likely secondary to Hymenoptera venom after multiple stings. First and foremost, the patient's airway should be assessed. If marked stridor is present or in respiratory arrest, prompt intubation should be performed to maintain patent airway. While preparing for possible intubation and establishing large bore IV access, the patient should receive intramuscular epinephrine which is the definitive and initial treatment for anaphylaxis. This drug works by decreasing mediator release from mast cells. It also prevents and reverses obstruction to airflow in the upper and lower respiratory tracts and can prevent cardiovascular collapse. Answer A: For a patient that presents in anaphylactic shock, he/she may require intravenous boluses of crystalloid solution. The patient in the vignette is not currently in shock. However, it would be appropriate to start maintenance intravenous fluids. Answer B: On occasion, emergent cricothyroidotomy is required if patient is in respiratory arrest and unable to be intubated. The patient in the vignette may require intubation since he has marked stridor and in respiratory distress, which is indicative of severe laryngeal edema, however emergent cricothyroidotomy is not currently indicated. Answer C: Intramuscular epinephrine is the initial treatment as it can be given quickly and repeated every 10-15 minutes as needed. Intravenous epinephrine is reserved for patients in anaphylaxis who do not respond to multiple rounds of intramuscular epinephrine. Intravenous epinephrine can cause arrhythmias and should be used only when absolutely necessary. Answer D: It is recommended to give intravenous glucocorticoids in anaphylaxis, however this is adjunct to the definitive treatment of intramuscular epinephrine and the onset of action is delayed. Although diphenhydramine is commonly administered in anaphylaxis there is no evidence to support its' role in anaphylaxis. Intravenous antihistamines, both H1 and H2 blockers may be administered and can improve urticarial rash, but do not reduce airflow obstruction in upper or lower airways. Bottom Line: In patients with anaphylaxis and respiratory compromise (hypoxia, stridor, laryngeal edema especially) the decision to intubate should not be delayed. Additionally, intramuscular epinephrine should be provided immediately.

A 35-year-old female presents to the emergency room with sudden onset of palpitations, shortness of breath, and chest pain. She is awake, alert, and oriented with a blood pressure of 130/85 mmHg. An electrocardiogram reveals narrow QRS complexes at regular intervals, but an increased rate at 180/min without any ST segment changes. The most appropriate management at this time is A. adenosine B. amiodarone C. diltiazem D.metoprolol E. vagal maneuvers

The correct answer is: E Vagal maneuvers are the first line therapy for treating supraventricular tachycardia (SVT). SVT is typically caused by hyper-excitability involving the "pacemaker" areas of the atrium, which include the atrioventricular (AV) node and the sinoatrial (SA) node. Vagal maneuvers control heart rate by using the parasympathetic nervous system to delay conduction through the AV node via the vagus nerve, hence the name. The idea is to use baroreceptor response within the proximal aorta or carotids; by increasing pressure in these areas, baroreceptor reflexes cause a compensatory decrease in heart rate. Vagal maneuvers include carotid massage, having the patient hold their breath and bear down, and immersing the patient's face into a tub of cold water. Answer A: Adenosine is a very short acting medication that works by blocking the action of the AV node. When the affects wear off, normal excitability has been reestablished within the AV node and normal rhythmic contractions begin again. Adenosine would be the first step after vagal maneuvers are tried. This treatment would be ineffective if the source of hyper-excitability/re-entry is something other than the AV node. Obviously adenosine should not be used in any patient with pre-existing conduction problems, such as prolonged QT syndrome or any heart block, as it's action of delaying conduction will acutely worsen these conditions. Answer B: Amiodarone is a class III anti-arrhythmic agent that works mainly by slowing down the repolarization phase of conduction via potassium channel blockade. However, amiodarone is unique among anti-arrhythmias in that it has some action at every phase of cardiac conduction and could be classified anywhere from I-IV. This medication is typically used, in conjunction with other therapies, for ventricular fibrillation and ventricular tachycardia, which can sometimes be confused with wide complex SVT. Common tested side effects of amiodarone include pulmonary fibrosis and hypo or hyperthyroidism. As with adenosine, amiodarone should not be used in patient's with pre-existing heart blocks. Answer C: Diltiazem is a class IV anti-arrhythmic which functions by calcium channel blockade which shortens the "plateau" phase of conduction and thus decreases the heart's contractility. This medication is best used in an irregular rhythm narrow QRS complex SVT usually caused by atrial fibrillation or flutter. You would also consider diltiazem after vagal maneuvers and adenosine have failed in the treatment of regular interval narrow SVT. Answer D: Metoprolol is a selective β1 receptor blocker that functions to slow cardiac rate by decreasing sympathetic activity to the heart. As with most beta blockers, metoprolol is a Class II anti arrhythmic agent. Along with diltiazem, metoprolol is mainly used for irregular rhythm narrow complex SVT with atrial fibrillation or flutter and is a common option for chronic suppression of these arrhythmias in conjunction with high blood pressure or congestive heart failure. Bottom Line: Vagal maneuvers are first line therapy for treatment of narrow QRS complex supraventricular tachycardia with a regular rhythm.

Conjunctivitis

The etiology of conjunctivitis includes bacterial conjunctivitis, viral conjunctivitis, allergic conjunctivitis, and neonatal conjunctivitis (ophthalmia neonatorum). Symptoms include clearer watery discharge with an allergic process or viral conjunctivitis, and stringy white mucus with allergic or viral infections and yellow purulent discharge. Preauricular adenopathy is seen with most viral infections including herpesvirus and adenovirus. Treatment options include Gram stain and culture of purulent discharge if bacterial etiology is considered, warm compresses, antibiotic ointment or drops if considering bacterial conjunctivitis, antiviral therapy if herpes likely, and meticulous hygiene.

Uveitis

Uveitis is the inflammation of the uvea. The uvea consist of the iris, ciliary body, and choroid. Uveitis is classified anatomically into anterior, intermediate, posterior, and panuveitis forms. Anterior uveitis is the most common form and is known as inflammation of the anterior chamber in iris. It is also known as iritis or iridocyclitis. Iritis is most commonly caused by trauma and uveitis can be caused by autoimmune diseases, surgery, or infections. Symptoms and signs include: an irregular pupil, decreased visual acuity, ocular pain, blurred vision, headaches, injected sclera, and cells and flare in the anterior chamber seen on slit lamp examination (cells appear like dust particles seen through a movie projector beam and flare appears to be smoke moving through a movie projector beam). Photophobia is a hallmark symptom because light causes the iris to move and if the iris is inflamed then this causes pain. Treatment includes analgesics, treating the infection if present, mydriatics, and consider steroids with an ophthalmology consult.


संबंधित स्टडी सेट्स

A Raisin in the Sun - Acts I-III- Quote Identification

View Set

Public Health - Quiz 2 - Ch. 3, 6, 8, 18, 19, 26, 27, 28

View Set

PSYC 4072 (Exam 3) - Relationships

View Set

Module 1 Study Quizzes - Microbiology

View Set